Exam 1 - Psychiatric/Mental Health Nursing, Mental health nursing Chapter 1-10, Contemporary Psychiatric-Mental Health Nursing (test 1) chaps 1-5 8-10, 13, 23, Psychiatric Nursing - PRELIMS, Midterm: Psychiatric Nursing, PSYCHIATRIC NURSING FINAL, Fi...

Lakukan tugas rumah & ujian kamu dengan baik sekarang menggunakan Quizwiz!

Thematic Apperception Test (TAT) p. 211

Projective personality test in which a series of cards shown one by one with pictures of people in various ambiguous situations. Clients are asked to describe what seems to be happening in the picture, what the people are feeling and thinking, and how the situation that is seen will be resolved.

The nurse is developing a plan of care for a client. Which of the following interventions must the nurse be careful to avoid?

Selecting interventions that conflict with the clients value system

stress

... broad class of experiences in which a demanding situation taxes a person's resources or capabilities, causing a negative effect

Suppression

... defense or coping mechanism in which unacceptable feelings and thoughts are consciously kept out of awareness

antisocial personality disorder (ASPD)

... personality disorder characterized by a pattern of disregard for and the violation of others' rights

Type A personality

...A behavior pattern associated with persons who are highly competitive and compulsive in their work habits; associated with coronary artery disease

Selective inattention

...The process of taking in some sensory stimuli while excluding others

stressor

...The source of stress; the demanding situation

Undoing

...an attempt to counteract the effects of an earlier behavior or thought that is inappropriate or hurtful in the hope that it will balance out. An attempt to take charge--to make things better, and perhaps patch up a relationship

Regression

...is falling back to an earlier psychosexual developmental stage (stages on page 79)

Maintenance Level of Lithium

0.4 to 1.3 mEq/L

Therapeutic range of lithium for maintenance

0.6-1.2 mEq/L

Therapeutic Level of Lithium

0.8 to 1.4 mEq/L

grief, p. 369

A multifaceted reaction to loss with emotional, physical, cognitive, behavioral, social, spiritual, and philosophical components

A client on the inpatient psychiatric unit asks the nurse, Why is the daily schedule so full of activities and why do we have so many rules to follow? Which of the following is the best response by the nurse?

A daily routine helps you keep on track and organize your thoughts. The rules help people live together respectfully.

Identification

A defense mechanism that helps deal with feelings of threat and anxiety by enabling us unconsciously to take on the characteristics of another person who seems more powerful or better able to cope.

Introjection

A defense or coping mechanism closely related to identification; the process of accepting another's values and opinions as one's own

Displacement

A defense or coping mechanism in which a person discharges pent-up feelings on persons less threatening than those who initially aroused the emotion

Rationalization

A defense or coping mechanism in which a person falsifies experience by constructing logical or socially approved explanations of behavior

Identification

A defense or coping mechanism in which a person wishes to be like another person and to assume the characteristics of that person's personality

repression

A defense or coping mechanism in which unacceptable feelings are kept out of awareness

negative symptoms, p. 334

A deficit in functioning; those features of being in the world that are missing with the illness (e.g., motivation, ambition).

folie a deux, p. 335

A delusion shared by two people who are usually emotionally close to each other.

conversion disorder

A disorder characterized by functional impairment without evidence of organic or physiologic involvement; impairment is of psychogenic origin

hypomania, p. 367

A distinct period during which there is an abnormal and persistently elevated, expansive, or irritable mood somewhat less intense than in mania and lasting at least 4 days

Durable power of attorney

A document that can delegate the authority to make health, financial, and/or legal decisions on a person's behalf. It must be in writing and must state that the designated person is authorized to make healthcare decisions.

Living will

A document that provides written directions about life-prolonging procedures to provide instructions when a person can no longer communicate in a life-threatening situation.

derealization

A feeling of disconnection from the environment

depersonalization

A feeling of strangeness or unreality about the self

remotivation therapy, 617

A form of resocialization group. The emphasis is on stimulating interest in the environment and relationships with others. Topics chosen by members of the group and may include world affairs, current local activates, happy experiences.

Postpartum depression

A new mother's feelings of inadequacy and sadness in the days and weeks after giving birth associated with hormonal changes, tryptophan metabolism, and cell alterations

Mental illness p. 5

A psychological group of symptoms, such as a pattern or a syndrome, in which the individual experiences distress (a painful symptom), or disability (impairment in one or more important areas of functioning), or a significantly increased risk of suffering, pain, loss of freedom, or death

The nurse is planning care for the client who presents with frequent reports of multiple physical complaints. Given knowledge of the leading causes of mental disability, the nurse should plan to include further data collection in which of the following priority areas?

Alcohol usage

auditory hallucinations that begin 48 hours after drinking has stopped.

Alcoholic hallucinations

Resistance p. 51

All the phenomena that interfere with and disrupt the smooth flow of feelings, memories, and thoughts in therapeutic work

Focus of childhood depression therapy

Alleviation of symptoms and strengthening of coping skills alongside family therapy

lamotrigine (Lamictal)

Anticonvulsant First line treatment for bipolar depression and is approved for acute and maintenance therapy. A potential life-threatening rash may occur.

Other drugs used together with antipsychotics

Antidepressants Antimanic Benzodiazepines ECT

Cluster C Personality Disorders

Anxious or fearful behavior; rigid patterns of social shyness, hypersensitivity, need for orderliness, and relationship dependency; Avoidant PD, Dependent PD, Obsessive-Compulsive PD

A client has been diagnosed with dependent personality disorder. Which behavior descriptions can the nurse *expect* to assess? A. Anxious, fearful B. Odd, eccentric C. Dramatic, emotional, erratic D. Disoriented, disorganized

Anxious, fearful

cluster suicide

Any excessive number of suicides occurring in close temporal or geographic proximity to each other

The mental health team nurse is having some role issues regarding how best to facilitate client progress toward therapeutic goals. What is the priority action by the nurse in order to aid the team as they assist the client?

Determine personal values, biases, and goals.

Apathetic affect

Disinterest in the environment

A nurse is teaching a group of students about the stigma that is often associated with mental illness. The nurse tells the group that stigma associated with mental illness is about which of the following?

Disrespect

depersonalization disorder

Dissociative disorder characterized by persistent or recurring feeling of being detached from one's body or thoughts; reality remains intact

Schizoid PD

Emotionally detached! Does not seek out or enjoy close relationships. this individual may be able to function in a solitary occupation but shows indifference to praise or criticism from others. Depersonalization may occur.

Inappropriate affect

Emotions are incongruent with the circumstances (ex. laughing when talking about sad issues)

A nurse conducting research is seeking data about outcomes for depressed patients who have been treated with electroconvulsive therapy. The nurse is engaged in the field of A) experimental epidemiology. B) descriptive epidemiology. C) clinical epidemiology. D) analytic epidemiology.

C) clinical epidemiology.

The quantitative study of the distribution of mental disorders in human populations is called: A) mortality. B) prevalence. C) epidemiology. D) clinical epidemiology

C) epidemiology.

The mental health or mental illness of a particular client can best be assessed by considering A) the degree of conformity of the individual to society's norms. B) the degree to which an individual is logical and rational. C) placement on a continuum from healthy to psychotic. D) the rate of intellectual and emotional growth.

C) placement on a continuum from healthy to psychotic.

A nursing diagnosis for a client with a psychiatric disorder serves the purpose of A) justifying the use of certain psychotropic medication. B) providing data essential for insurance reimbursement. C) providing a framework for selecting appropriate interventions. D) identifying information to be placed on DSM-IV-TR axis III.

C) providing a framework for selecting appropriate interventions.

If the patient is having a Parkinson's syndrome, what is your nursing management? a) Withdraw the meds and call the physician. b) avoid abrupt withdrawal and administer Anti-parkinson's drug. c) None of the above

b) avoid abrupt withdrawal and administer Anti-parkinson's drug.

major depressive disorder (MDD)

experience substantial pain and suffering. at least one depressive episode. Lasts at least two weeks. Represents a change from previous functioning. Causes some impairment in social or occupationals functioning.

A client with residual schizophrenia is uninterested in community activities. He lacks initiative, demonstrates both poverty of content of speech and poverty of speech, and seems unable to follow the schedule for taking his antipsychotic medication. The case manager continues to direct his care with the knowledge that his behavior is most likely prompted by A. chronic uncooperativeness. B. personality conflict. C. neural dysfunction. D. dependency needs.

neural dysfunction.

The nurse receives the shift report on a newly admitted client with a history of drug abuse and prostitution. Prior to hospitalization, the clients parental rights were terminated. Which of the following actions best demonstrates the nurses ability to enhance self-knowledge?

The nurse will examine his or her own feelings with regard to this client.

Several antipsychotic medications can cause photosensitivity, most commonly the phenothiazine antipsychotics chlorpromazine and thioridazine. What would be your intervention? a) instruct the patient to use sunblock b) cover eyes and skin from the sun c) give the patient a sunglass to cover his eyes from the sun

b) cover both the eyes and skin

Resulting from positive approval & leading to good feelings about self. a) Bad me b) Good me c) Not me

b) good me

If the patient is showing coarse hand tremors, it is: a) normal - one of the side effects of lithium b) lithium toxicity c) none of the above

b) lithium toxicity

Occurs as the drug is synthesized, released and metabolized & as it acts on the receptor sites of a neurotransmitter system. a) Tertiary Effects b) Primary Effects c) Secondary Effects

b) primary effects

These are the non benzodiazepine drugs except for this one: a) Buspirone b) Reperdone c) Barbiturates

b) repredone

-Emphasized the sense of POWER- the child feels as he attempts to control himself & others -Behavioral traits: they are headstrong & negativistic (favorite word "NO"), active, mobile, curious which make them vulnerable to accident, temper tantrums are common Parallel play a) preschool b) toddlerhood c) infancy

b) toddlerhood

It aims reducing the disability after a disorder. Examples: Rehabilitation programs, Vocational training, After-care support a) Secondary b) Tertiary c) Primary

b)Tertiary

The nurse is working with a client who becomes upset and tells the nurse, Ive decided to give up on finishing my bachelors degree. Which response best reflects the nurses belief that the client is able to find the solution to this concern?

it sounds like you feel it is too much for you to finish now.

A client has been using cocaine intranasally for 4 years. Two months ago she started freebasing. For the past week she has locked herself in her apartment and has used $8000 worth of cocaine. When brought to the hospital she was unconscious. Nursing measures should include A. induction of vomiting. B. administration of ammonium chloride. C. monitoring of opiate withdrawal symptoms. D. observation for hyperpyrexia and seizures.

observation for hyperpyrexia and seizures.

A client brought to the emergency department at the university hospital after PCP ingestion tries to run up and down the hallway. The nursing intervention that would be most therapeutic is A. taking him to the gym on the psychiatric unit. B. obtaining an order for seclusion and close observation. C. assigning a psychiatric technician to "talk him down." D. administering naltrexone as needed per hospital protocol.

obtaining an order for seclusion and close observation.

When the clinician mentions that a client has anhedonia, the nurse can expect that the client A. has poor retention of recent events. B. has weight loss of 10 lb or more from anorexia. C. obtains no pleasure from previously enjoyed activities. D. has difficulty with tasks requiring fine motor skills.

obtains no pleasure from previously enjoyed activities.

Research has indicated that antisocial personality may be characterized by: A. social isolation. B. lack of remorse. C. learning difficulties. D. difficulty with reality testing.

lack of remorse.

Symptoms that would signal opioid withdrawal include A. lacrimation, rhinorrhea, dilated pupils, and muscle aches. B. illusions, disorientation, tachycardia, and tremors. C. fatigue, lethargy, sleepiness, and convulsions. D. synesthesia, depersonalization, and hallucinations.

lacrimation, rhinorrhea, dilated pupils, and muscle aches.

A depressed client tells the nurse "There is no sense in trying. I am never able do anything right!" The nurse can identify this cognitive distortion as an example of A. self-blame. B. catatonia. C. learned helplessness. D. discounting positive attributes.

learned helplessness.

what is the main theme to keep the patient from staff splitting?

limit-setting (consistency among staff is imperative if the limit setting is to be carried out effectively)

The primary goal of milieu therapy for clients with personality disorders is A. manage the affect behavior has on the entire group. B. one-on-one therapy. C. to help the client remain uninvolved with other patients. D. a laissez faire attitude.

manage the affect behavior has on the entire group.

flight of ideas

nearly a continuous flow of accelerated speech w/ abrupt change from topic to topic that are usually based on understandable associations or play on words.

Beck suggests that the etiology of depression is related to A. sleep abnormalities. B. serotonin circuit dysfunction. C. negative processing of information. D. a belief that one has no control over outcomes.

negative processing of information.

Rape Trauma Syndrome

Sleep disturbances, nightmares. Anorexia, amotivation Fears, anxiety, phobias, suspicion. Relationship disruptions. Self-blame, guilt, shame. Lowered self-esteem. Somatic symptoms

Transient depression

Slightly below normal functioning, not seen as dysfunctional

The nurse is teaching the client regarding the concept of mental disorders. In instructing the client, what areas should be covered in the explanation of what impacts the determination of a mental disorder? Standard Text: Select all that apply.

Social conditions Biochemistry Brain structure Culture

The psychiatric nurse is asked to explain the primary focus in the assessment and treatment of mental illnesses during the mid-20th century. Given this request, the nurse would emphasize beliefs and actions related to which of the following?

Social dimension and drug treatment

resocialization groups, 617

Socialization enhancement with elders usually takes place in resocialization groups conducted in senior centers, adult day care, rehabilitation and long-term care facilities. The goal is to facilitate the elder's ability to interact with others and to renew interest in his/her surroundings.

Social boundaries between nurse and client

Society has a certain set of "rules" of how you interact pertaining to personal space, how you greet, etc. **One arm length away is a good starting point (their arms length)**

A suicidal client tells his psychiatric nurse practitioner, My death will benefit my family; Im 83-years-old and have outlived my usefulness. The nurse practitioner knows this is an example of:

Sociocultural theory.

hallucinations, p. 334

Subjective sensory experience that is not actually caused by external sensory stimuli; may be auditory (heard), visual (seen), olfactory (smelled), gustatory (tasted), or tactile (touched)

Former perpertrator of incest fears relapse so he forms a sex addicts anoymous gropp.

Sublimation

The nurse is presenting information to a gatekeeper training class about the people in a suicide victims life, including the most commonly recognized risk factors for suicidal behavior, symptoms of mental illness, and barriers to mental health treatment. The nurse teaches that both parents and friends were equally able to recognize many risk factors, but friends were better than parents at recognizing risk factors for:

Substance abuse.

substance withdrawal syndrome, p. 295

Substance withdrawal associated with distress and/or impairment in important areas of social functioning

During the shift report, a nurse describes a client as crazy. Which approach by the nurse would be best?

Suggest that staff use the term mentally ill.

When teaching students about suicide, the nursing instructor knows to include facts such as:

Suicide is the 11th leading cause of death among Americans, and as many as 44% of psychiatric emergency clients are at increased risk for suicide.

Egoistic suicide

Suicide that occurs when one is not well integrated into a social group.

ABRASION

Superficial layers of the skin are scraped or rubbed away. Area is reddened and may have localized bleeding or serous weeping

What is the ethical obligation of the nurse who has seen a peer divert a narcotic compared with the ethical obligation when the nurse observes a peer to be under the influence of alcohol? A. The nurse should immediately report the peer who is diverting narcotics and should defer reporting the alcohol-using nurse until a second incident takes place. B. Neither should be reported until the nurse has collected factual evidence. C. No report should be made until suspicions are confirmed by a second staff member. D. Supervisory staff should be informed as soon as possible in both cases.

Supervisory staff should be informed as soon as possible in both cases.

Stage I Hypomania

Symptoms not sufficiently severe to cause marked impairment in social or occupational functioning or to require hospitalization

N-Number

Symptoms occur 3-4xday

D-Duration

Symptoms occur 4 or more hours a day

Schizophrenia, undifferentiated type, p. 339

Symptoms of schizophrenia are present, yet client does not have prominent symptoms that match any of the other schizophrenia subtypes.

I-Intensity

Symptoms severe enough to cause extreme disturbances

Phase IV: Residual Phase

Symptoms usually follows an acute phase of the illness, flat affect, impairment in role functioning, impairment increases between episodes

-EFFLEURAGE -TAPOTEMENT -PETRISSAGE/KNEADING

TECHNIQUES USED IN BACK RUB

-EARLY MORNING CARE -MORNING CARE -HOUR SLEEP(HS) OR PM CARE -AS-NEEDED(PRN) CARE

TERMS TO DESCRIBE OF HYGIENIC CARE:

-PEDICULUS CAPITIS (head louse) -PEDICULUS CORPORIS (body louse) -PEDICULUS PUBIS (crab louse)

THREE COMMON KINDS OF LICE

TRUE OR FALSE. Diazepam (Valium) with alcohol is often FATAL.

TRUE

True or false...Prolonged exposure to stressors alters the thyroid system and depresses the immune system.

TRUE

TRUE OR FALSE..In every state - individuals who are a danger to self or others because of a mental disorder can be involuntarily treated.

TRUE...Also if "gravely disabled".

-CLEANSING BATHS -THERAPEUTIC BATHS

TWO CATEGORIES OF BATH GIVEN TO CLIENTS

-LIGHT STROKING -DEEP STROKING

TWO METHODS OF EFFLEURAGE

OCCUPIED BED UNOCCUPIED BED

TYPES OF BED

- COMPLETE BED BATH - SELF-HELP BED BATH - PARTIAL BED BATH (ABBREVIATED BATH) -BAG BATH

TYPES OF CLEANSING BATH

-HOT SITZ BATH -TEPID SPONGE BATH -MEDICATED TUB BATH -ALCOHOL SPONGE BATH

TYPES OF THERAPEUTIC BATH

Seclusion and Restraints

Table 16-2 pages 268-269-270

When considering communication skills, the nurse caring for an older client anticipates that the client will:

Take longer to respond.

dangerousness

The determination of who in a given setting poses a risk for violence and the perception that someone is more likely to be violent is known as dangerousness.

substance withdrawal, p. 295

The development of maladaptive physiologic, behavioral, and cognitive changes that are the result of reducing or stopping the heavy and regular use of a substance

Perception p. 185

The experience of sensing, interpreting and comprehending the world in which one lives. It is a highly personal and internal act.

THE CROWN

The exposed part of the tooth, which is outside the gum.

dysfunctional grieving, p. 369

The failure of an individual to follow the course of normal grieving to a point of resolution

identity diffusion

The failure to integrate various childhood identifications into a harmonious adult, psychosocial identity

These are generally used to treat major depressive disorder (MDD), otherwise known as depression and it can also work as an anti-smoking agent. a) Atypical Antidepressants b) Tricyclic Antidepressants c) MAOIs

a) Atypical Antidepressants

What medication should be given when the patient has a haldol poisoning? The most prominent feature of haloperidol overdose is severe extrapyramidal reactions such as tremor, rigidity and an intense feeling of physical restlessness, referred to as akathisia. a) Biperiden b) Trazodone c) Naloxone

a) Biperiden

uses a time-limited, goal-oriented-problem-solving, here-and-now approach. Focus is to solve presenting problem & not on why the problem occurred a) Brief Cognitive Therapy b) Behavior Therapy c) Brief Interpersonal Therapy

a) Brief Cognitive Therapy

-Able to understand cause & effect in concrete situations but cannot yet reason hypothetically -Reversibility = refers to completion of certain operation in reverse order & ending up the same -Accommodation developed = modifies ideas to fit reality a) Concrete operations b) Pre-Operational c) Formal operations

a) Concrete operations

what type of factor influences mental health that includes a person's biologic makeup, autonomy and independence, self-esteem, capacity for growth, vitality, ability to find meaning in life, emotional resilience or hardiness, sense of belonging, reality orientation, and coping or stress management abilities. a) Individual or Genetic b) Interpersonal or Childhood Experiences c) SOCIAL/CULTURAL/LIFE CIRCUMSTANCES

a) Individual or Genetic

First used in 1933 by Manfred Sakel after accidentally discovering that giving too much insulin reduces psychotic symptoms a) Insulin Coma Therapy b) Insulin Challenge

a) Insulin Coma Therapy

Personality development results from interaction with significant others & that the child internalizes approval or disapproval by significant others a) Interpersonal Framework (Sullivan's Theory) b) Psychodynamic Theory c) Psychosexual Theory

a) Interpersonal Framework (Sullivan's Theory)

It is a mood-stabilizing medication used to treat bipolar disorder. It works to decrease the intensity of manic symptoms and decrease the severity of depressive symptoms to help a patient achieve a more stabilized mood. a) Lithium b) Lorazepam c) Flurazepam

a) Lithium

It is one of the antidepressants that used to treat depression, but it is not usually given to patients because this can cause a hypertensive crisis, meaning a severe increase in blood pressure that can lead to a stroke. When patients take this medication, they must avoid high-tyramine. a) MAOI b) Atypical Antidepressant c) TCA

a) MAOI

It is a theory of psychology explaining human motivation based on the pursuit of different levels of needs. a) Maslow's Hierarchy of Needs b) Freud's Psychosexual Stages c) Freud's Psychoanalytic Theory

a) Maslow's Hierarchy of Needs

It is the science that deals with measures to promote mental health, prevent mental illness and suffering and facilitate rehabilitation. a) Mental Hygiene b) Mental Awareness c) Mental Assessment

a) Mental Hygiene

Belittling personal ability, action or response a) Minimization b) Magnification c) Dichotomous Thinking

a) Minimization

It is an enzyme that is involved in removing or breaking the neurotransmitters norepinephrine, serotonin and dopamine from the brain. a) Monoamine Oxidase b) Acetylcholine c) GABA

a) Monoamine Oxidase

-Assessment focuses on client's relationship with family -The therapist reinforces client's self-esteem, goal focused approach, supports positive use of defense mechanism & coping skills. a) Brief Cognitive Therapy b) Behavior Therapy c) Brief Interpersonal Therapy

c) Brief Interpersonal Therapy

These drugs are typical drugs except for this one: a) Chlorpromazine (Thorazine) b) Haloperidol (Haldol) c) Clozapine (Clozaril)

c) Clozapine

"All or nothing" patterns of thought a) Minimization b) Magnification c) Dichotomous Thinking

c) Dichotomous Thinking

After 2 days of taking antipsychotic drug, the patient is experiencing sudden spams or contractions of face, tongue, extraocular muscles, neck (torticollosis), dx in swallowing. a) Parkinson's Syndrome b) Tardive Dyskinesia c) Dystonia

c) Dystonia

Who said this quote? "No one is entirely mentally unhealthy and no one is fully healthy at all times." a) Maslow b) Erikson c) Ebersole and Hess

c) Ebersole and Hess

It is a characteristic of a nurse that sees beyond outward behavior and sense accurately another person's inner experiences. a) Genuineness/Congruence b) Unconditional Positive Regard c) Empathy

c) Empathy

They are neurotransmitters that react with beta receptors in the periphery to modulate pain transmission a) GABAs b) MAOs c) Enkephalins

c) Enkephalins

- Employs logical, mathematical & scientific reasoning. -Has solution to all kinds of problems -Abstract thinking is fully utilized a) Concrete operations b) Pre-Operational c) Formal operations

c) Formal operations

It is an amino acid that functions as the primary inhibitory neurotransmitter for the central nervous system (CNS). It functions to reduce neuronal excitability by inhibiting nerve transmission. Treatment for Anxiety. a) Monoamine Oxidase b) Acetylcholine c) GABA

c) GABA

These are the foods that are rich in tyramine except for this one: a) Aged cheese b) Caffeine c) Guava

c) Guava

It is a framework that focuses on conscious human experiences of here & now a) Behavioral framework b) Personality framework c) Humanistic Framework

c) Humanistic Framework

When the nurse remarks to a depressed client "I see you are trying not to cry. Tell me what is happening." The nurse should be prepared to A. wait quietly for the client to reply. B. prompt the client if the reply is slow. C. repeat the question if the client does not answer promptly. D. seek information from the client's significant others.

wait quietly for the client to reply.

The nursing diagnosis Imbalanced nutrition: less than body requirements has been identified for a client with severe depression. The most reliable evaluation of outcomes will be based on A. energy level. B. weekly weights. C. observed eating patterns. D. client statement of appetite.

weekly weights.

The purpose for a nurse periodically performing the Abnormal Involuntary Movement Scale (AIMS) assessment on a persistently mentally ill client who has schizophrenia is early detection of A. acute dystonia. B. tardive dyskinesia. C. cholestatic jaundice. D. pseudoparkinsonism.

tardive dyskinesia.

When a hyperactive manic client expresses the intent to strike another client, the initial nursing intervention should be A. questioning client motive. B. verbal limit setting. C. physical confrontation. D. seclusion.

verbal limit setting.

Derealization

-is a false perception that the environment has changed. Both of the above can be interpreted as loss of ego boundaries.

methadone (Dolophine)

-synthetic opiate that blocks the craving for and effects of heroin -*Highly Addictive* and produces withdrawal symptoms when stopped -Is the only approved treatment for pregnant opioid addicts -legally prescribed drug to replace street drugs

Vacillation

...(moving first one way, toward a desirable goal, that is talking to the nurse, then moving another way to avoid what is undesirable, that is being perceived as weak or as a bother).

The nurse finds the client crying in the room. The client states, Im so sad and lonely. Im sitting here crying like a baby. Which of the following responses best reflects the nurses sensitivity toward the client?

Are you feeling embarrassed because you are crying?

The nurse is admitting a client from the emergency room. Which of the following would be used to clarify the nurses understanding of the clients chief complaint?

Are you saying you feel that you are bleeding inside?

Which of the following is an example of clarifying a clients verbal response?

Are you saying you feel the medicine is helping you?

When working with suicidal clients, the nurse must be compassionate enough to:

Be able to form an effective link with suicidal clients without being overwhelmed by the clients pain.

Bipolar I disorder

Is experiencing or has experienced a FULL syndrome of manic or mixed symptoms, pseudo euphoria, may have experienced depressive episodes

FLUORIDE

Is often recommended because of its antibacterial protection.

The nurse gathering data from a client admitted to labor and delivery is overheard making the comment, You are lying. You need to tell me the truth so we can do what is best for your baby. The nurses communication is:

Nontherapeutic.

Transient depression symptoms

Normal healthy responses to everyday disappointments in life

refers to information that can be measured and shared

Objective data

The nurse has just taken a continuing education class on assertive communication techniques. Which response best demonstrates that the nurse understands assertive behavior?

No, I cannot work for you on Sunday.

NAPPI Training

Non-Aggressive Psychological and Physical Intervention

Anxiety

Nonspecific, unpleasant feeling of apprehension and discomfort that can be communicated interpersonally and that prompts the person to take some action to seek relief

Psychological factors affecting medical condition (PFAMC)

One or more behavioral factors (such as anxiety or depression) that cause or adversely affect a medical condition

Additional resources are needed to assist instructors and students

Online learning allows learner to participate in learning experiences that otherwise are not available outside of real life. The following are the advantages of distance learning except:

Childhood depression: 6-8

Physical complaints, aggressive behavior, clinging behavior

What environmental factors are implicated in aggression and violence?

Physical crowding, discomfort associated with rise in temp, use of substances, availability of firearms

Which of the following aspects of the structural environment pose the greatest risk to client safety in the hospital setting?

Program schedules posted on bulletin boards behind glass doors

harm reduction, p. 321

Programs that focus on minimizing the personal and social harms and costs associated with the spread of HIV through drug use

Dysthymic disorder

Sad or "down in the dumps" with no evidence of psychotic symptoms, essential feature is chronic depressed mood for most of the day and more days than not for AT LEAST 2 YEARS

The nurse is planning care for a patient with depression who will be discharged to home soon. What aspect of teaching should be the priority on the nurse's discharge plan of care? 1. Pharmacological teaching 2. Safety risk 3. Awareness of symptoms increasing depression 4. The need for interpersonal contact

Safety risk

The age of onset

Adolescence or early adulthood. People who already have sensory deficits may develop schizophrenia as they age. The aging process causes increased isolation and loss of other sensory stimuli

Bleuler's "Four A's" of schizophrenia

Affect-flat, blunt, inappropriate bizarre Association looseness-haphazard and confused thinking Autism-thinking not bound to reality Ambivalence-have two opposing emotions, attitudes, ideas, or wishes toward the same person, situation, or object at the same time

Anger

Affective state experience as the motivation to act in ways that warn, intimidate or attack those who are perceived as challenging or threatening

suicide

The willful act of ending one's own life

What prompts someones reaction?

Their inner experience

Peplau's Nursing Model

Theory that nursing is a human relationship which incorporates psychodynamic nursing role and stages of personality development

The skill most important with psychiatric patients?

Therapeutic communication

When does it become pathological depression?

When adaptation is ineffective

Intrapersonal communication p. 189

When people communicate within themselves.

When are coping strategies considered maladaptive?

When the conflict being experienced goes unresolved or intesifies

Countertransference

When the nurse projects feelings onto a pt from past experiences (negative or positive)

During a staff meeting, a nurse makes the following remark about the clients on the unit, These clients are just trying to avoid the problems of life. They just need to go out and work. Which of the following responses best demonstrates the charge nurses respectful attitude toward this nurse?

When you say that our clients are just avoiding life problems, it sounds like you are frustrated by the needs our clients express. Am I hearing you correctly?

Is based on the findings from a body of research evidence

Which of the following does evidence-based practice in nursing informatics perform, according to a research nurse?

Autonomy vs. Shame

Will

Integrity vs. Despair

Wisdom

Removes self from interaction with others and the environment. Often a sign of depression.

Withdrawal

death anxiety

Worry or fear related to death or dying.

Clonazepam (klonopin) and lorazepam (Ativan)

antianxiety drugs useful in treatment of acute mania in some patients who resist other treatments; also help to manage psychomotor agitation seen in mania.

A 16-year-old has stolen money from his invalid grandmother, uses drugs and alcohol, and frequently beats up acquaintances who disagree with him. Arrested for an assault in which he beat a classmate and caused brain damage, he stated in court "The guy deserved everything he got." The behaviors described are *most* consistent with the clinical picture of A. antisocial personality disorder. B. borderline personality disorder. C. schizotypal personality disorder. D. narcissistic personality disorder.

antisocial personality disorder.

Use of touch

approach with caution. Some people may or may not like it. If unsure, ask permission.

lithium and lamotrigine (Lamictal)

are the first line therapy for bipolar disorder.

These are drugs that relieve insomnia and induce a state of natural sleep. a) Antidepressants b) Hypnotics c) Antipsychotics

b) Hypnotics

A person who has numerous hypomanic and dysthymic episodes can be assessed as having A. bipolar II disorder. B. bipolar I disorder. C. cyclothymia. D. seasonal affective disorder.

cyclothymia.

cognitive-behavioral therapy (CBT)

involves identifying maladaptive cognition and behaviors that may be barriers to a person's recovery and ongoing mood stability. Focuses on medication adherence, early detection and intervention for manic or depressive episodes.

The main characteristic of therapeutic communication...

involves the patient as a person

Therapeutic alliance p. 47

is a conscious, growth-facilitating relationship between a helping person (the psychiatric-mental health nurse) and the client).

Thought disorder

is a frequent inability to sort, interpret, and respond to stimuli in a logical way. It is very hard for them to communicate and relate to others.

The first-line drug used to treat mania is A. lithium. B. carbamazepine. C. lamotrigine D. clonazepam.

lithium.

reflexology, 728

practice of acupressure on particular points of the feet, hands and ears. Offers benefits of a relaxing foot massage.

Commendations

praise; an official award or approval

A nurse is assigned to work with a client with borderline personality disorder. The nurse will need to consider strategies for dealing with the client's A. mood shifts, impulsivity, and splitting. B. grief, anger, and social isolation. C. altered sensory perceptions and suspicion. D. perfectionism and preoccupation with detail.

mood shifts, impulsivity, and splitting.

mood disorders

psychological disorders characterized by emotional extremes

Nursing assessment of an alcohol-dependent client 6 to 12 hours after the last drink would most likely reveal the presence of A. tremors. B. seizures. C. blackouts. D. hallucinations.

tremors.

Eats others cookies, then apologises, then goes and puts names on everyones snacks

undoing

tricyclic antidepressants (TCAs)

used to treat depression/anxiety, moderate anticholinergic side effects; can be lethal in overdose (cardiac)

imagery, 724

uses the healing power of a person's own imagination and positive thinking to create powerful mental pictures or images to reduce stress or promote healing. Some consider form of hypnosis.

guided imagery, 725

using an outside resource such as an actual person who guides the imagery process or a voice on an audiotape or CD--helps to create a series of images.

Non verbal question must be ?

validated

Mild anxiety

Seldom a problem, part of natural human coping process (keeps one productive), coping mechanisms

According to biologic theory, there is considerable evidence that what system, partly under genetic control, may influence the risk for suicidal behavior?

Serotonergic system

Coping strategies

The behaviors people under stress use in struggling to improve their situations

What's the purpose of a professional relationship?

The systematic working-through of troublesome thoughts, feelings, behaviors, with planned evaluation through stages

external and visible to others

(objective).

clonidine (Catapres)

-*NON-addicting* suppressor of opioid withdrawal symptoms -effective somatic treatment when combines with naltrexone (ReVia) - old medication for blood pressure

Nursing Guidelines for Obsessive-Compulsive PD

1. Guard against power struggles with patient. Need for control is very high. 2. Intellectualization, rationalization, reaction formation, isolation, and undoing are the most common defense mechanisms.

Therapeutic range of lithium for acute mania

1.0-1.5 mEq/L

Early Toxic Level of Lithium

1.5 to 2.0 mEq/L

What are the five hierarchy of needs?

5. self-actualization 4. self-esteem 3. love 2. security and safety 1. physiologic

Punitive superego

??????

A child diagnosed with attention deficit hyperactivity disorder will begin medication therapy. The nurse should prepare a plan to teach the family about which classification of medications? a. Central nervous system stimulants c. Antipsychotics b. Tricyclic antidepressants d. Anxiolytics

A Central nervous system stimulants, such as methylphenidate and pemoline (Cylert), increase blood flow to the brain and have proved helpful in reducing hyperactivity in children and adolescents with attention deficit hyperactivity disorder. The other medication categories listed would not be appropriate.

A parent diagnosed with schizophrenia and 13-year-old child live in a homeless shelter. The child formed a trusting relationship with a shelter volunteer. The child says, "My three friends and I got an A on our school science project." The nurse can assess that the child: a. displays resiliency. b. has a passive temperament. c. is at risk for posttraumatic stress disorder. d. uses intellectualization to deal with problems.

A Resiliency enables a child to handle the stresses of a difficult childhood. Resilient children can adapt to changes in the environment, take advantage of nurturing relationships with adults other than parents, distance themselves from emotional chaos occurring within the family, learn, and use problem-solving skills.

Denial

A defense or coping mechanism by which the mind refuses to acknowledge a thought, feeling, wish, need, or reality factor

A client's communication is marked by loose associations and word salad. Dysfunction of which portion of the brain is responsible for these symptoms? A) Cerebrum B) Cerebellum C) Brainstem D) Basal ganglia

A) Cerebrum

A nurse makes an initial visit to a homebound patient diagnosed with a serious mental illness. A family member offers the nurse a cup of coffee. Select the nurse's best response. a. "Thank you. I would enjoy having a cup of coffee with you." b. "Thank you, but I would prefer to proceed with the assessment." c. "No, but thank you. I never accept drinks from patients or families." d. "Our agency policy prohibits me from eating or drinking in patients' homes."

ANS: A Accepting refreshments or chatting informally with the patient and family represent therapeutic use of self and help to establish rapport. The distracters fail to help establish rapport.

A patient in a support group says, "I'm tired of being sick. Everyone always helps me, but I will be glad when I can help someone else." This statement reflects: a. altruism. b. universality. c. cohesiveness. d. corrective recapitulation.

ANS: A Altruism refers to the experience of being helpful or useful to others, a condition that the patient anticipates will happen. The other options are also therapeutic factors identified by Yalom.

Which technique is most applicable to aversion therapy? a. Punishment b. Desensitization c. Role modeling d. Positive reinforcement

ANS: A Aversion therapy is akin to punishment. Aversive techniques include pairing of a maladaptive behavior with a noxious stimulus, punishment, and avoidance training.

A Category V tornado hits a community, destroying many homes and businesses. Which nursing intervention would best demonstrate compassion and caring? a. Encouraging persons to describe their memories and feelings about the event b. Arranging transportation to the local community mental health center c. Referring a local resident to a community food bank d. Coordinating psychiatric home care services

ANS: A Disaster victims benefit from telling their story. Nurses show compassion by listening and offering hope. The distracters identify other aspects of psychological first aid and services on the mental health continuum.

Which nursing intervention has the highest priority for a patient diagnosed with bulimia nervosa? a. Assist the patient to identify triggers to binge eating. b. Provide corrective consequences for weight loss. c. Assess for signs of impulsive eating. d. Explore needs for health teaching.

ANS: A For most patients with bulimia nervosa, certain situations trigger the urge to binge; purging then follows. Often the triggers are anxiety-producing situations. Identification of triggers makes it possible to break the binge-purge cycle. Because binge eating and purging directly affect physical status, the need to promote physical safety assumes highest priority.

Physical assessment of a patient diagnosed with bulimia often reveals: a. prominent parotid glands. b. peripheral edema. c. thin, brittle hair. d. 25% underweight.

ANS: A Prominent parotid glands are associated with repeated vomiting. The other options are signs of anorexia nervosa and not usually seen in bulimia.

A nurse surveys medical records. Which finding signals a violation of patients' rights? a. A patient was not allowed to have visitors. b. A patient's belongings were searched at admission. c. A patient with suicidal ideation was placed on continuous observation. d. Physical restraint was used after a patient was assaultive toward a staff member.

ANS: A The patient has the right to have visitors. Inspecting patients' belongings is a safety measure. Patients have the right to a safe environment, including the right to be protected against impulses to harm self.

In the majority culture of the United States, which individual has the greatest risk to be labeled mentally ill? One who: a. describes hearing God's voice speaking. b. is usually pessimistic but strives to meet personal goals. c. is wealthy and gives away $20 bills to needy individuals. d. always has an optimistic viewpoint about life and having own needs met.

ANS: A The question asks about risk. Hearing voices is generally associated with mental illness, but in charismatic religious groups, hearing the voice of God or a prophet is a desirable event. Cultural norms vary, which makes it more difficult to make an accurate diagnosis. The individuals described in the other options are less likely to be labeled mentally ill.

Select all that apply. A patient referred to the eating disorders clinic has lost 35 pounds in 3 months. For which physical manifestations of anorexia nervosa should a nurse assess? a. Peripheral edema b. Parotid swelling c. Constipation d. Hypotension e. Dental caries f. Lanugo

ANS: A, C, D, F Peripheral edema is often present because of hypoalbuminemia. Constipation related to starvation is often present. Hypotension is often present because of dehydration. Lanugo is often present and is related to starvation. Parotid swelling is associated with bulimia. Dental caries are associated with bulimia. See relationship to audience response question.

Select all that apply. The health care team at an inpatient psychiatric facility drafts these criteria for admission. Which criteria should be included in the final version of the admission policy? a. Clear risk of danger to self or others b. Adjustment needed for doses of psychotropic medication c. Detoxification from long-term heavy alcohol consumption needed d. Respite for caregivers of persons with serious and persistent mental illness e. Failure of community-based treatment, demonstrating need for intensive treatment

ANS: A, C, E Medication doses can be adjusted on an outpatient basis. The goal of caregiver respite can be accomplished without hospitalizing the patient. The other options are acceptable, evidence-based criteria for admission of a patient to an inpatient service.

Select all that apply. Which comments by an elderly person best indicate successful completion of the developmental task? a. "I am proud of my children's successes in life." b. "I should have given to community charities more often." c. "My relationship with my father made life more difficult for me." d. "My experiences in the war helped me appreciate the meaning of life." e. "I often wonder what would have happened if I had chosen a different career."

ANS: A, D The developmental crisis for an elderly person relates to integrity versus despair. Pride in one's offspring indicates a sense of fulfillment. Recognition of the wisdom gained from difficult experiences (such as being in a war) indicates a sense of integrity. Blaming and regret indicate despair and unsuccessful resolution of the crisis.

Select all that apply. Which findings are signs of a person who is mentally healthy? a. Says, "I have some weaknesses, but I feel I'm important to my family and friends." b. Adheres strictly to religious beliefs of parents and family of origin. c. Spends all holidays alone watching old movies on television. d. Considers past experiences when deciding about the future. e. Experiences feelings of conflict related to changing jobs.

ANS: A, D, E Mental health is a state of well-being in which each individual is able to realize his or her own potential, cope with the normal stresses of life, work productively, and make a contribution to the community. Mental health provides people with the capacity for rational thinking, communication skills, learning, emotional growth, resilience, and self-esteem.

A patient diagnosed with anorexia nervosa is resistant to weight gain. What is the rationale for establishing a contract with the patient to participate in measures designed to produce a specified weekly weight gain? a. Because severe anxiety concerning eating is expected, objective and subjective data may be unreliable. b. Patient involvement in decision making increases sense of control and promotes compliance with treatment. c. Because of increased risk of physical problems with refeeding, the patient's permission is needed. d. A team approach to planning the diet ensures that physical and emotional needs will be met.

ANS: B A sense of control for the patient is vital to the success of therapy. A diet that controls weight gain can allay patient fears of too-rapid weight gain. Data collection is not the reason for contracting. A team approach is wise but is not a guarantee that needs will be met. Permission for treatment is a separate issue. The contract for weight gain is an additional aspect of treatment.

Which type of group is a staff nurse with 2 months' psychiatric experience best qualified to conduct? a. Psychodynamic/psychoanalytic group b. Medication education group c. Existential/Gestalt group d. Family therapy group

ANS: B All nurses receive information about patient teaching strategies and basic information about psychotropic medications, making a medication education group a logical group for a beginner to conduct. The other groups would need a leader with more education and experience.

Which level of prevention activities would a nurse in an emergency department employ most often? a. Primary b. Secondary c. Tertiary

ANS: B An emergency department nurse would generally see patients in crisis or with acute illness, so secondary prevention is used. Primary prevention involves preventing a health problem from developing, and tertiary prevention applies to rehabilitative activities.

Complete this analogy. NANDA: clinical judgment: NIC: _________________ a. patient outcomes b. nursing actions c. diagnosis d. symptoms

ANS: B Analogies show parallel relationships. NANDA, the North American Nursing Diagnosis Association, identifies diagnostic statements regarding human responses to actual or potential health problems. These statements represent clinical judgments. NIC (Nursing Interventions Classification) identifies actions provided by nurses that enhance patient outcomes. Nursing care activities may be direct or indirect.

A nurse wants to demonstrate genuineness with a patient diagnosed with schizophrenia. The nurse should: a. restate what the patient says. b. use congruent communication strategies. c. use self-revelation in patient interactions. d. consistently interpret the patient's behaviors.

ANS: B Genuineness is a desirable characteristic involving awareness of one's own feelings as they arise and the ability to communicate them when appropriate. The incorrect options are undesirable in a therapeutic relationship.

A black patient says to a white nurse, "There's no sense talking. You wouldn't understand because you live in a white world." The nurse's best action would be to: a. explain, "Yes, I do understand. Everyone goes through the same experiences." b. say, "Please give an example of something you think I wouldn't understand." c. reassure the patient that nurses interact with people from all cultures. d. change the subject to one that is less emotionally disturbing.

ANS: B Having the patient speak in specifics rather than globally will help the nurse understand the patient's perspective. This approach will help the nurse engage the patient. Reassurance and changing the subject are not therapeutic techniques.

A staff nurse completes orientation to a psychiatric unit. This nurse may expect an advanced practice nurse to perform which additional intervention? a. Conduct mental health assessments. b. Prescribe psychotropic medication. c. Establish therapeutic relationships. d. Individualize nursing care plans.

ANS: B In most states, prescriptive privileges are granted to master's-prepared nurse practitioners who have taken special courses on prescribing medication. The nurse prepared at the basic level is permitted to perform mental health assessments, establish relationships, and provide individualized care planning.

A nurse encounters an unfamiliar psychiatric disorder on a new patient's admission form. Which resource should the nurse consult to determine criteria used to establish this diagnosis? a. International Statistical Classification of Diseases and Related Health Problems (ICD-10) b. Diagnostic and Statistical Manual of Mental Disorders (DSM-5) c. A behavioral health reference manual d. Wikipedia

ANS: B The DSM-5 gives the criteria used to diagnose each mental disorder. The distracters may not contain diagnostic criteria for a psychiatric illness.

A 26-month-old displays negative behavior, refuses toilet training, and often says, "No!" Which stage of psychosexual development is evident? a. Oral b. Anal c. Phallic d. Genital

ANS: B The anal stage occurs from age 1 to 3 years and has as its focus toilet training and learning to delay immediate gratification. The oral stage occurs between birth and 1 year. The phallic stage occurs between 3 and 5 years, and the genital stage occurs between age 13 and 20 years.

A nurse influenced by Peplau's interpersonal theory works with an anxious, withdrawn patient. Interventions should focus on: a. rewarding desired behaviors. b. use of assertive communication. c. changing the patient's self-concept. d. administering medications to relieve anxiety.

ANS: B The nurse-patient relationship is structured to provide a model for adaptive interpersonal relationships that can be generalized to others. Helping the patient learn to use assertive communication will improve the patient's interpersonal relationships. The distracters apply to theories of cognitive, behavioral, and biological therapy.

A patient had psychotherapy weekly for 5 months. The therapist used free association, dream analysis, and facilitated transference to help the patient understand conflicts and foster change. Select the term that applies to this method. a. Rational-emotive behavior therapy b. Psychodynamic psychotherapy c. Cognitive-behavioral therapy d. Operant conditioning

ANS: B The techniques are aspects of psychodynamic psychotherapy. The distracters use other techniques.

During the first interview with a parent whose child died in a car accident, the nurse feels empathic and reaches out to take the patient's hand. Select the correct analysis of the nurse's behavior. a. It shows empathy and compassion. It will encourage the patient to continue to express feelings. b. The gesture is premature. The patient's cultural and individual interpretation of touch is unknown. c. The patient will perceive the gesture as intrusive and overstepping boundaries. d. The action is inappropriate. Psychiatric patients should not be touched.

ANS: B Touch has various cultural and individual interpretations. Nurses should refrain from using touch until an assessment can be made regarding the way in which the patient will perceive touch. The other options present prematurely drawn conclusions.

Which disorder is a culture-bound syndrome? a. Epilepsy b. Schizophrenia c. Running amok d. Major depression

ANS: C Culture-bound syndromes occur in specific sociocultural contexts and are easily recognized by people in those cultures. A syndrome recognized in parts of Southeast Asia is running amok, in which a person (usually a male) runs around engaging in furious, almost indiscriminate violent behavior.

Operant conditioning is part of the treatment plan to encourage speech in a child who is nearly mute. Which technique applies? a. Encourage the child to observe others talking. b. Include the child in small group activities. c. Give the child a small treat for speaking. d. Teach the child relaxation techniques.

ANS: C Operant conditioning involves giving positive reinforcement for a desired behavior. Treats are rewards and reinforce speech through positive reinforcement.

Which issues should a nurse address during the first interview with a patient with a psychiatric disorder? a. Trust, congruence, attitudes, and boundaries b. Goals, resistance, unconscious motivations, and diversion c. Relationship parameters, the contract, confidentiality, and termination d. Transference, countertransference, intimacy, and developing resources

ANS: C Relationship parameters, the contract, confidentiality, and termination are issues that should be considered during the orientation phase of the relationship. The remaining options are issues that are dealt with later.

Termination of a therapeutic nurse-patient relationship has been successful when the nurse: a. avoids upsetting the patient by shifting focus to other patients before the discharge. b. gives the patient a personal telephone number and permission to call after discharge. c. discusses with the patient changes that happened during the relationship and evaluates outcomes. d. offers to meet the patient for coffee and conversation three times a week after discharge.

ANS: C Summarizing and evaluating progress help validate the experience for the patient and the nurse and facilitate closure. Termination must be discussed; avoiding discussion by spending little time with the patient promotes feelings of abandonment. Successful termination requires that the relationship be brought to closure without the possibility of dependency-producing ongoing contact.

As a patient admitted to the eating disorders unit undresses, a nurse observes that the patient's body is covered by fine, downy hair. The patient weighs 70 pounds and is 5 feet 4 inches tall. Which term should be documented? a. Amenorrhea b. Alopecia c. Lanugo d. Stupor

ANS: C The fine, downy hair noted by the nurse is called lanugo. It is frequently seen in patients with anorexia nervosa. None of the other conditions can be supported by the data the nurse has gathered.

The patients below were evaluated in the emergency department. The psychiatric unit has one bed available. Which patient should be admitted? The patient: a. feeling anxiety and a sad mood after separation from a spouse of 10 years. b. who self-inflicted a superficial cut on the forearm after a family argument. c. experiencing dry mouth and tremor related to taking haloperidol (Haldol). d. who is a new parent and hears voices saying, "Smother your baby."

ANS: D Admission to the hospital would be justified by the risk of patient danger to self or others. The other patients have issues that can be handled with less restrictive alternatives than hospitalization.

Which outcome would be most appropriate for a symptom-management group for persons with schizophrenia? Group members will: a. state the names of their medications. b. resolve conflicts within their families. c. rate anxiety at least two points lower. d. describe ways to cope with their illness.

ANS: D An appropriate psychoeducational focus for patients with schizophrenia is managing their symptoms; coping with symptoms such as impaired memory or impaired reality testing can improve functioning and enhance their quality of life. Names of medications might be appropriate for a medication education group but would be a low priority for symptom management. Addressing intra-family issues would be more appropriate within a family therapy group or possibly a support group. Rating anxiety lower would be an expected outcome for a stress-management group.

Which aspect of direct care is an experienced, inpatient psychiatric nurse most likely to provide for a patient? a. Hygiene assistance b. Diversional activities c. Assistance with job hunting d. Building assertiveness skills

ANS: D Assertiveness training relies on the counseling and psychoeducational skills of the nurse. Assistance with personal hygiene would usually be accomplished by a psychiatric technician or nursing assistant. Diversional activities are usually the province of recreational therapists. The patient would probably be assisted in job hunting by a social worker or vocational therapist.

Which nursing intervention below is part of the scope of an advanced practice psychiatric/mental health nurse only? a. Coordination of care b. Health teaching c. Milieu therapy d. Psychotherapy

ANS: D Psychotherapy is part of the scope of practice of an advanced practice nurse. The distracters are within a staff nurse's scope of practice.

Which assessment finding for a patient diagnosed with an eating disorder meets criteria for hospitalization? a. Urine output 40 mL/hr b. Pulse rate 58 beats/min c. Serum potassium 3.4 mEq/L d. Systolic blood pressure 62 mm Hg

ANS: D Systolic blood pressure less than 70 mm Hg is an indicator for inpatient care. Many people without eating disorders have bradycardia (pulse less than 60 beats/min). Urine output should be more than 30 mL/hr. A potassium level of 3.4 mEq/L is within the normal range.

A citizen at a community health fair asks the nurse, "What is the most prevalent mental disorder in the United States?" Select the nurse's best response. a. Schizophrenia b. Bipolar disorder c. Dissociative fugue d. Alzheimer's disease

ANS: D The 12-month prevalence for Alzheimer's disease is 10% for persons older than 65 and 50% for persons older than 85. The prevalence of schizophrenia is 1.1% per year. The prevalence of bipolar disorder is 2.6%. Dissociative fugue is a rare disorder. See related audience response question.

Guidelines followed by the leader of a therapeutic group include focusing on recognizing dysfunctional behavior and thinking patterns, followed by identifying and practicing more adaptive alternate behaviors and thinking. Which theory is evident by this approach? a. Behavioral b. Interpersonal c. Psychodynamic d. Cognitive-behavioral

ANS: D The characteristics described are those of cognitive-behavioral therapy, in which patients learn to reframe dysfunctional thoughts and extinguish maladaptive behaviors. Behavioral therapy focuses solely on changing behavior rather than thoughts, feelings, and behaviors together. Interpersonal theory focuses on interactions and relationships. Psychodynamic groups focus on developing insight to resolve unconscious conflicts.

A psychiatric clinical nurse specialist uses cognitive-behavioral therapy for a patient diagnosed with anorexia nervosa. Which statement by the staff nurse supports this type of therapy? a. "What are your feelings about not eating foods that you prepare?" b. "You seem to feel much better about yourself when you eat something." c. "It must be difficult to talk about private matters to someone you just met." d. "Being thin doesn't seem to solve your problems. You are thin now but still unhappy."

ANS: D The correct response is the only strategy that attempts to question the patient's distorted thinking.

During an interview, a patient attempts to shift the focus from self to the nurse by asking personal questions. The nurse should respond by saying: a. "Why do you keep asking about me?" b. "Nurses direct the interviews with patients." c. "Do not ask questions about my personal life." d. "The time we spend together is to discuss your concerns."

ANS: D When a patient tries to focus on the nurse, the nurse should refocus the discussion back onto the patient. Telling the patient that interview time should be used to discuss patient concerns refocuses discussion in a neutral way. Telling patients not to ask about the nurse's personal life shows indignation. Saying that nurses prefer to direct the interview reflects superiority. "Why" questions are probing and non-therapeutic.

A college student said, "Most of the time I'm happy and feel good about myself. I have learned that what I get out of something is proportional to the effort I put into it." Which number on this mental health continuum should the nurse select? Mental Illness Mental Health 1 2 3 4 5 a. 1 b. 2 c. 3 d. 4 e. 5

ANS: E The student is happy and has an adequate self-concept. The student is reality-oriented, works effectively, and has control over own behavior. Mental health does not mean that a person is always happy.

Weight loss through dieting, fasting , or excessive exercise

Anorexia, Restricting Type

The intervention that can be practiced by an advanced practice registered nurse in psychiatric mental health but cannot be practiced by a basic level registered nurse is: a. Advocacy b. Psychotherapy c. Coordination of care d. Community-based care

B

A child known as the neighborhood bully says, "Nobody can tell me what to do." After receiving a poor grade on a science project, this child secretly loaded a virus on the teacher's computer. These behaviors support a diagnosis of: a. conduct disorder. b. oppositional defiant disorder. c. intermittent explosive disorder. d. attention deficit hyperactivity disorder.

B Oppositional defiant disorder is a repeated and persistent pattern of having an angry and irritable mood in conjunction with demonstrating defiant and vindictive behavior. Loading a virus is a vindictive behavior in retribution for a poor grade. Persons with conduct disorder are aggressive against people and animals; destroy property; are deceitful; violate rules; and have impaired social, academic, or occupational functioning. There is no evidence of explosiveness or distractibility.

A child reports to the school nurse of being verbally bullied by an aggressive classmate. What is the nurse's best first action? a. Give notice to the chief administrator at the school regarding the events. b. Encourage the victimized child to share feelings about the experience. c. Encourage the victimized child to ignore the bullying behavior. d. Discuss the events with the aggressive classmate.

B The behaviors by the bullying child create emotional pain and present the risk for physical pain. The nurse should first listen to the child's complaints and validate the child for reporting the events. Later, school authorities should be notified. School administrators are the most appropriate personnel to deal with the bullying child. The behavior should not be ignored; it will only get worse.

A client was admitted to the behavioral health unit for evaluation and diagnosis after being found wandering the streets. His personal hygiene is poor and his responses to questions are bizarre and inappropriate. The client's constitutional rights are violated when the nurse states: A) "We will help you make decisions that will keep you safe." B) "I am going to help you shower so you will not smell so bad." C) "Your pocket knife and nail clippers will be kept in the nurses' station." D) "You will be having a number of tests to help us learn about your condition."

B) "I am going to help you shower so you will not smell so bad."

A nurse works with an adolescent who was placed in a residential program after multiple episodes of violence at school. Establishing rapport with this adolescent is a priority because: (select all that apply) a. it is a vital component of implementing a behavior modification program. b. a therapeutic alliance is the first step in a nurse's therapeutic use of self. c. the adolescent has demonstrated resistance to other authority figures. d. acceptance and trust convey feelings of security for the adolescent. e. adolescents usually relate better to authority figures than peers.

B, D Trust is frequently an issue because the adolescent may never have had a trusting relationship with an adult. Trust promotes feelings of security and is the basis of the nurse's therapeutic use of self. Adolescents value peer relationships over those related to authority. Rewards for appropriate behavior are the main component of behavior modification programs.

HALITOSIS

Bad breath

Which of the drugs used by a polysubstance abuser is most likely to be responsible for withdrawal symptoms requiring both medical intervention and nursing support? A. Opiates B. Marijuana C. Barbiturates D. Hallucinogens

Barbiturates

acupressure, 728

Based on same principles as acupuncture but does not involve needles. Instead finger pressure or implements are used to stimulate the meridians.

the actual physical contact. An intentional touching of another's person, in a socially impermissible manner, without that person's consent.

Battery

Psychosis

Becoming out of touch with reality

Recurrent episodes of binge eating; short time, large amount. Feeling lack of control over eating behavior. Recurrent, inappropriate compensatory behavior to prevent weight gain Binging and compensating occur at least twice weekly over 3 months. Self-evaluation overly influenced by body shape and size

Bulimia Nervosa

A kindergartener is disruptive in class. This child is unable to sit for expected lengths of time, inattentive to the teacher, screams while the teacher is talking, and is aggressive toward other children. The nurse plans interventions designed to: a. promote integration of self-concept. b. provide inpatient treatment for the child. c. reduce loneliness and increase self-esteem. d. improve language and communication skills.

C Because of their disruptive behaviors, children with ADHD often receive negative feedback from parents, teachers, and peers, leading to self-esteem disturbance. These behaviors also cause peers to avoid the child with ADHD, leaving the child with ADHD vulnerable to loneliness. The child does not need inpatient treatment at this time. The incorrect options might or might not be relevant.

Shortly after the parents announced that they were divorcing, a 15-year-old became truant from school and assaulted a friend. The adolescent told the school nurse, "I'd rather stay in my room and listen to music. It's easier than thinking about what is happening in my family." Which nursing diagnosis is most applicable? a. Chronic low self-esteem related to role within the family b. Decisional conflict related to compliance with school requirements c. Ineffective coping related to adjustment to changes in family relationships d. Disturbed personal identity related to self-perceptions of changing family dynamics

C Depression is often associated with impulse control disorder. The correct nursing diagnosis refers to the patient's dysfunctional management of feelings associated with upcoming changes to the family. The teen displays self-imposed isolation. The distracters are not supported by data in the scenario.

An adolescent acts out in disruptive ways. When this adolescent threatens to throw a pool ball at another adolescent, which comment by the nurse would set appropriate limits? a. "Attention everyone: we are all going to the craft room." b. "You will be taken to seclusion if you throw that ball." c. "Do not throw the ball. Put it back on the pool table." d. "Please do not lose control of your emotions."

C Setting limits uses clear, sharp statements about prohibited behavior and guidance for performing a behavior that is expected. The incorrect options represent a threat, use of restructuring (which would be inappropriate in this instance), and a direct appeal to the child's developing self-control that may be ineffective.

A desired outcome for a 12-year-old diagnosed with attention deficit hyperactivity disorder (ADHD) is to improve relationships with other children. Which treatment modality should the nurse suggest for the plan of care? a. Reality therapy c. Social skills group b. Simple restitution d. Insight-oriented group therapy

C Social skills training teaches the child to recognize the impact of his or her behavior on others. It uses instruction, role-playing, and positive reinforcement to enhance social outcomes. The other therapies would have lesser or no impact on peer relationships.

What is the nurse's priority focused assessment for side effects in a child taking methylphenidate (Ritalin) for attention deficit hyperactivity disorder (ADHD)? a. Dystonia, akinesia, and extrapyramidal symptoms b. Bradycardia and hypotensive episodes c. Sleep disturbances and weight loss d. Neuroleptic malignant syndrome

C The most common side effects are gastrointestinal disturbances, reduced appetite, weight loss, urinary retention, dizziness, fatigue, and insomnia. Weight loss has the potential to interfere with the child's growth and development. The distracters relate to side effects of conventional antipsychotic medications.

Assessment data for a 7-year-old reveals an inability to take turns, blurting out answers to questions before a question is complete, and frequently interrupting others' conversations. How should the nurse document these behaviors? a. Disobedience c. Impulsivity b. Hyperactivity d. Anxiety

C These behaviors are most directly related to impulsivity. Hyperactive behaviors are more physical in nature, such as running, pushing, and the inability to sit. Inattention is demonstrated by failure to listen. Defiance is demonstrated by willfully doing what an authority figure has said not to do.

When a 5-year-old is disruptive, the nurse says, "You must take a time-out." The expectation is that the child will: a. go to a quiet room until called for the next activity. b. slowly count to 20 before returning to the group activity. c. sit on the edge of the activity until able to regain self-control. d. sit quietly on the lap of a staff member until able to apologize for the behavior.

C Time-out is designed so that staff can be consistent in their interventions. Time-out may require going to a designated room or sitting on the periphery of an activity until the child gains self-control and reviews the episode with a staff member. Time-out may not require going to a designated room and does not involve special attention such as holding. Counting to 10 or 20 is not sufficient.

A nursing assistant shares with the nurse that a client with schizophrenia is as difficult to communicate with as," someone with Alzheimer's." The nurse offers the following advice: A) "Try talking to him early in the day to get the best results. Fatigue disorganizes his thinking." B) "Schizophrenia and Alzheimer's disease both cause irreversible brain damage, so keep your conversations short when you talk to a client with either disorder." C) "His medication targets his disturbed thought and speech patterns. To maximize improvement he will need positive interactions and support." D) "Make sure he eats the comfort foods he is served because they increase serotonin production and will help normalize his thoughts and speech."

C) "His medication targets his disturbed thought and speech patterns. To maximize improvement he will need positive interactions and support."

A nurse prepares the plan of care for a 15-year-old diagnosed with moderate intellectual developmental disorder. What are the highest outcomes that are realistic for this patient? Within 5 years, the patient will: (select all that apply) a. graduate from high school. b. live independently in an apartment. c. independently perform own personal hygiene. d. obtain employment in a local sheltered workshop. e. correctly use public buses to travel in the community.

C, D, E Individuals with moderate intellectual developmental disorder progress academically to about the second grade. These people can learn to travel in familiar areas and perform unskilled or semiskilled work. With supervision, the person can function in the community, but independent living is not likely.

-CALLUS -UNPLEASANT ODOR -PLANTAR WARTS - FISSURES BETWEEN TOES - FUNGAL INFECTION such as ATHLETE'S FOOT, and INGROWN TOENAILS

COMMON FOOT PROBLEMS

-DANDRUFF -HAIR LOSS -TICKS -PEDICULOSIS(LICE) -HIRSUTISM

COMMON HAIR PROBLEMS

-HALITOSIS -GLOSSITIS -GINGIVITIS -PERIODONTAL DISEASE -REDDENED OR EXCORIATED MUSOCA -CHEILOSA -DENTAL CARIES -SORDES -STOMATITIS -PAROTITIS -PLAQUE -TARTAR -PYORRHEA

COMMON PROBLEMS OF THE MOUTH

-ABRASION -EXCESSIVE DRYNESS -AMMONIA DERMATITIS (DIAPER RASH) -ACNE -ERYTHEMA -HIRSUTISM

COMMON SKIN PROBLEMS

results in a period of severe disorganization resulting from the failure of an individual's usual coping mechanisms, the lack of usual resources, or both. Anxiety generally rises to severe or panic level. Person feels overwhelming helplessness and hopelessness. Dependency on others for guidance and assistance is a natural tendency. Without help, individual might become physically ill, become violent, or commit suicide to escape the pain.

CRISIS

The charge nurse has just given a presentation about the importance of practicing self-care. Which of the following staff behaviors will the nurse find concerning?

Calling the unit on days off to inquire about clients progress

6 differences between cultures

Communication, space, social organization, environmental conditions, biological differences, spiritual

What's the purpose of a social relationship?

Companionship, pleasure, sharing interesting and it evolves spontaneously

Industry vs. Inferiority

Competency

Attitudes

Complexes of beliefs and feelings that people have about specific ideas, situations, or other people

Powerlessness r/t...

Complicated grieving process Lifestyle Helplessness

Freud. Disharmony=

Conflict

CHEILOSIS

Cracking of lips

Maturational/developmental crises

Crises that occur in response to situations that trigger emotions related to unresolved conflicts in one's life

Focus on survival, safety, and security. Reestablish equilibrium and stabilization. Focus on strengths and adaptive coping. Offer suggestions for concrete, specific problem solving. Make provisions for follow-up care

Crisis Intervention Strategies

Psychiatric emergencies

Crisis situations in which general functioning has been severely impaired and the individual rendered incompetent or unable to assume personal responsibility

One's ability to cope with anxiety is overwhelmed. New coping behaviors must be developed to successfully resolve the source problem.

Crisis`

a state of maladaptive behavioral and psychological changes resulting from exposure to certain chemicals such as: mood swings, arguments, poor judgement. The body attempts to compensate by adapting to the substance. Physical and psychological tolerance develops.

Crucial, middle stage

The parent of a 6-year-old says, "My child is in constant motion and talks all the time. My child isn't interested in toys but is out of bed every morning before me." The child's behavior is most consistent with diagnostic criteria for: a. communication disorder. b. stereotypic movement disorder. c. intellectual development disorder. d. attention deficit hyperactivity disorder.

D Excessive motion, distractibility, and excessive talkativeness are seen in attention deficit hyperactivity disorder (ADHD). The behaviors presented in the scenario do not suggest intellectual development, stereotypic, or communication disorder.

Which child demonstrates behaviors indicative of a neurodevelopmental disorder? a. A 4-year-old who stuttered for 3 weeks after the birth of a sibling b. A 9-month-old who does not eat vegetables and likes to be rocked c. A 3-month-old who cries after feeding until burped and sucks a thumb d. A 3-year-old who is mute, passive toward adults, and twirls while walking

D Symptoms consistent with autistic spectrum disorders (ASD) are evident in the correct answer. Autistic spectrum disorder is one type of neurodevelopmental disorder. The behaviors of the other children are within normal ranges.

A child diagnosed with attention deficit hyperactivity disorder had this nursing diagnosis: impaired social interaction related to excessive neuronal activity as evidenced by aggression and demanding behavior with others. Which finding indicates the plan of care was effective? The child: a. has an improved ability to identify anxiety and use self-control strategies. b. has increased expressiveness in communication with others. c. shows increased responsiveness to authority figures. d. engages in cooperative play with other children.

D The goal should be directly related to the defining characteristics of the nursing diagnosis, in this case, improvement in the child's aggressiveness and play. The distracters are more relevant for a child with autism spectrum or anxiety disorder.

When a 5-year-old diagnosed with attention deficit hyperactivity disorder (ADHD) bounces out of a chair and runs over and slaps another child, what is the nurse's best action? a. Instruct the parents to take the aggressive child home. b. Direct the aggressive child to stop immediately. c. Call for emergency assistance from other staff. d. Take the aggressive child to another room.

D The nurse should manage the milieu with structure and limit setting. Removing the aggressive child to another room is an appropriate consequence for the aggressiveness. Directing the child to stop will not be effective. This is not an emergency. Intervention is needed rather than sending the child home.

An adolescent diagnosed with an impulse control disorder said, "I just want to die. I spend all my time getting even with people who have done wrong to me." When asked about a suicide plan, the adolescent replied, "I'll jump from the bridge near my home. My father threw kittens off that bridge, and they died because they couldn't swim." Rate the suicide risk. a. Absent c. Moderate b. Low d. High

D The suicide risk is high. The child is experiencing feelings of hopelessness and helplessness. The method described is lethal, and the means to carry out the plan are available.

37.7°C to 46°C ( 100°F to 115°F)

DESIRED TEMPERATURE FOR THERAPEUTIC BATH ORDERED FOR ADULTS

40.5°C (105°F)

DESIRED TEMPERATURE FOR THERAPEUTIC BATH ORDERED FOR INFANTS

Listing of all psychiatric disorders, including diagnostic criteria, associated features, prevalence, and differential diagnosis. Used to facilitate communication among mental health professionals -

DSM

Psychological strategies that help to lessen feelings of anxiety. Goal is to reduce uncomfortable negative emotions

Defense mechanisms

Patient is suspected of having hallucinations. Nurse would ask..

Do you hear or see things when others don't?

The student nurse asks why the nurse is documenting the clients nonverbal responses in addition to verbal responses during the initial assessment. Which of the following statements made by the nurse reflects the rationale for documenting both verbal and nonverbal responses?

Documenting both permits the reader to compare the behaviors for congruence.

Purpose of MSE (Mental status exam)

Examine thoughts, feelings and behaviors

Childhood depression: < 3 years old

Feeding problems, tantrums, lack of playfulness and emotional expressiveness

affect

Feeling, mood, or emotional tone; objective

Identity vs. Role Confusion

Fidelity

Which side effects of lithium can be expected at therapeutic levels? A. Fine hand tremor and polyuria B. Nausea and thirst C. Coarse hand tremor and gastrointestinal upset D. Ataxia and hypotension

Fine hand tremor and polyuria

The nurse observed that during a teaching session, the overall emotional tone of a client remained unchanged. The nurse documents this as:

Flat affect.

used primarily in obsessive compulsive disorders, by reducing symptoms

Fluvoxamine(luvox) SSRI'S

FIND

Frequency, Intensity, Number, and Duration

The nurse is asked to provide traits of a mentally healthy individual at a hospital in-service. The nurse knows that mentally healthy individuals are:

Independent and autonomous.

Seasonal features (seasonal affective disorder, [SAD]).

Indicates that episodes mostly begin in fall or winter and remit in spring. These patients have reduced cerebral metabolic activity. SAD is characterized by anergia, hypersomnia, overeating, weight gain, and a craving for carbohydrates; it responds to light therapy

PEDOCULOSIS

Infestation of lice

Feedback p. 192

It is the response to the message. It is the process by which performance is checked and malfunctions corrected; performs a regulatory function in the communication process.

Which of the following services are not typically offered by community mental health centers?

Legal assistance

Hypomania

Less than manic

Concrete thinking

Literal interpretations of the environment (ex. "what brought you to the psych hospital?" "a bus")

Observation of the behavior of the mental health team seems to indicate that one team member is primarily interested in client progress as a measure of their knowledge and expertise. Given the nurses knowledge of game theories, this team member might be functioning as which of the following?

Maximizer

The nurse on a psychiatric unit looks through a suicidal clients belongings, removing craft materials and mirrors. This is an example of:

Maximum suicide precautions.

Childhood depression: 9-12

Morbid thoughts and excessive worrying

For Opioid dependence:

Naloxone (Narcan) for reversal of overdose. Short lived effect. Methadone is used for maintenance treatment and relieves the drug hunger.

Early sign of Lithium Toxicity

Nausea, vomiting, diarrhea, thirst, polyuria, lethargy, slurred speech, muscle weakness, and fine hand tremor

Impact stage: Crisis intervention

Need for physical safety and emotional security Give simple directions; avoid accusations Provide phone number for crisis intervention

Omission (or commission) of an act that a reasonable and prudent person would (or would not) do

Negligence

It is a life-threatening neurologic emergency associated with the use of antipsychotic (neuroleptic) agents and characterized by a distinctive clinical syndrome of mental status change, rigidity, fever, and dysautonomia. a) Neuroleptic Malignant Syndrome b) Dengue Fever c) Brain Hemorrhage

Neuroleptic Malignant Syndrome

-THE CROWN -THE ROOT -THE PULP CAVITY

PARTS OF THE MOUTH

What's the key issue in management of this type of behavior?

PREVENTION

General Guidelines for Restraint Use

PRN orders not allowed - eminent risk Use for shortest time possible, reassess q 2 hrs for continued necessity. Pt observed constantly; document safety and comfort measures q 15 min. Pt must be debriefed after . Pt has the right to request notification of a family member. Death of any pt while in restraints is required to be reported to the FDA

-TO ENCOURAGE RELAXATION -TO RELIEVE MUSCLE TENSION -TO STIMULATE CIRCULATIONS TO TISSUES AND MUSCLES -TO ASSESS THE CLIENT'S SKIN

PURPOSES OF BACK RUB/MASSAGE

fetal alcohol syndrome (FAS) p. 302

Physical and mental defects found in babies of women who consumed alcohol during pregnancy

Sentence Completion Test p. 212

Projective personality test that asks clients to complete an extensive series of incomplete sentences with the first thoughts that come to mind

The nurse is caring for a patient experiencing mania. Which is the most appropriate nursing intervention? 1. Provide consistency among staff members when working with the patient. 2. Negotiate limits so the patient has a voice in the plan of care. 3. Allow only certain staff members to interact with the patient. 4. Attempt to control the patient's emotions.

Provide consistency among staff members when working with the patient.

Anticipatory guidance

Providing assistance in anticipation of the potential for crisis, thus averting it

Lasts weeks to months. Struggle to adapt Periods of acting "normal". Latter portion of phase—desire to talk about details of and feelings about trauma. Need for support and temporary dependence. Gradual awareness of full impact

RECOIL Of violence and trauma

Lasts months to years Gradual decrease in emotions of anxiety, fear, anger. Begins with reviewing and organizing what happened and why—blaming and justifying . Progresses to regaining sense of control and self-protection. Resolution of grief occurs. Unless resolution occurs, symptoms remaining qualify as PTSD

REORGANIZATON of violence and trauma

Racing thoughts

Rapid change of topics

Roles within a social relationship

Roles aren't delineated except for broad social norms (friend vs. lover)

Roles within the professional relationship

Roles for the client and the nurse are delineated.

Binges are triggered by

Sad mood. Stressful event. Hunger from dieting Feelings about body image, appearance, and weight

Ms. Aimes said to the nurse that she is not ready to talk about her recent divorce

Suppression

Do NO harm. Accept each client as a whole person. Develop a mutual trust. Explore behaviors and emotions. Encourage responsibility. Encourage effective adaptation. Provide consistency.

The Seven Principles of Mental Health Care

Subculture p. 186

The culture within the culture.

Understanding basic computer terminology

The following are the technology informatics guiding education reform (TIGER) competences, except:

ENAMEL

The hard substance that covers the tooth

COMPLETE BED BATH

The nurse washes the entire body of a dependent client in bed.

Transference p. 52

The process by which a client's unresolved feelings, attitudes, and wishes from the past are transferred into the present therapeutic relationship

flight of ideas, 367

The rapid and continuous succession of accelerated and overly productive responses to questions that seem related only by chance associations between one sentence fragment and another. Rhyming, clang associations, punning, and evidence of distractibility are often associated with flight of ideas.

Passive behavior p. 40

Timid holding back

Disadvantages of ECT

Treatment only provides temporary relief Memory Loss Adverse Physiologic effects

● Simple electronic audiotyping is often used during interviews whereby the interviews are entered into a word-processing program by clerical assistants in preparation for analysis. ● Using the internet for indirect and direct data collection in qualitative studies can also provide a vehicle for data analysis that yields a quantitative component as well as the qualitative analysis ● Computer are not only able to record the subjects responses to the questions but also routinely record the number of minutes the subject was online and the number of times they logged in.

Which of the following statements describe the data collection of a qualitative study? Select all that apply

Patient has several issues and nurse states "I want to be helpful to you" What kind of question would she ask that would be most therapeutic ?

Which one problem you like to work on?

Because of lacrimal fluid continually washes the eyes.

Why does eyes normally requires no special hygiene?

When patient is taking atypical antipsychotic drugs, what should be monitored? a) monitor glucose levels and infection b) monitor the electrical activity in the brain c) monitor the BP and HR

a) monitor glucose levels and infection

If the patient has a benzodiazepine toxicity, what drug should be given. a) Flumazenil b) Clozapine c) Reperdone

a)Flumazenil

An appropriate long-term goal/outcome for a recovering substance abuser would be that the client will A. discuss the addiction with significant others. B. state an intention to stop using illegal substances. C. abstain from the use of mood-altering substances. D. substitute a less-addicting drug for the present drug.

abstain from the use of mood-altering substances.

Problem solving Assertiveness Positive self-talk and self-acceptance Stress and anger management Communication and relationship skills Conflict resolution Time management Community living skills

adaptive coping techniques taught

A desired outcome for the maintenance phase of treatment for a manic client would be that the client will A. exhibit optimistic, energetic, playful behavior. B. adhere to follow-up medical appointments. C. take medication more than 50% of the time. D. use alcohol to moderate occasional mood "highs."

adhere to follow-up medical appointments.

Number one drug problem in North America

alcohol

: increased psychomotor activity as result as a consequence of alcohol.

alcohol withdrawel syndrome

For Stimulant Dependence

amantadine (Symmetrel), bromocriptine (Parlodel), carbamazine (Tegretol) and desipramine (Norpramin).

Benzodiazepines are useful for treating alcohol withdrawal because they A. block cortisol secretion. B. increase dopamine release. C. decrease serotonin availability. D. bind to ã-aminobutyric acid-benzodiazepine receptors.

bind to ã-aminobutyric acid-benzodiazepine receptors

Action oriented process, with the primary purpose of promoting adaptation & productivity in order to minimize pathology & promote the maintenance of health. Includes occupational therapy, music therapy, art therapy, psychodrama, dance/movement therapy & recreational therapy a) Somatic Therapy b) Individual Therapy c) Adjunct Therapy

c) Adjunct Therapy

These are the management of lithium toxicity except for this one: a) Lavage b) Forced Diuresis c) Administer activated charcoal d) PNSS

c) Administer activated charcoal

What type of agent that treats EPS? a) Anticonvulsant b) Antimatic agent c) Antiparkinson's agent

c) Antiparkinson's agent

Ability to experience intimacy Chum relationship - intense love relationship with a particular person of the same sex whom the child perceives to be similar to himself Child learns to put others need 1st Intimacy & acceptance provided by chum relationship reinforces positive self-esteem - leading to develop a healthy heterosexual relationship a) juvenile - schooler b) infancy c) preadolescence - schooler

c) preadolescence

results from interactions among the neurotransmitters, neuropeptides & hormones as they influence each others functions in the brain a) Tertiary Effects b) Primary Effects c) Secondary Effects

c) secondary effects

Learns to be economically, intellectually self sufficient a) juvenile b) preschool c) young adulthood

c) young adulthood

Anticonvulsant examples

carbamazapine (Tegretol) clonazepam (klonopin) valproic acid (Depakene, Depakote) lamotrigine (Lamictal) gabapentine (Neurontine, Gabarone)

Characteristics the nurse will assess in the client with antisocial personality disorder are A. deceitfulness, impulsiveness, and lack of empathy. B. perfectionism, preoccupation with detail, and verbosity. C. avoidance of interpersonal contact and preoccupation with being criticized. D. need for others to assume responsibility for decision-making and seeks nurture.

deceitfulness, impulsiveness, and lack of empathy.

Felt as if he was outside hos own body watching it happen

dissociation

Mental health is

the ability to cope and adjust

alter

An occurrence in dissociative identity disorder; an alter is a separate identity completely distinct from the host personality. Each alter performs different functions and stores different memories and feelings.

Weight loss is seen as improvement and a sign of extraordinary self control. Weight gain is seen as unacceptable failure in self control Onset after puberty - mid to late adolescents (14-18 years old). Mostly women Preoccupied with food. Concerned about eating in public

Anorexia

carbamazepine (Tegretol)

Anticonvulsant that works better in patients w/ rapid cycling and severely paranoid, angry, patients experiencing manias than euphoric, overactive, over-friendly patients experiencing manias.

valproate (depakote)

Anticonvulsant useful in treatment in lithium non-responders who are in acute mania, experience rapid cycles, are in dysphoric mania, or have not responded to cabamazepine; also helpful in prevention of future manic episodes

Hydroxyzine (vistaril)

Antihistamine/multipurpose agent. Used as mild tranquilizer and also pre or post- operative sedation to control vomiting, decrease anxiety and reduce amount of narcotics needed, also to relieve itching associated with allergic reactions Common adverse effects - mostly anticholinergic effects, blurred vision, constipation, mucosal dryness

Feelings of apprehension and uncertainty about the illness. Can produce sympathetic nervous response (fight-or-flight response)

Anxiety

Known as anxiolytics, tranquilizers & antianxiety Drug Classifications Benzodiazepines Azaspirones SSRI's - Selective Serotonin Reuptake Inhibitors Miscellaneous AntiAnxiety Agents

Anxiety Medications

A feeling of uneasiness, uncertainty, and helplessness .A normal emotional response to a real or imagined threat or stressor. A warning of impending danger.

Anxiety. Mild anxiety can increase learning by enhancing concentration and focus. Uncontrolled anxiety often leads to ineffective and maladaptive behavior. A normal part of survival and growth. Positively focused anxiety helps us to adapt, learn, and grow from our experiences. Responses to anxiety occur on four levels, ranging from mild to panic.

How to assess whether a person is hallucinating

Ask client what they are hearing, seeing and observe behavior that might indicate one is attending to internal stimuli Ask the client to describe the experience and note differences from your perception Ask what the voices are saying

Assertive behavior p. 40

Asking for what one wants or acting to get it in a way that respects other people

The deliberate threat coupled with the apparent ability to do physical harm to another. No actual contact is necessary. Verbally threatening a patient that you are going to force him or her to take medication against the patient's will is _______.

Assault

Upon arrival on the psychiatric unit this morning, which activity should be the nurses focus? The nurse should do which of the following?

Assess each client for whom the nurse will be providing care.

The nurse is caring for a newly admitted client who has not showered in several days and emits an offensive odor. Which of the following actions best conveys respect for the client?

Assess the clients abilities and needs related to performing self-care.

The nurse is caring for a client with depression. Which nursing intervention best demonstrates the nurses availability to the client?

Assist the client with the activities of daily living.

Jim is sometimes seen moving his lips silently or murmuring to himself when he does not realize others are watching. Sometimes when he is conversing with others, he suddenly stops, appears distracted for a moment, and then resumes. Based on these observations, Jim most likely is experiencing which symptom(s)? Select all that apply. 1. Illusions 2. Paranoia 3. Delusional thinking 4. Auditory hallucinations 5. Impaired reality testing 6. Stereotyped behaviors

Auditory hallucinations Impaired reality testing

AXIS 1-5 OF DSM

Axis I: Clinical disorders (e.g. schizophrenia, major depression, bipolar disorder) Axis II: Personality or developmental disorders (e.g. paranoid and borderline personality disorders, mental retardation) Axis III: General medical conditions that relate to axes I and II or have a bearing on treatment (e.g. endocrine disorders) Axis IV: Severity of psychosocial stressors (e.g. divorce, housing, educational issues) Axis V: Global assessment of functioning, on a scale of 0 to 100

Global Assessment of Functioning (GAF) p. 214

Axis V of the DSM-IV-TR reports the client' s overall level of functioning on psychologic, social, and occupational performance. The ratings on this scale fall within decile ranges and track both symptom severity and the functional level of an individual. Lowest level of functioning within the past 7 days.

Mechanism of action not fully known Advantage over others is that is has lower sedative properties and doesn't alter psychomotor functioning Requires 7-10 days before initial signs of improvement, 3-4 weeks of therapy

Azaspirone. Buspar

Role playing is associated with which type of therapy? A) Psychoanalysis B) Modeling C) Operant conditioning D) Systematic desensitization

B) Modeling

The nurse who provides therapeutic milieu management supports the clients best by: A) Allowing them to act out their fears and frustrations: B) Providing a safe place for them to practice coping skills C) Meeting their physical as well as emotional needs D) Encouraging them to talk about their problems with others

B) Providing a safe place for them to practice coping skills

Which attribute would be least helpful for a community mental health nurse to possess? A) Flexibility B) Reactive manner C) Nonjudgmental attitude D) Ability to cross service systems

B) Reactive manner

Which of the following is considered a primary behavioral theorist? A) Freud B) Skinner C) Sullivan D) Peplau

B) Skinner

Which of the following is classified as a circadian rhythm? A) Sex drive B) Sleep cycle C) Skeletal muscle contraction D) Maintenance of a focused stream of consciousness

B) Sleep cycle

A nurse suspects that a client has a personality disorder in addition to displaying symptoms of a mood disorder. To determine whether these observations are correct, the nurse could look in the client's medical record on the DSM-IV-TR A) axis I. B) axis II. C) axis III. D) axis IV.

B) axis II.

Regarding individuals with mental disorders, distress refers to a painful symptom, and disability refers to: A) the presence of deviant behavior. B) impairment in important areas of functioning. C) culturally appropriate responses to an event. D) a conflict between the individual and society.

B) impairment in important areas of functioning.

An ongoing, critically important responsibility of nurses working on an inpatient psychiatric unit is A) fostering research. B) milieu management. C) sympathetic listening. D) providing negative feedback.

B) milieu management.

The physician tells a client who demonstrates use of many rituals "We want to do an imaging study that will tell us which parts of your brain are particularly active. We believe the study will help us determine how to treat your symptoms." From this explanation, the nurse can determine that the physician will order a(n) A) computed tomography scan. B) positron emission tomography scan. C) ventriculogram. D) electroencephalogram.

B) positron emission tomography scan.

At the time of a home visit, the nurse notices that each parent and child in a family has his or her own personal online communication device. Each member of the family is in a different area of the home. Which nursing actions are appropriate? Select all that apply. a. Report the finding to the official child protection social services agency. b. Educate all members of the family about risks associated with cyberbullying. c. Talk with the parents about parental controls on the children's communication devices. d. Encourage the family to schedule daily time together without communication devices. e. Obtain the family's network password and examine online sites family members have visited.

B, C, D Education and awareness-based approaches have a chance of effectively reducing harmful online behavior, including risks associated with cyberbullying. Parental controls on the children's devices will support safe Internet use. Family time together will promote healthy bonding and a sense of security among members. There is no evidence of danger to the children, so a report to child protective agency is unnecessary. It would be inappropriate to seek the family's network password and an invasion of privacy to inspect sites family members have visited.

A nurse prepares to lead a discussion at a community health center regarding children's health problems. The nurse wants to use current terminology when discussing these issues. Which terms are appropriate for the nurse to use? Select all that apply. a. Autism b. Bullying c. Mental retardation d. Autism spectrum disorder e. Intellectual development disorder

B, D, E Some dated terminology contributes to the stigma of mental illness and misconceptions about mental illness. It's important for the nurse to use current terminology.

acupuncture, 728

Based on the belief that the vital life energy of the body (ch'i, or qi, pronounced "chee') circulates along 12 major and 8 secondary pathways, called medians. These pathways are linked to specific organ systems. Hair-thin needles placed at acupuncture sites are used to stimulate the meridians and influence the flow of energy that affects internal organs.

A nurse educator is teaching a group of students the definition of a mentally healthy individual. The nurse educator knows that an individual is considered mentally healthy when which of the following concepts give evidence to psychological, emotional, and social health?

Behavior

Deviance p. 3

Behavior outside the norm of a specific group; should not be construed to mean negative behavior

Senescence

Bereavement overload, symptoms of depression often confused with symptoms of dementia

Theories of Depression

Biological: genetic, biochemical factors, alterations in hormonal regulation Psychodynamic: exaggerated stress response Cognitive: Beck's cognitive triad: 1. a negative self deprecating view of self 2. a pessimistic view of the world 3. no validation for self will continue

The home health nurse is caring for a number of clients with chronic illnesses. Given World Health Organization (WHO) research, the nurse realizes that the client with which of the following is at greatest risk for mental disability?

Bipolar disorder

Alterations in behavior:

Bizarre behavior-eccentric dress or grooming, rituals, stilted rigid demeanor Extreme motor agitation Stereotyped behaviors Automatic obedience Waxy flexibility Stupor Negativism

Side effects of TCAs

Blurred vision, constipation, urinary retention, orthostatic HoTN, reduction of seizure threshold, tachycardia, arryhthmias, photosensitivity, weight gain

Used to treat seasonal affective disorder Exposure to intense artificial light Mediated by the eyes, not the skin Might help bulimia, insomnia, depression Very safe to use

Bright light therapy (former phototherapy)

Orientation stage, working stage and termination stage are all a part of

Building a therapeutic relationship

More common than anorexia.Binge eating and purging (self induced vomiting) twice a week Binging - eating in a limited period of time (usually 2 hours) an amount of food that is definitely larger than most people would eat under similar circumstances. Usually high calorie, high carbohydrate foods, Mostly female (90%).Late adolescents and early adulthood

Bulimia Nervosa

Purging. Laxatives and diuretics. Excessive exercise

Bulimia compensation

Norepinephrine Dopamine Reuptake Inhibitor (NDRI)

Bupropion (Wellbutrin) Blocks the reuptake of norepinephrine and dopamine; Stimulant action may reduce appetite; May increase sexual desire; Used as an aid to quit smoking Contraindicated in pt with seizure and eating disorders

How can the nurse gain self awareness?

By clarifying values, beliefs, and attitudes

Epidemiological studies contribute to improvements in care for individuals with mental disorders by: a. Providing information about effective nursing techniques. b. Identifying risk factors that contribute to the development of a disorder. c. Identifying who in the general population will develop a specific disorder. d. Identifying which individuals will respond favorably to a specific treatment.

C

Which statement about mental illness is true? a. Mental illness is a matter of individual nonconformity withsocietal norms. b. Mental illness is present when individual irrational and illogicalbehavior occurs. c. Mental illness changes with culture, time in history, politicalsystems, and the groups defining it. d. Mental illness is evaluated solely by considering individual controlover behavior and appraisal of reality.

C

Which statement best describes a major difference between a DSM-5 diagnosis and a nursing diagnosis? a. There is no functional difference between the two; both serve to identify a human deviance. b. The DSM-5 diagnosis disregards culture, whereas the nursing diagnosis takes culture into account. c. The DSM-5 is associated with present symptoms, whereas a nursing diagnosis considers past, present, and potential responses to actual mental health problems. d. The DSM-5 diagnosis impacts the choice of medical treatment, whereas the nursing diagnosis offers a framework for identifying multidisciplinary interventions

C

A nurse will prepare teaching materials for the parents of a child newly diagnosed with attention deficit hyperactivity disorder (ADHD). Which medication will the information focus on? a. Paroxetine (Paxil) c. Methyphenidate (Ritalin) b. Imipramine (Tofranil) d. Carbamazepine (Tegretol)

C CNS stimulants are the drugs of choice for treating children with ADHD: Ritalin and dexedrine are commonly used. None of the other drugs are psychostimulants used to treat ADHD.

Which of the following is associated with managed care organizations? A) NIC B) NOC C) MBHO D) IDTP

C) MBHO

Generativity vs. Stagnation

Care

Empathy is good

Caring and concern for patient but can not fully experience patients feelings.

Nonverbal communication p. 187

Carries more social meaning than spoken words. It is communication between two or more people without the use of words: body movements, including facial expressions and hand gestures; pitch, rate and volume of the voice; the use of personal and social space; touch; and the use of cultural artifacts ( such as clothing, jewelry and cosmetics)

● Evaluation ● Synthesis ● Analyses

Case studies, virtual labs, computer simulation models, and multiple choice questions. What kinds of questions are asked? Choose all that apply

waxy flexibility, p. 339

Catatonic posturing in which the client holds a bizarre posture for periods of time

AMMONIA DERMATITIS ( DIAPER RASH)

Caused by skin bacteria reacting with urea in the urine. The skin becomes reddened and is sore.

Prodromal syndrome

Certain behaviors predictive of impending violence - rigid posture, clenched fists/jaw, grim defiant affect, rapid speech, loud voice, arguing, demanding, profanity, threats, agitation, pacing, pounding and slamming

vegetative symptoms, p. 363

Changes in physiological functioning, such as appetite, sleep patterns, and elimination patterns; experienced during depression

Bipolar II disorder

Characterized by bouts of major depression with episodic occurrence of hypomania, never meet criteria for full mania

Major depressive disorder

Characterized by depressed mood, lack of interest or pleasure in activities, and social/occupational functioning is impaired for AT LEAST 2 WEEKS **No hx of mania and can't be attributed to substances or condition**

Bipolar disorders

Characterized by mood swings from profound depression to extreme euphoria/mania with intervening periods of normalcy, can have dellusions and hallucinations

When teaching students about suicide, the nursing instructor knows that basic suicide precautions include:

Checking the clients whereabouts and safety every 15 minutes.

Who established guidelines for diagnosis and treatment of childhood bipolar disorder?

Child and Adolescent Bipolar Foundation (CABF)

Clang associations

Choice of words is governed by sound (often rhyming)

Cyclothymia disorder

Chronic mood disturbance for AT LEAST 2 YEARS with numerous episodes of hypomania and depressed mood of insufficient severity to meet criteria for bipolar disorders

Which of the following community support programs should the nurse recommend to family members of a client with severe and persistent mental illness (SPMI) who is living independently in the community? Standard Text: Select all that apply.

4. Case management 5. Medication management

Nurse leans forward and does not interupt.

Active listening

Psychiatricmental health nursing interventions occur at which of the following levels of communication?

Interpersonal

Interactions that occur between two or more persons The verbal and nonverbal messages that are sent and received during every interaction

Interpersonal communication

Recovered memory

Memories that emerge into consciousness after being repressed for a period of time, sometimes years

ability to adjust and cope to everyday stresses

Mental health

rapid cycling

>4 mood episodes in a 12-month period. Used to indicate more severe symptoms, such as poorer global functioning, high risk recurrence risk, and resistances to conventional somatic treatments

Which assessment finding would cause the nurse to consider a child to be most at risk for the development of mental illness? a. The child has been raised by a parent with chronic major depression. b. The child's best friend was absent from the child's birthday party. c. The child was not promoted to the next grade one year. d. The child moved to three new homes over a 2-year period.

A Children raised by a depressed parent have an increased risk of developing an emotional disorder. Familial risk factors correlate with child psychiatric disorders, including severe marital discord, low socioeconomic status, large families and overcrowding, parental criminality, maternal psychiatric disorders, and foster-care placement. The chronicity of the parent's depression means it has been a consistent stressor. The other factors are not as risk- enhancing.

vicarious traumatization

A condition in which psychological aftereffects are experienced by those who assist victims of traumatic events. Also known as secondary trauma response

dysthymic disorder, p. 365

A condition of chronically depressed mood that occurs on a nearly daily basis for most of the day

Crises resulting from traumatic stress

A crisis that is precipitated by an unexpected, external stressor over which the individual has little or no control and from which he or she feels emotionally overwhelmed and defeated

Advanced-practice registered nurse (APRN) p. 24

A licensed registered nurse educationally prepared as a clinical nurse specialist or a nurse practitioner at the master's or higher level in the specialty of psychiatric-mental health nursing

personality disorder (PD)

A lifelong maladaptive pattern of perceiving, thinking, and relating that impair social or occupational functioning

Systemic Review

A literature review that follow a certain methodology to standardize the critique of research findings.

seasonal affective disorder (SAD), p. 366

A major depressive episode that follows a seasonal pattern; usually occurs in winter months

Blunted Affect, p. 347

A marked decrease in the variation or intensity of emotional expression

The client has frequently presented to the clinic with multiple physical complaints. The multiple physical complaints would warrant the nurse to screen the client for:

A mental disorder.

hypomania

A mild manic state in which the individual seems infectiously merry, extremely talkative, charming, and tireless.

Benton Visual Retention Test p. 212

A neuropsychological assessment instrument that measures aspects of a person's cognitive functioning, sometimes used as a quick screening device for cognitive dysfunction. The test taker is asked to reproduce various geometric designs after examining the designs for a few seconds.

Duty To Warn Others

A nurse who is aware of patient's intentions to cause harm to self or others must communicate this to other professionals and take steps to protect the potential recipient of harm. Not all comments or vague threats should be reported. Must be "specific threat" to a "readily identifiable" person or persons. When possible, discuss with clinical team. Balance patient's rights with those of 3rd party's. Documentation is critical. Nurse who fails to take prudent action can be held liable

Battering, p. 528

A pattern of repeated physical assau

mania, p. 366

A period of abnormal mood lasting at least 1 week characterized by persistently elevated, expansive, or irritable mood accompanied by at least three additional symptoms, such as inflated self-esteem or grandiosity, decreased need for sleep, pressured speech, flight of ideas, and others

major depressive disorder, p. 363

A period of at least 2 weeks during which there is either depressed mood or the loss of interest or pleasure in nearly all activities

histrionic personality disorder (HPD)

A personality disorder characterized by a pervasive and excessive emotionality and attention-seeking behavior; the person is overly dramatic and flamboyant

Dependent personality disorder (DPD)

A personality disorder characterized by a pervasive and excessive need to be taken care of that leads to submissive and clinging behavior and fears of separation

paranoid personality disorder

A personality disorder characterized by a pervasive distrust and suspiciousness of others and an expectation of being exploited, demeaned, or harmed by others

narcissistic personality disorder (NPD)

A personality disorder characterized by a pervasive pattern of grandiosity, need for admiration of others, a sense of entitlement, and lack of empathy for others

schizotypal personality disorder

A personality disorder characterized by a pervasive pattern of peculiarities of ideas, appearance, and behaviors, which results in impaired interpersonal relatedness

Violence

A physical act of force intended to cause harm to a person or an object (in addition to aggression verbal threats)

restorative care, 620

A planned, systematic program that focuses on restoration and maintenance of optimal function and assisting adults to compensate for impairments. Emphasizes prevention of deterioration.

Neologism p. 187

A private, unshared meaning of a word or term. Neologisms are frequently characteristic of the language of schizophrenic individuals.

Million Clinical Multiaxial Inventory-II (MCMI-II) p. 211

A profile of the presence and intensity of personality traits consistent with the DSM-IV-TR Axis II personality types. This test can provide valuable assistance in clarifying underlying stable personality features that can strongly influence the way clients interact with, and present symptoms to, health care providers

Mental disorder p. 5

A psychological group of symptoms, such as a pattern or a syndrome, in which the individual experiences distress (a painful symptom), or disability (impairment in one or more important areas of functioning), or a significantly increased risk of suffering, pain, loss of freedom, or death

Schizophreniform disorder

A psychotic disorder lasting less than 6 months in duration, with features that resemble schizophrenia

Beck Depression Inventory p. 211

A series of questions that ask the client to rate the presence and intensity of various symptoms of depression

suicide precautions

A set of protocols or guidelines for observing and monitoring client behavior

Which room placement would be best for a client experiencing a manic episode? A. A shared room with a client with dementia B. A single room near the unit activities area C. A single room near the nurse's station D. A shared room away from the unit entrance

A single room near the nurse's station

Underload p. 192

A situation that occurs when delay or lack of information interferes with one person's ability to comprehend the message of another

life review, 617

A structured process involving the recall of past events in one's life in an effort to find meaning in those events. Systematically reviews remote memories and addresses the expression of related feelings and the recognition of conflicts. Useful intervention for elders experiencing self-esteem disturbance, grief, hopelessness, powerlessness and altered role performance and social isolation.

The medication prescribed for a client acts by blocking reuptake of both serotonin and norepinephrine. The nurse evaluates the treatment as successful when observing: A) Laughing at a joke B) Exercising a sore shoulder C) Writes down his telephone number D) Going to his room to 'calm down'

A) Laughing at a joke

In addition to physicians, what other mental health discipline members have been identified as possessing the knowledge, skill, ability, and legal authority to intervene in the full range of mental health care? A) Nurses B) Social workers C) Clinical psychologists D) Chemical dependency counselors

A) Nurses

Most clients who are diagnosed with chronic mental illness are not likely to have their psychiatric mental health experiences covered by: A) Private insurance B) Medicare C) Medicaid D) Social Security

A) Private insurance

How are clients' rights affected after admission to a behavioral health unit? A) Rights as citizens remain intact. B) The right to legal counsel is lost. C) The right to refuse treatment is lost. D) The right to publicly complain is restricted.

A) Rights as citizens remain intact.

Treatment of mental illnesses with psychotropic drugs is directed at: A) altering brain neurochemistry. B) correcting brain anatomical defects. C) regulating social behaviors. D) activating the body's normal response to stress.

A) altering brain neurochemistry.

A patient repeatedly stated, "I'm stupid." Which statement by that patient would show progress resulting from cognitive behavioral therapy? a. "Sometimes I do stupid things." b. "Things always go wrong for me." c. "I always fail when I try new things." d. "I'm disappointed in my lack of ability."

ANS: A "I'm stupid" is a cognitive distortion. A more rational thought is "Sometimes I do stupid things." The latter thinking promotes emotional self-control. The distracters reflect irrational or distorted thinking. This item relates to an audience response question.

A community mental health nurse has worked with a patient for 3 years but is moving out of the city and terminates the relationship. When a novice nurse begins work with this patient, what is the starting point for the relationship? a. Begin at the orientation phase. b. Resume the working relationship. c. Initially establish a social relationship. d. Return to the emotional catharsis phase.

ANS: A After termination of a long-term relationship, the patient and new nurse usually have to begin at ground zero, the orientation phase, to build a new relationship. If termination is successfully completed, the orientation phase sometimes progresses quickly to the working phase. Other times, even after successful termination, the orientation phase may be prolonged.

Volition

Impairment in ability to initiate goal-directed activity

Disability p. 5

Impairment in one or more important areas of function in daily life.

A leader plans to start a new self-esteem building group. Which intervention would be most helpful for assuring mutual respect within the group? a. Describe the importance of mutual respect in the first session and make it a group norm. b. Exclude potential members whose behavior suggests they are likely to be disrespectful. c. Give members a brochure describing the purpose, norms, and expectations of the group. d. Explain that mutual respect is expected and confront those who are not respectful.

ANS: A It is helpful to motivate members to behave respectfully by describing how mutual respect benefits all members and is necessary for the group to be fully therapeutic. Setting a tone and expectation of mutual respect from the outset is the most helpful intervention listed. Excluding members because of how they might behave could exclude members who would have been appropriate, depriving them of the potential benefits of the group. Conveying expectations by brochure is less effective than doing so orally, because it lacks the connection to each member a skilled leader can create to motivate members and impart the expectation of respect. Confronting inappropriate behavior is therapeutic but only addresses existing behavior rather than preventing all such undesired behavior.

During a one-on-one interaction with the nurse, a patient frequently looks nervously at the door. Select the best comment by the nurse regarding this nonverbal communication. a. "I notice you keep looking toward the door." b. "This is our time together. No one is going to interrupt us." c. "It looks as if you are eager to end our discussion for today." d. "If you are uncomfortable in this room, we can move someplace else."

ANS: A Making observations and encouraging the patient to describe perceptions are useful therapeutic communication techniques for this situation. The other responses are assumptions made by the nurse.

A nurse leads a psychoeducational group for patients in the community diagnosed with schizophrenia. A realistic outcome for group members is that they will: a. discuss ways to manage their illness. b. develop a high level of trust and cohesiveness. c. understand unconscious motivation for behavior. d. demonstrate insight about development of their illness.

ANS: A Patients with schizophrenia almost universally have problems associated with everyday living in the community, so discussing ways to manage the illness would be an important aspect of psychoeducation. Discussing concerns about daily life would be a goal to which each could relate. Developing trust and cohesion is desirable but is not the priority outcome of a psychoeducational group. Understanding unconscious motivation would not be addressed. Insight would be difficult for a patient with residual schizophrenia because of the tendency toward concrete thinking.

A group begins the working phase. One member has a childhood history of neglect and ridicule by parents. Which comment would the group leader expect from this member? a. "My boss is always expecting more of me than the others, but talking to him would only make it worse." b. "I'm sorry for talking all the time, but there is so much going on in my life. I can't remember what I already said." c. "Thanks for the suggestions everyone. Maybe some of them will help. It won't hurt to give them a try." d. "This group is stupid. Nobody here can help anybody else because we are all so confused. It's a waste."

ANS: A People who frequently complain, yet reject help or suggestions when offered, tend to have histories of severe deprivation as children, often accompanied by neglect or abuse. The other comments reflect dynamics other than the help-rejecting complainer, such as the monopolizer who apologizes for talking too much, the person who is insightful and agrees to try a peer's suggestion, and the demoralizing member.

One bed is available on the inpatient eating disorders unit. Which patient should be admitted to this bed? The patient whose weight decreased from: a. 150 to 100 pounds over a 4-month period. Vital signs are temperature, 35.9° C; pulse, 38 beats/min; blood pressure 60/40 mm Hg b. 120 to 90 pounds over a 3-month period. Vital signs are temperature, 36° C; pulse, 50 beats/min; blood pressure 70/50 mm Hg c. 110 to 70 pounds over a 4-month period. Vital signs are temperature 36.5° C; pulse, 60 beats/min; blood pressure 80/66 mm Hg d. 90 to 78 pounds over a 5-month period. Vital signs are temperature, 36.7° C; pulse, 62 beats/min; blood pressure 74/48 mm Hg

ANS: A Physical criteria for hospitalization include weight loss of more than 30% of body weight within 6 months, temperature below 36° C (hypothermia), heart rate less than 40 beats/min, and systolic blood pressure less than 70 mm Hg.

Which technique will best communicate to a patient that the nurse is interested in listening? a. Restating a feeling or thought the patient has expressed. b. Asking a direct question, such as "Did you feel angry?" c. Making a judgment about the patient's problem. d. Saying, "I understand what you're saying."

ANS: A Restating allows the patient to validate the nurse's understanding of what has been communicated. Restating is an active listening technique. Judgments should be suspended in a nurse-patient relationship. Close-ended questions such as "Did you feel angry?" ask for specific information rather than showing understanding. When the nurse simply states that he or she understands the patient's words, the patient has no way of measuring the understanding.

Three members of a therapy group share covert glances as other members of the group describe problems. When one makes a statement that subtly criticizes another speaker, the others nod in agreement. Which group dynamic should the leader suspect? a. Some members are acting as a subgroup instead of as members of the main group. b. Some of the members have become bored and are disregarding others. c. Three members are showing their frustration with slower members. d. The leadership of the group has been ineffective.

ANS: A Subgroups, small groups isolated within a larger group and functioning separately from it, sometimes form within therapy groups. When this occurs, subgroup members are cohesive with other subgroup members but not with the members of the larger group. Members of the subgroup may be bored or frustrated or expressing passive aggression, but the primary dynamic is the splitting off from the main group.

Select the example of tertiary prevention. a. Helping a person diagnosed with a serious mental illness learn to manage money b. Restraining an agitated patient who has become aggressive and assaultive c. Teaching school-age children about the dangers of drugs and alcohol d. Genetic counseling with a young couple expecting their first child

ANS: A Tertiary prevention involves services that address residual impairments, with a goal of improved independent functioning. Restraint is a secondary prevention. Genetic counseling and teaching school-age children about substance abuse and dependence are examples of primary prevention.

A patient is suspicious and frequently manipulates others. To which psychosexual stage do these traits relate? a. Oral b. Anal c. Phallic d. Genital

ANS: A The behaviors in the stem develop as the result of attitudes formed during the oral stage, when an infant first learns to relate to the environment. Anal-stage traits include stinginess, stubbornness, orderliness, or their opposites. Phallic-stage traits include flirtatiousness, pride, vanity, difficulty with authority figures, and difficulties with sexual identity. Genital-stage traits include the ability to form satisfying sexual and emotional relationships with members of the opposite sex, emancipation from parents, a strong sense of personal identity, or the opposites of these traits.

Which patient is the best candidate for brief psychodynamic therapy? a. An accountant with a loving family and successful career who was involved in a short extramarital affair b. An adult with a long history of major depression who was charged with driving under the influence (DUI) c. A woman with a history of borderline personality disorder who recently cut both wrists d. An adult male recently diagnosed with anorexia nervosa

ANS: A The best candidates for psychodynamic therapy are relatively healthy and well-functioning individuals, sometimes referred to as the "worried well," who have a clearly circumscribed area of difficulty and are intelligent, psychologically minded, and well-motivated for change. Patients with psychosis, severe depression, borderline personality disorders, and severe character disorders are not appropriate candidates for this type of treatment.

Which finding best indicates that the goal "Demonstrate mentally healthy behavior" was achieved? A patient: a. sees self as capable of achieving ideals and meeting demands. b. behaves without considering the consequences of personal actions. c. aggressively meets own needs without considering the rights of others. d. seeks help from others when assuming responsibility for major areas of own life.

ANS: A The correct response describes an adaptive, healthy behavior. The distracters describe maladaptive behaviors.

mantra, 727

People may medicate using a mantra which is a syllable, word or name that is repeatedly changed aloud.

Ms. Day unconsiously wishes for her mom to be dead but tells nurse how wonderful her mom is

Reaction formation

ERYTHEMA

Redness associated with a variety of conditions, such as rashes, exposure to sun, elavated body temperature

Personal boundary difficulties

-clients often lack a sense of where their bodies end in a relationship to where others begin. They also might say they are merging with inanimate objects.

A client with dependent personality disorder who had been living with her newly married son was admitted a week ago for treatment of depression, which began after her son suggested that she move out. Which remark by the client would the nurse evaluate as showing *improvement* in the client's condition? A. "My son's suggestion hurt me greatly." B. "My son is less at fault than my daughter-in-law." C. "I'm going to need help to afford to rent an apartment." D. "How will I ever live alone with no one to look after my affairs?"

"I'm going to need help to afford to rent an apartment."

A new psychiatric technician mentions to the nurse, "Depression seems to be a disease of old people. All the depressed clients on the unit are older than 60 years." The reply by the nurse that clarifies prevalence is A. "That is a good observation. Depression does mostly strike people older than 50 years." B. "Depression is seen in people of all ages, from childhood to old age." C. "Depression is most often seen among the middle adult age group." D. "The age of onset for most depressive episodes is given as 18 years."

"Depression is seen in people of all ages, from childhood to old age."

A client with paranoid schizophrenia refuses food. He states the voices are telling him the food is contaminated and will change him from a male to a female. A therapeutic response for the nurse would be A. "You are safe here in the hospital, nothing bad will happen to you." B. "The voices are wrong about the hospital food. It is not contaminated." C. "I understand that the voices are very real to you, but I do not hear them." D. "Other people are eating the food and nothing is happening to them."

"I understand that the voices are very real to you, but I do not hear them."

A client with obsessive-compulsive personality disorder takes the nurse aside and mentions "I've observed you interacting with Mr. D. You are not approaching him properly. You should be more forceful with him." The *best* response for the nurse would be A. "I will be continuing to follow the care plan for Mr. D." B. "I see you are trying to control Mr. D's therapy as well as your own." C. "Your eye for perfection extends even to my nursing interventions." D. "Mr. D's care is really of no concern to you or to other clients."

"I will be continuing to follow the care plan for Mr. D."

A nurse caring for a nearly mute depressed client wishes to show acceptance of the client. An intervention that would meet this objective would be to say A. "I will be spending time with you each day to try to improve your mood." B. "I would like to sit with you for 15 minutes now and again this afternoon." C. "Each day we will spend time together to talk about things that are bothering you." D. "It is important for you to share your thoughts with someone who can help you evaluate whether your thinking is realistic."

"I would like to sit with you for 15 minutes now and again this afternoon."

General adaptation syndrome

(GAS) The structural and chemical changes that stress produces in the body; first identified by Hans Selye

violence

(a term used interchangeably with aggression) refers to behavior that is harmful to others. can be expressed verbally (e.g. screaming, threatening, abusive remarks) and nonverbally (e.g. physical assault). Can also occur on a continuum ranging from threatened assault to physical abuse to acts of terrorism.

internal and less evident to observers

(subjective).

Nursing Guidelines for Avoidant PD

1. A friendly, accepting, reassuring approach is the best way to treat patients. 2. Being pushed into social situations can cause extreme and severe anxiety.

Nursing Guidelines for Schizoid PD

1. Avoid being too "nice" or "friendly." 2. Do not try to increase socialization. 3. Perform thorough diagnostic assessment as needed to identify symptoms or disorders the patient is reluctant to discuss.

Cardiac neurosis

A condition in which the individual is overly concerned about the heart and its functioning; may occur following a cardiac illness and may lead individual to restrict behavior

maturational crisis

A crisis that involves life cycle changes or normal transitions of human development

pain disorder

A somatoform disorder with pain as the predominant feature. Psychological factors greatly influence the onset, severity, and exacerbation of the pain

Intellectualization

A defense or coping mechanism in which intellectual processes are overused to avoid closeness or affective experience or expression

Reaction formation

A defense or coping mechanism in which unacceptable feelings are disguised by repression of the real feeling and reinforcement of the opposite feeling

Fantasy

A defense or coping mechanism that is a sequence of mental images, like a daydream; may be conscious or unconscious

Dissociation

A defense or coping mechanism that protects the self from a threatening awareness of uncomfortable feelings by denying their existence in awareness

spliting

A defensive mechanism that prevents one from uniting the good and bad aspects of oneself or of one's image of another person. The person views the self as all good or all bad, failing to integrate the positive and negative qualities of the self and others into a cohesive image.

Nervous Breakdown p. 15

A general, nonspecific term for an incapacitating but otherwise unspecified type of mental disorder

Stage III Delirious Mania

A grave form of the disorder, characterized by severe clouding of consciousness and representing an intensification of symptoms associated with acute mania

hypervigilance

A heightened state of watchfulness in which the person in always on guard and unable to relax; occurs most often in those who are mistrustful and overly suspicious

Individual values and practices

A highly personal matter is determined by:

Milieu therapy

A humanistic approach to institutional treatment based on the belief that institutions can help patients recover by creating a climate that promotes self-respect, responsible behavior, and meaningful activity

postpartum mood episode, p. 368

A mood disorder with postpartum onset

Positive reinforcement

A response to the specific behavior that is pleasurable or produces the desired results

Negative reinforcement

A response to the specific behavior that prevents an undesirable result from occurring, removal of a stimulus after a particular response to increase the likelihood that the response will reoccur

Self-awareness p. 32

A sense of knowing what one is experiencing, a major goal of therapy

Psychiatric-mental health nursing p. 18

A specialized area of nursing practice committed to promoting mental health through the assessment, diagnosis, and treatment of human responses to mental health problems and psychiatric disorders

cohesion, p. 627

A spirit of common purpose; togetherness

Depression

A state wherein the person experiences profound sadness.

Crisis

A sudden event in one's life that disturbs homeostasis, during which usual coping mechanisms cannot resolve the problem

Johari Window, p. 625

A theoretical tool used to represent self-awareness and self-disclosure in relation to other people.

CALLUS

A thickened portion of epidermis, a mass of keratotic materials Caused by pressure from shoes

Codependency p. 297

A type of behavior that encourages another person to maintain a maladaptive dependency. Codependent people are often at least partially unaware of this outcome of their behavior.

A cognitive therapist would help a client restructure the thought "I am stupid!" to: A) "What I did was stupid." B) "I am not as smart as others." C) "Things usually go wrong for me." D) "Things like this should not happen to anyone."

A) "What I did was stupid."

What is the most common cause of adolescent suicide?

Abandonment by parent(s) or close peer

Detached concern p. 32

Ability to distance oneself in order to help others. It is essential to avoid burnout, to use appropriate assertiveness and to maintain empathic abilities in stressful situations.

Patient states "I do cocaine", Nurse responds showing...

Acceptance

Childhood depression: 3-5

Accident proneness, phobias, excessive self-reproach

Fight-or-flight response

Aggression (fight)-withdrawal (flight) response to stress

The nurse plans to implement health promotion activities at the local senior citizen center. To meet the goal of promoting knowledge related to maximizing mental health and functional ability, the nurses teaching is guided by World Health Organization research and should include discussion of which priority area specific to the leading causes of mental disability?

Alcohol

Data Mining

Along with the proliferation of clinical diagnostics measurement system, there has been a rapid expansion of unique applications. One of these is the powerful tool in the knowledge discovery process that can now be done with a number of commercial and software packages

Advanced directive

Also called a living will, this is a document in which a patient formally states preferences for health care in the event that he or she later becomes mentally incapacitated, and names a person who has durable power of attorney to serve as a substitute decision maker to implement the patient's stated preferences.

PERINEAL-GENITAL CARE

Also referred to as perineal care or pericare.

The psychiatricmental health nurse reflecting on professional role activities is referred to the standards of professional performance by a colleague. To which organization should the nurse look for guidance?

American Nurses Association

EITHER CLOSED OR OPEN

An unoccupied bed can be __________

What is a stressor?

Anything that causes stress, can be environmental, familial, traumatic, biological, social, chemical, or psychological

PLANTAR WARTS

Appear on the sole of the foot These are warts that are caused by the virus, papovavirus hominis

The nurse is planning care for a client who has been withdrawn and isolated for the last three days. Which action will best demonstrate the nurses empathy for this client?

Approach the client regularly and spend time with the client.

Patient is smiling and humming. Affect?

Appropriate

FEET

Are essential for ambulation and merit attention even when people are confined to bed.

A nurse case manager is assigned to a client with recurring substance abuse issues. Which of the following strategies would be most important in providing care to this client?

Assessment and problem identification

Grandiosity

Believes ability to be better than everyone elses

The nurse knows that basic suicide precautions may be started without a physicians order and maximum suicide precautions (or restrictions):

Can be instituted without a physicians order only under emergency conditions.

A WARM OR HOT BATH

Can dilates superficial arterioles, bringing more blood and nourishment to the skin. Vigorous rubbing has the same effect.

EXCESSIVE BATHING

Can interfere with the intended lubricating effect of the sebum, causing dryness of the skin.

What are the general strategies for relapse prevention?

Cognitive and behavioral: -recognize and learn how to avoid or cope with threats to recovery -changing lifestyle -learn how to participate in society w/out drugs - secure help from others/social support

Raven's Progressive Matrices Test p. 212

Cognitive function test that provides data on intellectual ability in a relatively culturally unbiased manner. It asks the client to solve two-dimensional visual-spatial items of increasing difficulty.

Cognitive psychological management of A/V

Commendations, information, education, thought stopping, contracts

Alprazolam (Xanax) Chlordiazepozide (Librium) Clorazepate(Tranzene) Diazepam (Valium) Lorazepam (Ativan) Oxazepam (Serax)

Common Benzodiazepines

Spiritual distress r/t...

Complicated grieving process AEB anger towards God, questioning meaning of own existence, inability to participate in usual religious practices

When a client is admitted to a behavioral health unit, which of the following does he or she NOT have the right to do? A) Refuse treatment B) Send and receive mail C) Seek legal counsel D) Accessibility to all possessions

D) Accessibility to all possessions

In order to be most affective, the community mental health nurse involved in assertive community treatment (ACT) needs to possess: A) knowledge of both national and local political activism. B) the ability to cross service systems. C) an awareness of own cultural and personal values. D) creative problem solving and intervention skills.

D) creative problem solving and intervention skills.

The prevalence rate over a 12-month period for major depressive disorder is: A) lower than the prevalence rate for panic disorders. B) greater than the prevalence rate for psychotic disorders. C) equal to the prevalence rate for psychotic disorders. D) greater than the prevalence rate for generalized anxiety.

D) greater than the prevalence rate for generalized anxiety.

The nurse providing anticipatory guidance to the mother of a toddler should advise that childhood temper tantrums are best handled by: A) giving the child what he is asking for. B) scolding the child when he displays tantrum behaviors. C) spanking the child at the onset of the tantrum behaviors. D) ignoring the tantrum and giving attention when the child acts appropriately.

D) ignoring the tantrum and giving attention when the child acts appropriately.

When assessing and planning treatment for a client who has recently arrived in the United States from China, the nurse should be alert to the possibility that the client's explanatory model for his illness reflects A) supernatural causes. B) negative forces. C) inheritance. D) imbalance.

D) imbalance.

The community mental health movement was least influenced by: A) the advent of antipsychotic medications. B) increasing public awareness of the poor care given in some large psychiatric hospitals. C) the proliferation of federal entitlement programs, making it possible to move the mentally ill out of hospitals. D) the increasingly larger numbers of advanced practice nurses prepared to care for the mentally ill in the community.

D) the increasingly larger numbers of advanced practice nurses prepared to care for the mentally ill in the community.

Which nursing diagnosis would be least useful for a depressed client who shows psychomotor retardation? A. Constipation B. Death anxiety C. Activity intolerance D. Self-care deficit: bathing/hygiene

Death anxiety

Behavioral patterns that protect the individual against a real or perceived threat. These are primarily unconscious behaviors though some are within voluntary control.. These offer protection to the self from a stressful situation Block conscious awareness of threatening feelings

Defense mechanisms

Unconscious behaviors

Defense mechanisms

Anergia

Deficiency of energy

Circumstantiality

Delay in reaching the point of a communication because of unnecessary and tedious details

body invents sensory input (visual and tactile hallucinations), has extreme motor agitation, tremors, high anxiety, tonic-clonic seizures possible.

Delirium Tremens

An alcoholic states "I can quit drinking at any time"

Denial

26 BONES, 107 LIGAMENTS, AND 19 MUSCLES

Each foot contains how many bones, ligaments and muscles?

Crises reflecting psychopathology

Emotional crises in which preexisting psychopathology significantly impairs or complicates adaptive resolution

The nurse engaged in a therapeutic relationship with a client uses nonverbal communication to:

Enhance verbal messages.

The nurse case manager is participating in a treatment team meeting where members are discussing treatment options for a client with a history of medication and treatment nonadherence. The clients health insurance plan does not cover the cost of expensive atypical antipsychotic depot medications. Which of the following is not an appropriate action by the nurse in this situation?

Enrolling the client in another health insurance plan

A child or adolescent who experiences a parents or friends suicide are at greater risk for suicide and should be:

Evaluated for depression.

hypersomnia

Excessive sleeping

Object loss

Experiences loss of significant other during first 6 months of life **predisposes individual to depression**

What took is used for making diagnosis of bipolar in children/teens?

FIND

Delusions

False personal beliefs

Excel

For a mechanism that translates and transfer source data to prepare it for analysis, what generic programs serve multiple needs?

Avolition

General lack of drive or motivation to pursue meaningful goals

Axis III

General medical disorders

The nurse knows that family members of suicide victims rarely seek assistance from mental health care professionals. One way to help survivors may be to

Get families involved with the American Association of Suicidology.

CLEANSING BATH

Given chiefly for hygienic purposes

Mistakes most Nurses make in the care of patient?

Giving advice

Axis V

Global assessment of functioning rated on the Global Assessment of Functioning (GAF) scale which measures individual's psychological, social, and occupational functioning

A client approaches the nurse grimacing, talking in a whisper, and waving his arms. Which of the following actions best demonstrates the nurses ability to develop a therapeutic relationship?

Greet the client by name to demonstrate caring.

Responsibility (suicide)

Grief process when a client succeeds in suicide, feel you could have done more, "it was on my watch"

The nurse is teaching staff at a community mental health clinic about what constitutes a mental disorder. Which comment by staff indicates to the nurse the need for further teaching?

Grieving after a loss may signal a mental disorder.

Word salad

Group of words put together in a random fashion that don't make sense

PERIODONTAL DISEASE

Gums appear spongy and bleeding

suicidal ideation

Having thoughts of harming or killing oneself

Irritable mood

Hostility, rage, often over trivial matters

Shock, denial, disbelief, confusion, fear, hysteria, horror, anger, shame, helplessness and vulnerability, physiologic responses, disturbed sleeping and eating. Delayed reactions possible. Occasionally, dissociative symptoms, intrusive memories, severe anxiety

IMPACT of violence and trauma

Magical thinking

Idea that if one things something, it must be true (ex. I can fly!)

When I get out of here I want to be a nurse just like you

Identification

Empathy

Identification with and understanding of another's situation, feelings, and motives

Overload p. 192

In communication theory, sensory input that exceeds a person's tolerance level or capacity

Statistical analysis services Statistical package for social sciences

In general, the statistical analysis steps of the quantitative process rely heavily on functions specific to a variety of statistical software applications. What are two of the most popular programs use today?

Comparing the effects of vegetarian and omnivorous diets on weight management

In her research, a nurse researcher used informatics and chose qualitative research. Which of the following approaches to qualitative research is most likely to be successful?

Data Coding

In qualitative research, the researcher must decide on which text may be of interest and use a word processing program to search for words, phrases or other markers within a text file using any number of word processing software packages. This term refers to:

Coding

In quantitative studies, the data for the variables of interest are collected in a numerical form. These numerical values are entered into designated fields in the process of:

Phase III: Schizophrenia

In the active phase of the disorder there are psychotic symptoms (delusions, hallucinations) and impairment in work, social relations, and self-care

anhedonia

Inability or decreased ability to experience pleasure, joy, intimacy, and closeness.

Mutism

Inability or refusal to speak

Anhedonia, p. 338

Inability to experience pleasure

anhedonia, p, 363

Inability to experience pleasure

Anhedonia

Inability to feel pleasure

Tangentiality

Inability to get to the point of communication d/t introduction of many new topics

Avolition, p. 337

Inability to pursue and persist in goal-directed activities

dissociative amnesia

Inability to recall significant personal information which is usually traumatic

A 22-year-old suicidal client acknowledges being molested by her stepfather since she was eight-years-old. The nurse knows this clients self-destructive behavior is best explained by:

Interpersonal theory.

Take place within oneself and commonly are referred to as our "self-talk" or "self-dialogu

Intrapersonal

Analyzing others like his therapist would

Introjection

critical incident stress management (CISM)

Is a comprehensive, integrative and multifaceted approach to crisis intervention that spans the time sequence of a crisis. Based on notion that no single intervention alone is effective in crisis work.

CORN

Is a keratosis caused by friction and pressure from a shoe ​

AS-NEEDED (prn) CARE

Is provided as required by the client.

goblet issues, p. 631

Issues of minor importance to the group that help members get to know one another better and to test out each other

Is there a time line on a relationship?

It can vary in length but is time-limited with an expected termination or ending

Learner - interface

It deals with how students interact with the technology medium used in the course

Physiologic data

It exist to help researchers unfamiliar with data-mining algorithms to use data mining for analysis, prediction and reporting purposes.

Clinical research informatics

It involves the use of informatics in the discovery and management of new knowledge relating to health and disease. Includes the management of information related to clinical trials and also involves informatics related to secondary research use of clinical data.

Schizophrenia

It is a devastating brain disease characterized by profound withdrawal from interpersonal relationships; cognitive and perceptual disturbances. Loss of contact with reality. It is also described as a psychotic disorder.

HOT SITZ BATH

It is done to reduce inflammation and clean the perineal and perianal area.

The use of body language

Kinesics

Care for the patient with an eating disorder

Medications:Don't really work well Sometimes antidepressants. Olanapine (Zyprexa) Used to treat bizarre body image distortion Cognitive Therapy..Used successfully Change distorted thinking and coping mechanisms Hospitalized when life-threatening fluid and electrolyte imbalance, organ failure, complete inability to eat

Altruistic suicide

Occurs where ties to the group or community are considered more important than individual identity

MORNING CARE

Often provided after client have breakfast, although it may be provided BEFORE breakfast which usually includes providing elimination of needs, a bath, or a shower, perineal care, back massages and oral, nail and hair care, making a client's bed is part of morning care.

Which statement about antidepressant medications, in general, can serve as a basis for client and family teaching? A. Onset of action is from 1 to 6 weeks. B. They tend to be more effective for men. C. They may cause recent memory impairment. D. They often cause the client to have diurnal variation.

Onset of action is from 1 to 6 weeks.

Defense Mechanisms

Operations outside of a person's awareness that protect against anxiety; also called coping mechanisms or mental mechanisms

Nurses learn about patients needs, build trust and begin assessment

Orientation phase 1

In an effort to prevent suicide, an important advocacy strategy for all nurses to implement is to:

Reduce barriers to health care.

-PROMOTION OF PATIENT'S COMFORT -TO PROVIDE A CLEAN NEAT ENVIRONMENT FOR THE CLIENT -TO PREVENT THE SPREAD OF INFECTION

PURPOSES OF BED MAKING

What symptoms herald the onset of full psychosis in schizophrenia?

Perceptual abnormalities, ideas of reference, and suspiciousness

Moderate anxiety

Perceptual field diminishes, hard time paying attention and focusing, ego defense mechanisms

Severe anxiety

Perceptual field is so diminished that concentration centers on one detail only or on many extraneous details, psycho-pyhsiologic-neurotic responses

Indications of mood-stabilizing agents

Prevent and treatment of manic episodes associated with bipolar disorder

EYELIDS AND LASHES

Prevent the entrance of foreign particles

EARLY MORNING CARE

Provided to clients as they awaken in the morning which consists of providing a urinal or bedpan to the client, confined in bed, washing the face and hands and giving of oral care.

HOUR SLEEP (HS) or PM CARE

Provided to clients before they retire for the night usually includes providing elimination of needs, washing the face and hands, giving oral care and massage.

How people use their space

Proxemics

The nurse is serving on a committee charged with reviewing the roles and responsibilities of the nurses on the psychiatric unit. Which publication should the nurse bring to the first meeting?

PsychiatricMental Health Nursing Standards of Practice

Axis IV

Psychosocial and environmental problems

Complicated grieving r/t...

Real or perceived loss Bereavement overload

(Freud) Ego

Reality

as sedation wears off, psychomotor activity increases.

Rebound phenomenon:

The psychiatric home health nurse has made repeated attempts to make a home visit to a homebound client, only to find that the client is not at home at the scheduled time. What is the best action by the nurse?

Reevaluate the clients homebound status.

Reorganization stage: Long-term counseling, if needed

Refer for long-term counseling for axis I disorder

Patient lost their job and feels depressed and worthless because of it. What kind of stressor is this?

Situational stressor

Therapeutic nurse-client relationship p. 46

Relationship in which the nurse uses theoretical understanding, personal attributes, and appropriate clinical techniques to provide the opportunity for a corrective emotional experience for clients. Also called one-to-one relationship

Interpersonal p. 4

Relationships between oneself and others.

Hardiness p. 4

Remain healthy even under high levels of stress or in the face of loss or trauma.

BATHING

Removes accumulated oil, perspiration, dead skin cells, and some bacteria.

Repeating exact words of patient to remind them of what they said or for them to know you are listening

Restating

- broad term used to characterize any form of limiting a person's movement or access to his or her own body. Examples include: physical holds, bed rails, lap trays, restraint devices, and medications.

Restraint

Voluntary patients can accept or refuse treatment. Involuntary patients do NOT lose their right to give informed consent Court must decide if person is not competent and therefore treatment can be imposed (such as medications. In case of psychiatric emergency, meds can be given without consent to prevent harm to self or others

Right to Give or Refuse Consent to Treatment..even the mentally ill

Patient rights

Right to refuse medications, right to least-restrictive treatment, right to self-determination, right to confidentiality, right to informed consent

Associative loosness

Shift of ideas from one unrelated topic to another (like having too much coffee in the morning :P)

An overwhelming emotion that paralyzes the individual's ability to process information No decision making by the individual is possible Individual is unable to sort through information received

Shock

Which of the following client behaviors is the best illustration of a positive response to the therapeutic environment in an inpatient ward?

Signing up during the community meeting for ward duties

Allows patient time to think and say more. Maintain eye contact. Convey interest and concern with facial expressions

Silence

somatization disorder

Somatoform disorder characterized by multiple physical complaints with no evidence of physiologic impairment

Care of patient after rape

Stay with patient. Provide empathy, support, safety Provide medical care, information. Move slowly, give rationales for care. Protect rights. Shame might interfere with reaching out for support.Referrals: Crisis intervention, support group, counseling

Right/wrong. Conscious mind.

Superego

Incorporation

Taking on a client's symptoms, their problems become yours

Techniques to deal with aggression

Talk down, physical outlets, meds, assistance from staff, restraints, observation and documentation

The nurse is admitting a client who is from Kenya to the psychiatric unit. Which of the following actions will demonstrate cultural competence?

Talk with the client to determine fluency in English.

A nurse is admitting a client who is from Japan. What is the first step the nurse should take?

Talk with the client to determine the clients level of fluency.

TAPOTEMENT

Tapping, hacking movements pf the hand on the back.

Which side effect of antipsychotic medication has no known treatment? A. Anticholinergic effects B. Pseudoparkinsonism C. Dystonic reaction D. Tardive dyskinesia

Tardive dyskinesia

evaluation of patient, summary of pt., referrals, and discussing the termination

Termination stage 3

What message does violence convey?

That the perpetrator's point of view is correct, not the victims

Symptoms of transient depression

The "blues", some crying, some difficulty getting mind off disappointment, feeling tired and listless

● Georgetown Home Health Care Classification ● North American Nursing Diagnosis Association ● Patient Data Set ● Nursing Interventions Classification ● Nursing Outcomes Classification

The American Nurses Association (ANA) has supported the need to standardize nursing care terms for computer-based patient care systems. The ANA has accepted 7 systems terminology for the description of nursing practice. Select all that apply.

Grief

The emotional response to loss and its accompanying changes.

Empathy p. 43

The ability to feel what others feel and respond to and understand the experience of others on their terms

stalking, p. 532

The act of following, viewing, communicating with, or moving threateningly toward another person

impulsiveness

The actions of those who act without considering the consequences of their behavior

Mourning

The actions or expressions of the bereaved, including the symbols, clothing, and ceremonies that make up the outward manifestations of grief.

palliative care, 610

The active total care of clients whose disease is not responsive to curative treatment. Not all palliative care occurs at the end of life, and much of it aims to help clients and their families reach personal goals, reconcile conflicts and derive meaning at the end of life.

here-and-now activation, p. 638

The first task of the group therapist—steering the group into the here-and-now

INGROWN TOENAIL

The growing inward of nail into the soft tissues around it, most often results from improper nail trimming

HIRSUTISM

The growth of excessive body hair is called ______

Alogia

The inability to speak because of mental defect, confusions, or aphasia

personality

The individual qualities, including habitual behavior patterns, that make a person unique

Grieving

The internal process the person uses to work through the response to loss.

Transcranial magnetic stimulation

The introduction of short pulses of magnetic energy to stimulate nerve cells in the brain used to treat depression

THE DENTIN

The ivory-colored internal part of the crown below the enamel.

Computer technology learning

The most efficient for integrating EBP is to have CDSS clinical decision support system, a computer system designed to impact clinical decision making about individual patients at the moment those decisions are mader. Most CDSS is made up of 3 essential components, except.

Panic

The most intense state of anxiety, psychotic response

physical abuse, p. 528

The nonaccidental use of physical force that results in bodily injury, pain, or impairment

The client asks the nurse if certain changes can be made in the unit milieu. Which action by the nurse indicates understanding of the nursing role in the therapeutic milieu?

The nurse discusses the desired changes with the client.

During a group session, a client expresses anger at the nurse. The nurse sits tensely with arms and legs crossed while verbally agreeing that the clients point of view is correct. Which of the following messages is being sent by the nurse?

The nurse is sending a mixed message

Antipsychotics

The standard antipsychotics are used to treat the positive

bereavement, p. 369

The state of loss; a natural process rather than a mental illness

Learner-content

The student studies the course content and participates in course activities.

Epidemiology p. 6

The study of the distribution and determinants of mental disorders in human populations. It is used to do the following: -determine causative factors for specific disorders -Id groups at high risk of developing certain disorders -recognize changes in health problems especially emerging new problems plan for current health needs and predict future needs -evaluate preventive and therapeutic measures

Axis system is used to?

Understanding the patients medical condition

A woman has been living in a shelter with her children after escaping her abusive husband. Her move-out date is getting closer. She states, Im afraid to leave here. Im afraid for my safety and the safety of my children. Which response by the nurse most accurately conveys empathy?

This is a difficult and scary transition. Lets work on developing a plan to keep you and your family safe.

The abnormal involuntary movement scale AIMS

This is done approx every few weeks with clients on antipsychotic therapy. ` The following is tested: Facial and oral movements Extremity movements Trunk movement Global judgments Dental status

Distractibility

Unable to focus on school lessons, work, etc.

means that with continued use, more of the substance is needed to produce the same effect

Tolerance

Benzodiazepines Antidepressants Antihistamines Propranolol Anxiolytics

Treatments for anxiety

poverty of speech, p. 337

Using a single word or a very limited number of words to express experiences or thoughts; thought to be symptomatic of diminished thoughts

Wernicke's - Korsakoff syndrome treatment

Vit B IM Increase fluids Increase calories in diet

Bland or flat affect

Weak emotional tone

Narcissistic PD

arrogance w/ grandiose view of self-importance. Has the need for constant admiration, along w/ a lack of empathy for others, which strains relationships. results in exploitation of others. Underneath this personality is a person w/ intense shame and fear of abandonment.

Physical interventions that produce behavioral changes a) Group Therapy b) Individual Therapy c) Somatic Therapy

c) Somatic Therapy

affection need, p. 630

consists of being able to love other people or to be close and intimate to a satisfactory degree, and having other s loveyou or be close and intimate with you to a satisfactory degree.

A withdrawn client is assessed as having distorted thinking that is not reality based. A nursing diagnosis that should be considered for her would be A. impaired verbal communication. B. disturbed thought processes. C. disturbed self-esteem. D. defensive coping.

disturbed thought processes.

Reiki, 729

gentle laying on of hands corresponding t the seven main chakras. Popular form of spiritual healing, energy flows through the healer's hands.

A client with disorganized schizophrenia would have greatest difficulty with the nurse A. interacting with a neutral attitude. B. using concrete language. C. giving multistep directions. D. providing nutritional supplements

giving multistep directions.

Characteristic behaviors the nurse will assess in the narcissistic client are A. dramatic expression of emotion, being easily led. B. perfectionism and preoccupation with detail. C. grandiose, exploitive, and rage-filled behavior. D. angry, highly suspicious, aloof, withdrawn behavior

grandiose, exploitive, and rage-filled behavior.

A bipolar client tells the nurse "I have the finest tenor voice in the world. The three tenors who do all those TV concerts are going to retire because they can't compete with me." The nurse would make the assessment that the client is displaying A. flight of ideas. B. distractibility. C. limit testing. D. grandiosity.

grandiosity.

Pseudo euphoria

not really THAT happy, just hyper?

The most helpful message to transmit about relapse to the recovering alcoholic client is that lapses A. are an indicator of treatment failure. B. are caused by physiological changes. C. result from lack of good situational support. D. can be learning situations to prolong sobriety.

result from lack of good situational support.

psychological autopsy

review of the circumstances and events that preceded an individual's completed suicide

Belittling jokes, sexual advances, pornography, sexual assault

sexual harrassment

undifferentiated somatoform disorder

somatoform disorder A condition characterized by unexplained physical complaints of at least 6 months' duration

withdrawal symptoms and tolerance are signs that

the person has physical dependence on the drug.

One of the most important tools for communicating with mentally ill clients is

therapeutic listening.

A patient with acute depression states, "God is punishing me for my past sins." What is the nurse's most therapeutic response? a. "You sound very upset about this." b. "God always forgives us for our sins." c. "Why do you think you are being punished?" d. "If you feel this way, you should talk to your minister."

ANS: A The nurse reflects the patient's comment, a therapeutic technique to encourage sharing for perceptions and feelings. The incorrect responses reflect probing, closed-ended comments, and giving advice, all of which are non-therapeutic.

A patient diagnosed with schizophrenia has been stable for 2 months. Today the patient's spouse calls the nurse to report the patient has not taken prescribed medication and is having disorganized thinking. The patient forgot to refill the prescription. The nurse arranges a refill. Select the best outcome to add to the plan of care. a. The patient's spouse will mark dates for prescription refills on the family calendar. b. The nurse will obtain prescription refills every 90 days and deliver to the patient. c. The patient will call the nurse weekly to discuss medication-related issues. d. The patient will report to the clinic for medication follow-up every week.

ANS: A The nurse should use the patient's support system to meet patient needs whenever possible. Delivery of medication by the nurse should be unnecessary for the nurse to do if patient or a significant other can be responsible. The patient may not need more intensive follow-up as long as medication is taken as prescribed.

At what point in the nurse-patient relationship should a nurse plan to first address termination? a. During the orientation phase b. At the end of the working phase c. Near the beginning of the termination phase d. When the patient initially brings up the topic

ANS: A The patient has a right to know the conditions of the nurse-patient relationship. If the relationship is to be time-limited, the patient should be informed of the number of sessions. If it is open-ended, the termination date will not be known at the outset, and the patient should know that the issue will be negotiated at a later date. The nurse is responsible for bringing up the topic of termination early in the relationship, usually during the orientation phase.

During group therapy, one patient says to another, "When I first started in this group, you were unable to make a decision, but now you can. You've made a lot of progress. I am beginning to think that maybe I can conquer my fears too." Which therapeutic factor is evident by this statement? a. Hope b. Altruism c. Catharsis d. Cohesiveness

ANS: A The patient's profession that he may be able to learn to cope more effectively reflects hope. Groups can instill hope in individuals who are demoralized or pessimistic. Altruism refers to doing good for others, which can result in positive feelings about oneself. Catharsis refers to venting of strong emotions. Cohesion refers to coming together and developing a connection with other group members.

Which principle has the highest priority when addressing a behavioral crisis in an inpatient setting? a. Resolve the crisis with the least restrictive intervention possible. b. Swift intervention is justified to maintain the integrity of a therapeutic milieu. c. Rights of an individual patient are superseded by the rights of the majority of patients. d. Patients should have opportunities to regain control without intervention if the safety of others is not compromised.

ANS: A The rule of using the least restrictive treatment or intervention possible to achieve the desired outcome is the patient's legal right. Planned interventions are nearly always preferable. Intervention may be necessary when the patient threatens harm to self.

Consider this comment from a therapist: "The patient is homosexual but has kept this preference secret. Severe anxiety and depression occur when the patient anticipates family reactions to this sexual orientation." Which perspective is evident in the speaker? a. Theory of interpersonal relationships b. Classical conditioning theory c. Psychosexual theory d. Behaviorism theory

ANS: A The theory of interpersonal relationships recognizes the anxiety and depression as resulting from unmet interpersonal security needs. Behaviorism and classical conditioning theories do not apply. A psychosexual formulation would focus on uncovering unconscious material that relates to the patient problem.

Which scenario best depicts a behavioral crisis? A patient is: a. waving fists, cursing, and shouting threats at a nurse. b. curled up in a corner of the bathroom, wrapped in a towel. c. crying hysterically after receiving a phone call from a family member. d. performing push-ups in the middle of the hall, forcing others to walk around.

ANS: A This behavior constitutes a behavioral crisis because the patient is threatening harm to another individual. Intervention is called for to defuse the situation. The other options speak of behaviors that may require intervention of a less urgent nature because the patients in question are not threatening harm to self or others.

A patient diagnosed with anorexia nervosa virtually stopped eating 5 months ago and lost 25% of body weight. A nurse asks, "Describe what you think about your present weight and how you look." Which response by the patient is most consistent with the diagnosis? a. "I am fat and ugly." b. "What I think about myself is my business." c. "I'm grossly underweight, but that's what I want." d. "I'm a few pounds overweight, but I can live with it."

ANS: A Untreated patients with anorexia nervosa do not recognize their thinness. They perceive themselves to be overweight and unattractive. The patient with anorexia will usually tell people perceptions of self. The patient with anorexia does not recognize his or her thinness and will persist in trying to lose more weight.

An outpatient diagnosed with anorexia nervosa has begun refeeding. Between the first and second appointments, the patient gained 8 pounds. The nurse should: a. assess lung sounds and extremities. b. suggest use of an aerobic exercise program. c. positively reinforce the patient for the weight gain. d. establish a higher goal for weight gain the next week.

ANS: A Weight gain of more than 2 to 5 pounds weekly may overwhelm the heart's capacity to pump, leading to cardiac failure. The nurse must assess for signs of pulmonary edema and congestive heart failure. The incorrect options are undesirable because they increase the risk for cardiac complications.

A nurse says, "I am the only one who truly understands this patient. Other staff members are too critical." The nurse's statement indicates: a. boundary blurring. b. sexual harassment. c. positive regard. d. advocacy.

ANS: A When the role of the nurse and the role of the patient shift, boundary blurring may arise. In this situation the nurse is becoming over-involved with the patient as a probable result of unrecognized countertransference. When boundary issues occur, the need for supervision exists. The situation does not describe sexual harassment. Data are not present to suggest positive regard or advocacy.

Select all that apply. An experienced nurse says to a new graduate, "When you've practiced as long as I have, you instantly know how to take care of psychotic patients." What information should the new graduate consider when analyzing this comment? a. The experienced nurse may have lost sight of patients' individuality, which may compromise the integrity of practice. b. New research findings should be integrated continuously into a nurse's practice to provide the most effective care. c. Experience provides mental health nurses with the essential tools and skills needed for effective professional practice. d. Experienced psychiatric nurses have learned the best ways to care for mentally ill patients through trial and error. e. An intuitive sense of patients' needs guides effective psychiatric nurses.

ANS: A, B Evidence-based practice involves using research findings and standards of care to provide the most effective nursing care. Evidence is continuously emerging, so nurses cannot rely solely on experience. The effective nurse also maintains respect for each patient as an individual. Overgeneralization compromises that perspective. Intuition and trial and error are unsystematic approaches to care.

Select all that apply. A nurse ends a relationship with a patient. Which actions by the nurse should be included in the termination phase? a. Focus dialogues with the patient on problems that may occur in the future. b. Help the patient express feelings about the relationship with the nurse. c. Help the patient prioritize and modify socially unacceptable behaviors. d. Reinforce expectations regarding the parameters of the relationship. e. Help the patient to identify strengths, limitations, and problems.

ANS: A, B The correct actions are part of the termination phase. The other actions would be used in the working and orientation phases.

Select all that apply. Which benefits are most associated with use of telehealth technologies? a. Cost savings for patients b. Maximize care management c. Access to services for patients in rural areas d. Prompt reimbursement by third party payers e. Rapid development of trusting relationships with patients

ANS: A, B, C Telehealth has shown it can maximize health and improve disease management skills and confidence with the disease process. Many rural parents have felt disconnected from services; telehealth technologies can solve those problems. Although telehealth's improved health outcomes regularly show cost savings for payers, one significant barrier is the current lack of reimbursement for remote patient monitoring by third party payers. Telehealth technologies have not shown rapid development of trusting relationships.

Select all that apply. Which statements by patients diagnosed with a serious mental illness best demonstrate that the case manager has established an effective long-term relationship? "My case manager: a. talks in language I can understand." b. helps me keep track of my medication." c. gives me little gifts from time to time." d. looks at me as a whole person with many needs." e. lets me do whatever I choose without interfering."

ANS: A, B, D Each correct answer is an example of appropriate nursing foci: communicating at a level understandable to the patient, providing medication supervision, and using holistic principles to guide care. The distracters violate relationship boundaries or suggest a laissez faire attitude on the part of the nurse.

Select all that apply. A psychiatric nurse discusses rules of the therapeutic milieu and patients' rights with a newly admitted patient. Which rights should be included? The right to: a. have visitors b. confidentiality c. a private room d. complain about inadequate care e. select the nurse assigned to their care

ANS: A, B, D Patients' rights should be discussed shortly after admission. Patients have rights related to receiving/refusing visitors, privacy, filing complaints about inadequate care, and accepting/refusing treatments (including medications). Patients do not have a right to a private room or selecting which nurse will provide care.

Select all that apply. A person in the community asks, "People with mental illnesses went to state hospitals in earlier times. Why has that changed?" Select the nurse's accurate responses. a. "Science has made significant improvements in drugs for mental illness, so now many persons may live in their communities." b. "There's now a better selection of less restrictive treatment options available in communities to care for people with mental illness." c. "National rates of mental illness have declined significantly. There actually is not a need for state institutions anymore." d. "Most psychiatric institutions were closed because of serious violations of patients' rights and unsafe conditions." e. "Federal legislation and payment for treatment of mental illness has shifted the focus to community rather than institutional settings."

ANS: A, B, E The community is a less restrictive alternative than hospitals for treatment of persons with mental illness. Funding for treatment of mental illness remains largely inadequate but now focuses on community rather than institutional care. Antipsychotic medications improve more symptoms of mental illness; hence, management of psychiatric disorders has improved. Rates of mental illness have increased, not decreased. Hospitals were closed because funding shifted to the community. Conditions in institutions have improved.

Select all that apply. A nurse is interacting with patients in a psychiatric unit. Which statements reflect use of therapeutic communication? a. "Tell me more about that situation." b. "Let's talk about something else." c. "I notice you are pacing a lot." d. "I'll stay with you a while." e. "Why did you do that?"

ANS: A, C, D The correct responses demonstrate use of the therapeutic techniques making an observation and showing empathy. The incorrect responses demonstrate changing the subject and probing, which are non-therapeutic techniques.

Select all that apply. A patient diagnosed with schizophrenia lives in the community. On a home visit, the community psychiatric nurse case manager learns that the patient: • wants to attend an activity group at the mental health outreach center. • is worried about being able to pay for the therapy. • does not know how to get from home to the outreach center. • has an appointment to have blood work at the same time an activity group meets. • wants to attend services at a church that is a half-mile from the patient's home. Which tasks are part of the role of a community mental health nurse? a. Rearranging conflicting care appointments b. Negotiating the cost of therapy for the patient c. Arranging transportation to the outreach center d. Accompanying the patient to church services weekly e. Monitoring to ensure the patient's basic needs are met

ANS: A, C, E The correct answers reflect the coordinating role of the community psychiatric nurse case manager. Negotiating the cost of therapy and accompanying the patient to church services are interventions the nurse would not be expected to undertake. The patient can walk to the church services; the nurse can provide encouragement.

Select all that apply. Which comments by an adult best indicate self-actualization? a. "I am content with a good book." b. "I often wonder if I chose the right career." c. "Sometimes I think about how my parents would have handled problems." d. "It's important for our country to provide basic health care services for everyone." e. "When I was lost at sea for 2 days, I gained an understanding of what is important."

ANS: A, D, E Self-actualized persons enjoy privacy, have a sense of democracy, and show positive outcomes associated with peak experiences. Self-doubt, defensiveness, and blaming are not consistent with self-actualization.

Select all that apply. A novice psychiatric nurse has a parent with bipolar disorder. This nurse angrily recalls feelings of embarrassment about the parent's behavior in the community. Select the best ways for this nurse to cope with these feelings. a. Seek ways to use the understanding gained from childhood to help patients cope with their own illnesses. b. Recognize that these feelings are unhealthy. The nurse should try to suppress them when working with patients. c. Recognize that psychiatric nursing is not an appropriate career choice. Explore other nursing specialties. d. The nurse should begin new patient relationships by saying, "My own parent had mental illness, so I accept it without stigma." e. Recognize that the feelings may add sensitivity to the nurse's practice, but supervision is important.

ANS: A, E The nurse needs support to explore these feelings. An experienced psychiatric nurse is a resource that may be helpful. The knowledge and experience gained from the nurse's relationship with a mentally ill parent may contribute sensitivity to compassionate practice. Self-disclosure and suppression are not adaptive coping strategies. The nurse should not give up on this area of practice without first seeking ways to cope with the memories.

Which characteristic would be more applicable to a community mental health nurse than to a nurse working in an operating room? a. Kindness b. Autonomy c. Compassion d. Professionalism

ANS: B A community mental health nurse often works autonomously. Kindness, compassion, and professionalism apply to both nurses.

A nurse introduces the matter of a contract during the first session with a new patient because contracts: a. specify what the nurse will do for the patient. b. spell out the participation and responsibilities of each party. c. indicate the feeling tone established between the participants. d. are binding and prevent either party from prematurely ending the relationship.

ANS: B A contract emphasizes that the nurse works with the patient rather than doing something for the patient. "Working with" is a process that suggests each party is expected to participate and share responsibility for outcomes. Contracts do not, however, stipulate roles or feeling tone, and premature termination is forbidden.

Which patient statement would lead the nurse to suspect unsuccessful completion of the developmental task of infancy? a. "I have very warm and close friendships." b. "I'm afraid to allow anyone to really get to know me." c. "I'm always absolutely right, so don't bother saying more." d. "I'm ashamed that I didn't do things correctly in the first place."

ANS: B According to Erikson, the developmental task of infancy is the development of trust. The correct response is the only statement clearly showing lack of ability to trust others. Warm, close relationships suggest the developmental task of infancy was successfully completed; rigidity and self-absorption are reflected in the belief one is always right; and shame for past actions suggests failure to resolve the crisis of initiative versus guilt.

During a support group, a patient diagnosed with schizophrenia says, "Sometimes I feel sad that I will never have a good job like my brother. Then I dwell on it and maybe I should not." Select the nurse leader's best comment to facilitate discussion of this issue. a. "It is often better to focus on our successes rather than our failures." b. "How have others in the group handled painful feelings like these?" c. "Grieving for what is lost is a normal part of having a mental disorder." d. "I wonder if you might also experience feelings of anger and helplessness."

ANS: B Asking others to share their experiences will facilitate discussion of an issue. Giving information may serve to close discussion of the issue because it sounds final. Suggesting a focus on the positives implies a discussion of the issue is not appropriate. Suggesting other possible feelings is inappropriate at this point, considering the patient has identified feelings of sadness and seems to have a desire to explore this feeling. Focusing on other feelings will derail discussion of the patient's grief for his perceived lost potential.

A patient says, "All my life I've been surrounded by stupidity. Everything I buy breaks because the entire American workforce is incompetent." This patient is experiencing a: a. self-esteem deficit. b. cognitive distortion. c. deficit in motivation. d. deficit in love and belonging.

ANS: B Automatic thoughts, or cognitive distortions, are irrational and lead to false assumptions and misinterpretations. See related audience response question.

The patient says, "My marriage is just great. My spouse and I always agree." The nurse observes the patient's foot moving continuously as the patient twirls a shirt button. The conclusion the nurse can draw is that the patient's communication is: a. clear. b. mixed. c. precise. d. inadequate.

ANS: B Mixed messages involve the transmission of conflicting or incongruent messages by the speaker. The patient's verbal message that all was well in the relationship was modified by the nonverbal behaviors denoting anxiety. Data are not present to support the choice of the verbal message being clear, explicit, or inadequate.

A nurse inspects an inpatient psychiatric unit and finds that exits are free of obstructions, no one is smoking, and the janitor's closet is locked. These observations relate to: a. coordinating care of patients. b. management of milieu safety. c. management of the interpersonal climate. d. use of therapeutic intervention strategies.

ANS: B Nursing staff are responsible for all aspects of milieu management. The observations mentioned in this question directly relate to the safety of the unit. The other options, although part of the nurse's concerns, are unrelated to the observations cited.

An appropriate intervention for a patient diagnosed with bulimia nervosa who binges and purges is to teach the patient: a. to eat a small meal after purging. b. not to skip meals or restrict food. c. to increase oral intake after 4 PM daily. d. the value of reading journal entries aloud to others.

ANS: B One goal of health teaching is normalization of eating habits. Food restriction and skipping meals lead to rebound bingeing. Teaching the patient to eat a small meal after purging will probably perpetuate the need to induce vomiting. Teaching the patient to eat a large breakfast but no lunch and increase intake after 4 PM will lead to late-day bingeing. Journal entries are private.

Which assessment finding for a patient in the community deserves priority intervention by the psychiatric nurse? The patient: a. receives Social Security disability income plus a small check from a trust fund every month. b. was absent from two of six planned Alcoholics Anonymous meetings in the past 2 weeks. c. lives in an apartment with two patients who attend partial hospitalization programs. d. has a sibling who was recently diagnosed with a mental illness.

ANS: B Patients who use alcohol or illegal substances often become medication noncompliant. Medication noncompliance, along with the disorganizing influence of substances on cellular brain function, promotes relapse. The distracters do not suggest problems.

Which activity is appropriate for a nurse engaged exclusively in community-based primary prevention? a. Medication follow-up b. Teaching parenting skills c. Substance abuse counseling d. Making a referral for family therapy

ANS: B Primary prevention activities are directed to healthy populations to provide information for developing skills that promote mental health. The distracters represent secondary or tertiary prevention activities.

Select the example of primary prevention. a. Assisting a person diagnosed with a serious mental illness to fill a pill-minder b. Helping school-age children identify and describe normal emotions c. Leading a psychoeducational group in a community care home d. Medicating an acutely ill patient who assaulted a staff person

ANS: B Primary preventions are directed at healthy populations with a goal of preventing health problems from occurring. Helping school-age children describe normal emotions people experience promotes coping, a skill that is needed throughout life. Assisting a person with serious and persistent mental illness to fill a pill-minder is an example of tertiary prevention. Medicating an acutely ill patient who assaulted a staff person is a secondary prevention. Leading a psychoeducational group in a community care home is an example of tertiary prevention.

Which personality characteristic is a nurse most likely to assess in a patient diagnosed with anorexia nervosa? a. Carefree flexibility b. Rigidity, perfectionism c. Open displays of emotion d. High spirits and optimism

ANS: B Rigid thinking, inability to demonstrate flexibility, and difficulty changing cognitions are characteristic of patients with eating disorders. The incorrect options are rare in a patient with an eating disorder. Inflexibility, controlled emotions, and pessimism are more the rule.

A nurse wants to enhance growth of a patient by showing positive regard. The nurse's action most likely to achieve this goal is: a. making rounds daily. b. staying with a tearful patient. c. administering medication as prescribed. d. examining personal feelings about a patient.

ANS: B Staying with a crying patient offers support and shows positive regard. Administering daily medication and making rounds are tasks that could be part of an assignment and do not necessarily reflect positive regard. Examining feelings regarding a patient addresses the nurse's ability to be therapeutic.

A patient is fearful of riding on elevators. The therapist first rides an escalator with the patient. The therapist and patient then stand in an elevator with the door open for five minutes and later with the elevator door closed for five minutes. Which technique has the therapist used? a. Classic psychoanalytic therapy b. Systematic desensitization c. Rational emotive therapy d. Biofeedback

ANS: B Systematic desensitization is a form of behavior modification therapy that involves the development of behavior tasks customized to the patient's specific fears. These tasks are presented to the patient while using learned relaxation techniques. The patient is incrementally exposed to the fear.

A nurse wants to find a description of diagnostic criteria for anxiety disorders. Which resource would have the most complete information? a. Nursing Outcomes Classification (NOC) b. Diagnostic and Statistical Manual of Mental Disorders (DSM-5) c. The ANA's Psychiatric-Mental Health Nursing Scope and Standards of Practice d. International Statistical Classification of Diseases and Related Health Problems (ICD-10)

ANS: B The DSM-5 details the diagnostic criteria for psychiatric clinical conditions. The other references are good resources but do not define the diagnostic criteria.

The nurse is planning a new sexuality group for patients. Which location would best enhance the effectiveness of this group? a. The hospital auditorium b. A small conference room c. A common area, such as a day room d. The corner of the music therapy room

ANS: B The conference room would provide a quiet, private area with few distractions, separate from other patient areas and effective for teaching and learning about a private topic. The auditorium is too large, and members' anxiety or lack of trust might lead them to spread out too far from each other, interfering with group process. The day room and the music therapy room are too busy and exposed, reducing privacy and increasing distractions.

Which assessment finding most clearly indicates that a patient may be experiencing a mental illness? The patient: a. reports occasional sleeplessness and anxiety. b. reports a consistently sad, discouraged, and hopeless mood. c. is able to describe the difference between "as if" and "for real." d. perceives difficulty making a decision about whether to change jobs.

ANS: B The correct response describes a mood alteration, which reflects mental illness. The distracters describe behaviors that are mentally healthy or within the usual scope of human experience.

As a patient diagnosed with a mental illness is being discharged from a facility, a nurse invites the patient to the annual staff picnic. What is the best analysis of this scenario? a. The invitation facilitates dependency on the nurse. b. The nurse's action blurs the boundaries of the therapeutic relationship. c. The invitation is therapeutic for the patient's diversional activity deficit. d. The nurse's action assists the patient's integration into community living.

ANS: B The invitation creates a social relationship rather than a therapeutic relationship.

Which component of treatment of mental illness is specifically recognized by Quality and Safety Education for Nurses (QSEN)? a. All genomes are unique. b. Care is centered on the patient. c. Healthy development is vital to mental health. d. Recovery occurs on a continuum from illness to health.

ANS: B The key areas of care promoted by QSEN are patient-centered care, teamwork and collaboration, evidence-based practice, quality improvement, safety, and informatics.

A student nurse says, "I don't need to interact with my patients. I learn what I need to know by observation." An instructor can best interpret the nursing implications of Sullivan's theory to this student by responding: a. "Interactions are required in order to help you develop therapeutic communication skills." b. "Nurses cannot be isolated. We must interact to provide patients with opportunities to practice interpersonal skills." c. "Observing patient interactions will help you formulate priority nursing diagnoses and appropriate interventions." d. "It is important to pay attention to patients' behavioral changes, because these signify adjustments in personality."

ANS: B The nurse's role includes educating patients and assisting them in developing effective interpersonal relationships. Mutuality, respect for the patient, unconditional acceptance, and empathy are cornerstones of Sullivan's theory. The nurse who does not interact with the patient cannot demonstrate these cornerstones. Observations provide only objective data. Priority nursing diagnoses usually cannot be accurately established without subjective data from the patient. The other distracters relate to Maslow and behavioral theory. This item relates to an audience response question.

Which nursing intervention has the highest priority as a patient diagnosed with anorexia nervosa begins to gain weight? a. Assess for depression and anxiety. b. Observe for adverse effects of refeeding. c. Communicate empathy for the patient's feelings. d. Help the patient balance energy expenditures with caloric intake.

ANS: B The nursing intervention of observing for adverse effects of refeeding most directly relates to weight gain and is a priority. Assessing for depression and anxiety, as well as communicating empathy, relate to coping. Helping the patient achieve balance between energy expenditure and caloric intake is an inappropriate intervention.

The parent of a child diagnosed with schizophrenia tearfully asks the nurse, "What could I have done differently to prevent this illness?" Select the nurse's best response. a. "Although schizophrenia results from impaired family relationships, try not to feel guilty. No one can predict how a child will respond to parental guidance." b. "Schizophrenia is a biological illness resulting from changes in how the brain and nervous system function. You are not to blame for your child's illness." c. "There is still hope. Changing your parenting style can help your child learn to cope effectively with the environment." d. "Most mental illnesses result from genetic inheritance. Your genes are more at fault than your parenting."

ANS: B The parent's comment suggests feelings of guilt or inadequacy. The nurse's response should address these feelings as well as provide information. Patients and families need reassurance that the major mental disorders are biological in origin and are not the "fault" of parents. One distracter places the burden of having faulty genes on the shoulders of the parents. The other distracters are neither wholly accurate nor reassuring.

A patient in a detoxification unit asks, "What good it will do to go to Alcoholics Anonymous and talk to other people with the same problem?" The nurse's best response would be to explain that self-help groups such as AA provide opportunities for: a. newly discharged alcoholics to learn about the disease of alcoholism. b. people with common problems to share their experiences with alcoholism and recovery. c. patients with alcoholism to receive insight-oriented treatment about the etiology of their disease. d. professional counselors to provide guidance to individuals recovering from alcoholism.

ANS: B The patient needs basic information about the purpose of a self-help group. The basis of self-help groups is sharing by individuals with similar problems. Self-help is based on the belief that an individual with a problem can be truly understood and helped only by others who have the same problem. The other options fail to address this or provide incorrect information.

When a nursing student expresses concerns about how mental health nurses "lose all their nursing skills," the best response by the mental health nurse is: a. "Psychiatric nurses practice in safer environments than other specialties. Nurse-to-patient ratios must be better because of the nature of the patients' problems." b. "Psychiatric nurses use complex communication skills as well as critical thinking to solve multidimensional problems. I am challenged by those situations." c. "That's a misconception. Psychiatric nurses frequently use high technology monitoring equipment and manage complex intravenous therapies." d. "Psychiatric nurses do not have to deal with as much pain and suffering as medical-surgical nurses do. That appeals to me."

ANS: B The practice of psychiatric nursing requires a different set of skills than medical-surgical nursing, though there is substantial overlap. Psychiatric nurses must be able to help patients with medical as well as mental health problems, reflecting the holistic perspective these nurses must have. Nurse-patient ratios and workloads in psychiatric settings have increased, just like other specialties. Psychiatric nursing involves clinical practice, not just documentation. Psychosocial pain and suffering are as real as physical pain and suffering.

Select all that apply. Which descriptors exemplify consistency regarding nurse-patient relationships? a. Encouraging a patient to share initial impressions of staff b. Having the same nurse care for a patient on a daily basis c. Providing a schedule of daily activities to a patient d. Setting a time for regular sessions with a patient e. Offering solutions to a patient's problems

ANS: B, C, D Consistency implies predictability. Having the same nurse see the patient daily and provide a daily schedule of patient activities and a set time for regular sessions will help a patient predict what will happen during each day and develop a greater degree of security and comfort. Encouraging a patient to share initial impressions of staff and giving advice are not related to consistency and would not be considered a therapeutic intervention.

Select all that apply. Which activities represent the art of nursing? a. Administering medications on time to a group of patients b. Listening to a new widow grieve her husband's death c. Helping a patient obtain groceries from a food bank d. Teaching a patient about a new medication e. Holding the hand of a frightened patient

ANS: B, C, E Peplau described the science and art of professional nursing practice. The art component of nursing consists of the care, compassion, and advocacy nurses provide to enhance patient comfort and well-being. The science component of nursing involves the application of knowledge to understand a broad range of human problems and psychosocial phenomena, intervening to relieve patients' suffering and promote growth. See related audience response question.

The next-to-last meeting of an interpersonal therapy group is taking place. The leader should take which actions? Select all that apply. a. Support appropriate expressions of disagreement by the group's members. b. Facilitate discussion and resolution of feelings about the end of the group. c. Encourage members to reflect on their progress and that of the group itself. d. Remind members of the group's norms and rules, emphasizing confidentiality. e. Help members identify goals they would like to accomplish after the group ends. f. Promote the identification and development of new options for solving problems.

ANS: B, C, E The goals for the termination phase of groups are to prepare the group for separation, resolve related feelings, and prepare each member for the future. Contributions and accomplishments of members are elicited, post-group goals are identified, and feelings about the group's ending are discussed. Group norms are the focus of the orientation phase, and conflict and problem solving are emphasized in the working phase.

Select all that apply. A novice nurse tells a mentor, "I want to convey to my patients that I am interested in them and that I want to listen to what they have to say." Which behaviors will be helpful in meeting the nurse's goal? a. Sitting behind a desk, facing the patient b. Introducing self to a patient and identifying own role c. Maintaining control of discussions by asking direct questions d. Using facial expressions to convey interest and encouragement e. Assuming an open body posture and sometimes mirror imaging

ANS: B, D, E Trust is fostered when the nurse gives an introduction and identifies his or her role. Facial expressions that convey interest and encouragement support the nurse's verbal statements to that effect and strengthen the message. An open body posture conveys openness to listening to what the patient has to say. Mirror imaging enhances patient comfort. A desk would place a physical barrier between the nurse and patient. A face-to-face stance should be avoided when possible and a less intense 90- or 120-degree angle used to permit either party to look away without discomfort.

A leader begins the discussion at the first meeting of a new group. Which comments should be included? Select all that apply. a. "We use groups to provide treatment because it's a more cost-effective use of staff in this time of budget constraints." b. "When someone shares a personal experience, it's important to keep the information confidential." c. "Talking to family members about our group discussions will help us achieve our goals." d. "Everyone is expected to share a personal experience at each group meeting." e. "It is important for everyone to arrive on time for our group."

ANS: B, E The leader must set ground rules for the group before members can effectively participate. Confidentiality of personal experiences should be maintained. Arriving on time is important to the group process. Talking to family members would jeopardize confidentiality. While groups are cost-effective, blaming the budget would not help members feel valued. Setting an expectation to share may be intimidating for a withdrawn patient.

As a nurse escorts a patient being discharged after treatment for major depression, the patient gives the nurse a necklace with a heart pendant and says, "Thank you for helping mend my broken heart." Which is the nurse's best response? a. "Accepting gifts violates the policies and procedures of the facility." b. "I'm glad you feel so much better now. Thank you for the beautiful necklace." c. "I'm glad I could help you, but I can't accept the gift. My reward is seeing you with a renewed sense of hope." d. "Helping people is what nursing is all about. It's rewarding to me when patients recognize how hard we work."

ANS: C Accepting a gift creates a social rather than therapeutic relationship with the patient and blurs the boundaries of the relationship. A caring nurse will acknowledge the patient's gesture of appreciation, but the gift should not be accepted. See relationship to audience response question.

A patient referred to the eating disorders clinic has lost 35 pounds during the past 3 months. To assess eating patterns, the nurse should ask the patient: a. "Do you often feel fat?" b. "Who plans the family meals?" c. "What do you eat in a typical day?" d. "What do you think about your present weight?"

ANS: C Although all the questions might be appropriate to ask, only "What do you eat in a typical day?" focuses on the eating patterns. Asking if the patient often feels fat focuses on distortions in body image. Questions about family meal planning are unrelated to eating patterns. Asking for the patient's thoughts on present weight explores the patient's feelings about weight.

When a new bill introduced in Congress reduces funding for care of persons with mental illness, a group of nurses writes letters to their elected representatives in opposition to the legislation. Which role have the nurses fulfilled? a. Recovery b. Attending c. Advocacy d. Evidence-based practice

ANS: C An advocate defends or asserts another's cause, particularly when the other person lacks the ability to do that for self. Examples of individual advocacy include helping patients understand their rights or make decisions. On a community scale, advocacy includes political activity, public speaking, and publication in the interest of improving the human condition. Since funding is necessary to deliver quality programming for persons with mental illness, the letter- writing campaign advocates for that cause on behalf of patients who are unable to articulate their own needs.

During which phase of the nurse-patient relationship can the nurse anticipate that identified patient issues will be explored and resolved? a. Preorientation b. Orientation c. Working d. Termination

ANS: C During the working phase, the nurse strives to assist the patient in making connections among dysfunctional behaviors, thinking, and emotions and offers support while alternative coping behaviors are tried.

A nurse conducting group therapy on the eating disorders unit schedules the sessions immediately after meals for the primary purpose of: a. maintaining patients' concentration and attention. b. shifting the patients' focus from food to psychotherapy. c. promoting processing of anxiety associated with eating. d. focusing on weight control mechanisms and food preparation.

ANS: C Eating produces high anxiety for patients with eating disorders. Anxiety levels must be lowered if the patient is to be successful in attaining therapeutic goals. Shifting the patients' focus from food to psychotherapy and focusing on weight control mechanisms and food preparation are not desirable. Maintaining patients' concentration and attention is important, but not the primary purpose of the schedule.

A 5-year-old child was diagnosed with encopresis. Which assessment finding would the nurse expect associated with this diagnosis? The child: a. frequently smears feces on clothing and toys. b. experiences frequent nocturnal episodes of bedwetting. c. has accidents of defecation at kindergarten three times a week. d. has occasional episodes of voiding accidents at the day care center.

ANS: C Encopresis refers to unsuccessful bowel control. Bowel control is expected by age 5, so frequent involuntary defecation is associated with this diagnosis. Smearing feces is behavioral. Enuresis refers to the voiding of urine during the day (diurnal) or at night (nocturnal).

A category 5 tornado occurred in a community of 400 people resulting in destruction of many homes and businesses. In the 2 years after this disaster, 140 individuals were diagnosed with posttraumatic stress disorder (PTSD). Which term best applies to these newly diagnosed cases? a. Prevalence b. Co-morbidity c. Incidence d. Parity

ANS: C Incidence refers to the number of new cases of mental disorders in a healthy population within a given period of time. Prevalence describes the total number of cases, new and existing, in a given population during a specific period of time, regardless of when they became ill. Parity refers to equivalence, and legislation required insurers that provide mental health coverage to offer annual and lifetime benefits at the same level provided for medical/surgical coverage. Co-morbidity refers to having more than one mental disorder at a time.

The spouse of a patient diagnosed with schizophrenia says, "I don't understand how events from childhood have anything to do with this disabling illness." Which response by the nurse will best help the spouse understand the cause of this disorder? a. "Psychological stress is the basis of most mental disorders." b. "This illness results from developmental factors rather than stress." c. "Research shows that this condition more likely has a biological basis." d. "It must be frustrating for you that your spouse is sick so much of the time."

ANS: C Many of the most prevalent and disabling mental disorders have strong biological influences. Genetics are only one part of biological factors. Empathy does not address increasing the spouse's level of knowledge about the cause of the disorder. The other distracters are not established facts.

A group is in the working phase. One member states, "That is the stupidest thing I've ever heard. Everyone whines and tells everyone else what to do. This group is a waste of my time." Which initial action by the group leader would be most therapeutic? a. Advise the member that hostility is inappropriate. Remove the member if it continues. b. Keep the group's focus on this member so the person can express the anger. c. Meet privately with the member outside of group to discuss the anger. d. Change to a more positive topic of discussion in this group session.

ANS: C Meeting privately with the member can convey interest and help defuse the anger so that it is less disruptive to the group. Removing the member would be a last resort and used only when the behavior is intolerably disruptive to the group process and all other interventions have failed. Decreasing the focus on the hostile member and focusing more on positive members can help soften the anger. Angry members often hide considerable vulnerability by using anger to keep others at a distance and intimidated. Changing the subject fails to respond to the behavior.

A nurse consistently encourages patient to do his or her own activities of daily living (ADLs). If the patient is unable to complete an activity, the nurse helps until the patient is once again independent. This nurse's practice is most influenced by which theorist? a. Betty Neuman b. Patricia Benner c. Dorothea Orem d. Joyce Travelbee

ANS: C Orem emphasizes the role of the nurse in promoting self-care activities of the patient; this has relevance to the seriously and persistently mentally ill patient.

Over the past year, a woman has cooked gourmet meals for her family but eats only tiny servings. This person wears layered loose clothing. Her current weight is 95 pounds, a loss of 35 pounds. Which medical diagnosis is most likely? a. Binge eating b. Bulimia nervosa c. Anorexia nervosa d. Eating disorder not otherwise specified

ANS: C Overly controlled eating behaviors, extreme weight loss, preoccupation with food, and wearing several layers of loose clothing to appear larger are part of the clinical picture of an individual with anorexia nervosa. The individual with bulimia usually is near normal weight. The binge eater is often overweight. The patient with eating disorder not otherwise specified may be obese. See relationship to audience response question.

A patient diagnosed with schizophrenia had an exacerbation related to medication noncompliance and was hospitalized for 5 days. The patient's thoughts are now more organized, and discharge is planned. The patient's family says, "It's too soon for discharge. We will just go through all this again." The nurse should: a. ask the case manager to arrange a transfer to a long-term care facility. b. notify hospital security to handle the disturbance and escort the family off the unit. c. explain that the patient will continue to improve if the medication is taken regularly. d. contact the health care provider to meet with the family and explain the discharge rationale.

ANS: C Patients do not stay in a hospital until every symptom disappears. The nurse must assume responsibility to advocate for the patient's right to the least restrictive setting as soon as the symptoms are under control and for the right of citizens to control health care costs. The health care provider will use the same rationale. Shifting blame will not change the discharge. Security is unnecessary. The nurse can handle this matter.

A patient in a group therapy session listens to others and then remarks, "I used to think I was the only one who felt afraid. I guess I'm not as alone as I thought." This comment is an example of: a. altruism. b. ventilation. c. universality. d. group cohesiveness.

ANS: C Realizing that one is not alone and that others share the same problems and feelings is called universality. Ventilation refers to expressing emotions. Altruism refers to benefitting by being of help to others. Group cohesiveness refers to the degree of bonding among members of the group.

A patient usually watches television all day, seldom going out in the community or socializing with others. The patient says, "I don't know what to do with my free time." Which member of the treatment team would be most helpful to this patient? a. Psychologist b. Social worker c. Recreational therapist d. Occupational therapist

ANS: C Recreational therapists help patients use leisure time to benefit their mental health. Occupational therapists assist with a broad range of skills, including those for employment. Psychologists conduct testing and provide other patient services. Social workers focus on the patient's support system.

The case manager plans to discuss the treatment plan with a patient's family. Select the case manager's first action. a. Determine an appropriate location for the conference. b. Support the discussion with examples of the patient's behavior. c. Obtain the patient's permission for the exchange of information. d. Determine which family members should participate in the conference.

ANS: C The case manager must respect the patient's right to privacy, which extends to discussions with family. Talking to family members is part of the case manager's role. Actions identified in the distracters occur after the patient has given permission.

A patient was hospitalized for 24 hours after a reaction to a psychotropic medication. While planning discharge, the case manager learned that the patient received a notice of eviction immediately prior to admission. Select the case manager's most appropriate action. a. Postpone the patient's discharge from the hospital. b. Contact the landlord who evicted the patient to further discuss the situation. c. Arrange a temporary place for the patient to stay until new housing can be arranged. d. Determine whether the adverse medication reaction was genuine because the patient had nowhere to live.

ANS: C The case manager should intervene by arranging temporary shelter for the patient until an apartment can be found. This activity is part of the coordination and delivery of services that falls under the case manager role. None of the other options is a viable alternative.

Select all that apply. Which statements most clearly reflect the stigma of mental illness? a. "Many mental illnesses are hereditary." b. "Mental illness can be evidence of a brain disorder." c. "People claim mental illness so they can get disability checks." d. "Mental illness results from the breakdown of American families." e. "If people with mental illness went to church, their symptoms would vanish."

ANS: C, D, E Stigma is represented by judgmental remarks that discount the reality and validity of mental illness. Many mental illnesses are genetically transmitted. Neuroimaging can show changes associated with some mental illnesses.

A nurse interacts with a newly hospitalized patient. Select the nurse's comment that applies the communication technique of "offering self." a. "I've also had traumatic life experiences. Maybe it would help if I told you about them." b. "Why do you think you had so much difficulty adjusting to this change in your life?" c. "I hope you will feel better after getting accustomed to how this unit operates." d. "I'd like to sit with you for a while to help you get comfortable talking to me."

ANS: D "Offering self" is a technique that should be used in the orientation phase of the nurse-patient relationship. Sitting with the patient, an example of "offering self," helps to build trust and convey that the nurse cares about the patient. Two incorrect responses are ineffective and non-therapeutic. The other incorrect response is therapeutic but is an example of "offering hope."

A nurse listens to a group of recent retirees. One says, "I volunteer with Meals on Wheels, coach teen sports, and do church visitation." Another laughs and says, "I'm too busy taking care of myself to volunteer to help others." Which developmental task do these statements contrast? a. Trust and mistrust b. Intimacy and isolation c. Industry and inferiority d. Generativity and self-absorption

ANS: D Both retirees are in middle adulthood, when the developmental crisis to be resolved is generativity versus self-absorption. One exemplifies generativity; the other embodies self-absorption. This developmental crisis would show a contrast between relating to others in a trusting fashion and being suspicious and lacking trust. Failure to negotiate this developmental crisis would result in a sense of inferiority or difficulty learning and working as opposed to the ability to work competently. Behaviors that would be contrasted would be emotional isolation and the ability to love and commit oneself.

A nurse is part of a multidisciplinary team working with groups of depressed patients. Half the patients receive supportive interventions and antidepressant medication. The other half receives only medication. The team measures outcomes for each group. Which type of study is evident? a. Incidence b. Prevalence c. Co-morbidity d. Clinical epidemiology

ANS: D Clinical epidemiology is a broad field that addresses studies of the natural history (or what happens if there is no treatment and the problem is left to run its course) of an illness, studies of diagnostic screening tests, and observational and experimental studies of interventions used to treat people with the illness or symptoms. Prevalence refers to numbers of new cases. Co-morbidity refers to having more than one mental disorder at a time. Incidence refers to the number of new cases of mental disorders in a healthy population within a given period. See related audience response question.

Outpatient treatment is planned for a patient diagnosed with anorexia nervosa. Select the most important desired outcome related to the nursing diagnosis Imbalanced nutrition: less than body requirements. Within 1 week, the patient will: a. weigh self accurately using balanced scales. b. limit exercise to less than 2 hours daily. c. select clothing that fits properly. d. gain 1 to 2 pounds.

ANS: D Only the outcome of a gain of 1 to 2 pounds can be accomplished within 1 week when the patient is an outpatient. The focus of an outcome would not be on the patient weighing self. Limiting exercise and selecting proper clothing are important, but weight gain takes priority.

The nursing care plan for a patient diagnosed with anorexia nervosa includes the intervention "monitor for complications of refeeding." Which system should a nurse closely monitor for dysfunction? a. Renal b. Endocrine c. Integumentary d. Cardiovascular

ANS: D Refeeding resulting in too-rapid weight gain can overwhelm the heart, resulting in cardiovascular collapse. Focused assessment is a necessity to ensure the patient's physiological integrity. The other body systems are not initially involved in the refeeding syndrome.

Which individual is demonstrating the highest level of resilience? One who: a. is able to repress stressors. b. becomes depressed after the death of a spouse. c. lives in a shelter for two years after the home is destroyed by fire. d. takes a temporary job to maintain financial stability after loss of a permanent job.

ANS: D Resilience is closely associated with the process of adapting and helps people facing tragedies, loss, trauma, and severe stress. It is the ability and capacity for people to secure the resources they need to support their well-being. Repression and depression are unhealthy. Living in a shelter for two years shows a failure to move forward after a tragedy. See related audience response question.

Although ego defense mechanisms and security operations are mainly unconscious and designed to relieve anxiety, the major difference is that: a. defense mechanisms are intrapsychic and not observable. b. defense mechanisms cause arrested personal development. c. security operations are masterminded by the id and superego. d. security operations address interpersonal relationship activities.

ANS: D Sullivan's theory explains that security operations are interpersonal relationship activities designed to relieve anxiety. Because they are interpersonal, they are observable. Defense mechanisms are unconscious and automatic. Repression is entirely intrapsychic, but other mechanisms result in observable behaviors. Frequent, continued use of many defense mechanisms often results in reality distortion and interference with healthy adjustment and emotional development. Occasional use of defense mechanisms is normal and does not markedly interfere with development. Security operations are ego-centered. This item relates to an audience response question.

A 26-month-old displays negative behavior, refuses toilet training, and often says, "No!" Which psychosocial crisis is evident? a. Trust versus mistrust b. Initiative versus guilt c. Industry versus inferiority d. Autonomy versus shame and doubt

ANS: D The crisis of autonomy versus shame and doubt relates to the developmental task of gaining control of self and environment, as exemplified by toilet training. This psychosocial crisis occurs during the period of early childhood. Trust versus mistrust is the crisis of the infant. Initiative versus guilt is the crisis of the preschool and early-school-aged child. Industry versus inferiority is the crisis of the 6- to 12-year-old child.

A patient's relationships are intense and unstable. The patient initially idealizes the significant other and then devalues him or her, resulting in frequent feelings of emptiness. This patient will benefit from interventions to develop which aspect of mental health? a. Effectiveness in work b. Communication skills c. Productive activities d. Fulfilling relationships

ANS: D The information given centers on relationships with others that are described as intense and unstable. The relationships of mentally healthy individuals are stable, satisfying, and socially integrated. Data are not present to describe work effectiveness, communication skills, or activities.

Select the best response for the nurse who receives a question from another health professional seeking to understand the difference between a Diagnostic and Statistical Manual of Mental Disorders (DSM-5) diagnosis and a nursing diagnosis. a. "There is no functional difference between the two. Both identify human disorders." b. "The DSM-5 diagnosis disregards culture, whereas the nursing diagnosis takes culture into account. c. The DSM-5 diagnosis describes causes of disorders whereas a nursing diagnosis does not explore etiology." d. "The DSM-5 diagnosis guides medical treatment, whereas the nursing diagnosis offers a framework for identifying interventions for issues a patient is experiencing."

ANS: D The medical diagnosis is concerned with the patient's disease state, causes, and cures, whereas the nursing diagnosis focuses on the patient's response to stress and possible caring interventions. Both tools consider culture. The DSM-5 is multiaxial. Nursing diagnoses also consider potential problems.

A patient was diagnosed with anorexia nervosa. The history shows the patient virtually stopped eating 5 months ago and lost 25% of body weight. The serum potassium is currently 2.7 mg/dL. Which nursing diagnosis applies? a. Adult failure to thrive related to abuse of laxatives as evidenced by electrolyte imbalances and weight loss b. Disturbed energy field related to physical exertion in excess of energy produced through caloric intake as evidenced by weight loss and hyperkalemia c. Ineffective health maintenance related to self-induced vomiting as evidenced by swollen parotid glands and hyperkalemia d. Imbalanced nutrition: less than body requirements related to reduced oral intake as evidenced by loss of 25% of body weight and hypokalemia

ANS: D The patient's history and lab result support the nursing diagnosis Imbalanced nutrition: less than body requirements. Data are not present that the patient uses laxatives, induces vomiting, or exercises excessively. The patient has hypokalemia rather than hyperkalemia.

A patient says, "People should be allowed to commit suicide without interference from others." A nurse replies, "You're wrong. Nothing is bad enough to justify death." What is the best analysis of this interchange? a. The patient is correct. b. The nurse is correct. c. Neither person is correct. d. Differing values are reflected in the two statements.

ANS: D Values guide beliefs and actions. The individuals stating their positions place different values on life and autonomy. Nurses must be aware of their own values and be sensitive to the values of others.

A nurse is talking with a patient, and 5 minutes remain in the session. The patient has been silent most of the session. Another patient comes to the door of the room, interrupts, and says to the nurse, "I really need to talk to you." The nurse should: a. invite the interrupting patient to join in the session with the current patient. b. say to the interrupting patient, "I am not available to talk with you at the present time." c. end the unproductive session with the current patient and spend time with the interrupting patient. d. tell the interrupting patient, "This session is 5 more minutes; then I will talk with you."

ANS: D When a specific duration for sessions has been set, the nurse must adhere to the schedule. Leaving the first patient would be equivalent to abandonment and would destroy any trust the patient had in the nurse. Adhering to the contract demonstrates that the nurse can be trusted and that the patient and the sessions are important. The incorrect responses preserve the nurse-patient relationship with the silent patient but may seem abrupt to the interrupting patient, abandon the silent patient, or fail to observe the contract with the silent patient.

S/S Of alcoholism

Absenteeism, drowsiness, slurred speech, inattention to appearance, increasing isolation, "secretive" disappearances, Tremors, flushed face, watery or reddened eyes, spacie-ness, breath odor of alcohol or strong mint smell. High number of physical complaints Raiding the medicine cabinet. Denial prevents the individual from linking problems in life to alcohol or drugs.

SORDES

Accumulation of foul matter (food, microorganisms, and epithelial elements) in the mouth

Crisis intervention: Phase 3 Intervention

Actions identified in planning phase are implemented, reality-oriented approach to address immediate basic needs, problem-solving

Paying attention to patient. Face the pt., maintain eye contact, be open, be alert and be patient.

Active listening

Increase in goal-directed activity/psychomotor agitation

Activities become obsessive, increased speed of completion

symptoms during or immediately after the distressing event

Acute Stress Disorder

Alcoholic Encephalopathy (Wernicke's encephalopathy) p. 302

Acute phase of alcohol-induced persisting amnestic disorder; a neurologic disease associated with chronic alcoholism and is characterized by ataxia, sixth cranial nerve palsy, nystagmus, and confusion

alcohol-induced persisting amnestic disorder (Korsakoff's syndrome) p. 302

Acute phase of alcohol-induced persisting amnestic disorder; a neurologic disease associated with chronic alcoholism and is characterized by ataxia, sixth cranial nerve palsy, nystagmus, and confusion

Patient is seeking ways to deal with stress and to resolve it

Adaptation

Solves the problem that is causing the anxiety, so the anxiety is decreased. The patient is objective, rational, and productive

Adaptive Coping

Patient has pancreatic cancer. To help patient cope, the nurse would use

Adaptive measures

When a client with paranoid schizophrenia tells the nurse "I have to get away. The volmers are coming to execute me," an appropriate response for the nurse would be A. "You are safe here. This is a locked unit and no one can get in." B. "I do not believe I understand the word volmers. Tell me more about them." C. "Why do you think someone or something is going to harm you?" D. "It must be frightening to think something is going to harm you."

"It must be frightening to think something is going to harm you."

A female patient tells the nurse that she would like to begin taking St. John's Wort for depression. What teaching should the nurse provide? 1. "St. John's wort should be taken several hours after your other antidepressant." 2. "St. John's wort has generally been shown to be effective in treating depression." 3. "This supplement is safe to take if you are pregnant." 4. "St. John's wort is regulated by the FDA, so you can be assured of its safety."

"St. John's wort has generally been shown to be effective in treating depression."

disulfiram (Antabuse)

-causes adverse reactions when person drinks -last dose will last up to 2 weeks - must know about and stay away from "hidden" alcohol in food, medicines, and preparations that are applied to the skin

LAAM (l-a-acetylmethadol)

-for opioid (heroin) addiction -Only need every 3 days *Addictive* narcotic, similar to morphine

Depersonalization

-is a non specific feeling that a person has lost his or her identity, that the self is different or unreal.

acamprosate (Campral)

-treat alcoholism; maintain abstinence *long term* -works to reduce the intake of alcohol by suppressing excitatory neurotransmission and enhancing inhibitory transmission (suppresses the food feeling you get by using the substances)

naltrexone (ReVia)

-used for narcotic addition and alcoholism -blocks opiate receptors reducing or eliminating the craving -low toxicity, with very few side effects -*Not* addictive -*Only need to take every 3 days*

topiramate (Topamax)

-works to decrease alcohol cravings by inhibiting the release of dopamine -reduces the pleasurable effects of substance

Explain the assessment guidelines for chemically impaired patients

1) check for severe or major withdrawal syndrome 2) check for overdose to a drug or alcohol 3)check for suicidal thoughts/self-destructive behavior 4)check for physical complications related to drug abuse 5)does patient want to fix substance problem? 6)does patient/family know about community resources for drug/alcohol treatment

What's the DSM criteria for substance-induced delirium

1) impaired consciousness 2)change in cognition:memory, impairment, disorientation) 3)develops over short period of time and fluctuates over day 4)evidence of substance abuse and withdrawal

Which 2 questions can be asked to detect alcohol or drug problems?

1) in the last year, have you ever drunk or used more drugs than you meant to? 2) have you ever felt you wanted or needed to cut down on your drinking or drug use in the last year? ** red flags: rationalizations, automatic responses as if question were predicted, slow, prolonged response( thinking of what to say)

Explain the 3 basic concepts that are fundamental to 12 step programs

1) individuals with addictive disorders are powerless over their addiction, and their lives are unmanageable 2)although not responsible for disease, responsible for recovery 3)can no longer blame people, places, and things for addiction, must face problems and feelings

How is addiction characterized

1) loss of control of substance consumption 2) substance use despite associated problems 3)tendency to relapse

Nursing Guidelines for Dependent PD

1. Identify and help address current stresses. 2. Try to satisfy patient's needs at the same time that limits are set up in such a manner that patient does not feel punished and withdraw. 3. Be aware that strong countertransference often develops in clinicians because of patient's excessive clinging (demands of extra time, nighttime calls, crisis before vacations); therefore, supervision is well advised. 4. Teach and role-model assertiveness.

A psychiatric home health nurse makes home visits in a neighborhood that has a high incidence of reported crimes. What reasonable safety measures should the nurse implement during home visits?

1. Make all visits in the daytime 2. Ask for a police escort during visits 3. Call the client before arrival at the home 4. Carry a cell phone at all times 5. Ask to be accompanied on visits by a co-worker

Which of the following services are typically performed by psychiatric nurses employed in managed care organizations?

1. Performing mental health assessments 3. Providing stress management classes 4. Contacting medication prescribers to arrange for changes 5. Triaging initial requests for services

Nursing Guidelines for Narcissistic PD

1. Remain neutral; avoid engaging in power struggles or becoming defensive in response to the patient's disparaging remarks. 2. Convey unassuming self-confidence.

Nursing Guidelines for Schizotypal PD

1. Respect patient's need for social isolation. 2. Be aware of patient's suspiciousness, and employ appropriate interventions. 3. As with schizoid patient, perform careful diagnostic assessment as needed to uncover any other medical or psychological symptoms that may need intervention (e.g., suicidal thoughts).

While taking care of a client at the mental health clinic, the student nurse notices scars and healing lacerations on the clients arms. The client states, I did all of that. The student nurse knows this type of behavior is: Standard Text: Select all that apply.

1. Self-destructive. 3. A maladaptive measure. 4. A coping mechanism.

Men and eating disorders

10% of people with eating disorders are men. More likely to be involved in athletics .."Make the weight" Diagnosis and treatment are same as with women Obese before more often then women. Less guilty feelings. Have trouble trusting. Advertisements and movies may play a part

5 stages of Maslows

1Physiological Needs (food, shelter, clothing) 2Safety and security (Family, job, and finacial security) 3Love and Belonging (Work group, social group) 4Esteem needs (Recongnize ME) 5Self actualization (Achievments, success)

The nurse knows that when performing a lethality assessment, asking the client, Have you ever thought of taking your own life? may: Standard Text: Select all that apply.

2. Alleviate the clients anxiety about considering suicide. 3. Be direct enough to promote effective communication. 5. Cause the client to open up about other attempts.

The psychiatric nurse knows that maladaptive grief reactions and perceptions of blame are reduced in nurse-led counseling programs. Nursing students should be taught to include the following outreach services for families and significant others who survive suicide: Standard Text: Select all that apply.

2. Arrange for staff or a representative to attend any funeral services. 3. Involve families in psychoeducational and family network services. 4. Make frequent telephone calls to the family immediately after the suicide and periodically until the first anniversary of the death. 5. Involve the family in a bereavement support group.

neglect, p. 530

An act of omission that results in harm; includes lack of adequate physical care, nutrition, and shelter; unsanitary conditions; and lack of human contact and nurturance

tyramine, p. 378

An amino acid

Nurses' Observation Scale for Inpatient Evaluations (NOSIE) p. 209

An assessment tool designed specifically for use by inpatient nurses. Useful in quickly assessing client functioning in six areas, three on positive features, three on negative features (typically done within 3 days of admission)

lethality assessment

An attempt to predict the likelihood of suicide

Tangential reply p. 192

An inappropriate response to a statement in which the content of the statement is disregarded. The reply is directed toward an incidental aspect of the statement, the type of language used, the emotions of the sender, or another facet of the same topic.

): tolerance for the chemical is usually quite high. The substance now lead to loss of control over one's behaviors. Without the chemical life is miserable. Daily living becomes a nightmare. All waking energy and effort are focused on obtaining and using the now required substance

Chronic, late stage

Axis I

Clinical disorders and other conditions that may be a focus of clinical attention

A client is admitted to the psychiatric unit exhibiting behaviors indicating a high level of anxiety following a personal crisis. Which of the following communication skills should the nurse utilize when interacting with this client?

Closed-ended questions

The nurse educator is teaching a group of students about stigma. The educator states that stigma can affect the judgment of which of the following people about the person who is labeled as mentally ill?

Co-workers

Advanced signs of Lithium Toxicity

Coarse hand tremor, persistent gastrointestinal upset, mental confusion, muscle hyperirritability, electroencephalographic changes, incoordination, sedation

Emotional ambivalence

Coexistence of opposite emotions toward same object, person, or situation

Burnout p. 37

Condition in which health care professionals lose their concern and feeling for their clients and come to treat them in detached or even dehumanized ways; an attempt to cope with the intense stress of interpersonal work by distancing

control need, p. 630

Consists of the ability to take charge to a satisfactory degree, and the ability to establish and maintain a feeling of respect for the competence and responsibileness of others to a satisfying degree to yourself.

PERINEAL AND GENITAL CARE

Consists of washing the client's external genitalia and surrounding skin. Is routine performed when bathing any client and is provided more frequently to a client with an indwelling catheter or perineal infection or after perineal or after surgery and childbirth.

The nurse is told that the client most likely has the diagnosis of obsessive-compulsive disorder. The nurse is not sure of the assessment data and behaviors that accompany this disorder. Which action would be most appropriate for the nurse to take?

Consult the Diagnostic and Statistical Manual of Mental Disorders for diagnostic criteria.

eye movement desensitization reprocessing (EMDR), 734

Controversial intervention suggested for PTSD and dissociative identity disorder. Clients asked to recall traumatic memories or a feared stimulus while make a series of rapid lateral eye movements.

Student wakes up late for exam and feels too ill to take test because of headache. Does not realize that 2 hours of cramming left her unprepared.

Conversion

Which of the following activities is the central element that differentiates case management from other types of care?

Coordinating one episode of care across multiple treatment settings

Include efforts to directly face and handle the problem. Are aimed at making the threat less meaningful by changing one's perception

Coping mechanisms

In trying to understand other cultures, what should the nurse know about how cultural values influence health beliefs?

Cultural values may shape perceptions of health, disease, prevention, and treatment.

A nursing student new to psychiatric mental health nursing asks a peer what resources he can use to figure out which symptoms are present in a specific psychiatric disorder. The best answer would be: a. Nursing Interventions Classification (NIC) b. Nursing Outcomes Classification (NOC) c. NANDA-I nursing diagnoses d. DSM-5

D

Patient has lung cancer and blames it on bad genes.

Denial

stages of grieving:

Denial Anger Bargaining Depression/resignation Acceptance

Defense mechanisms used by substance abusers:

Denial Rationalization Projection

The nurse is researching statistics of the five psychiatric disorders that comprise the top 10 causes of disability worldwide. Given this information, the nurse chooses which of the following as a priority screening for clients?

Depression

What statement about the comorbidity of depression is accurate? A. Depression most often exists in an individual as a single entity. B. Depression is commonly seen among individuals with medical disorders. C. Substance abuse and depression are seldom seen as comorbid disorders. D. Depression may coexist with other disorders but is rarely seen with schizophrenia.

Depression is commonly seen among individuals with medical disorders.

Substance/medication-induced depressive disorder

Depression onset by use of substances or caused by effects of medical disorder

Therapeutic Touch (TT), 730

Developed by Dolores Krieger, nursing professor. intentionally directed process of energy exchange during which TT practitioner uses the hands as a focus to facilitate the healing process. Practiced by nurses alone.

Healing Touch, (HT), 730

Developed by Janet Mentagan, nurse. uses light touch or near-touch to activated the human energy system and direct it toward healing. practiced by nurses and broader variety of practitioners.

Social isolation r/t...

Developmental regression Egocentric behavior Fear of rejection or failure in interaction

The psychiatric mental health nursing student is preparing to attend a meeting of the psychiatric mental health care team to discuss possible updates to clients diagnoses. In preparing for this meeting, the nursing student should consult which of the following references?

Diagnostic and Statistical Manual of Mental Disorders

For Hallucinogen Dependence

Diazepam (Valium) terminates the episodes of anxiety, violence and paranoid ideations.

Nurse is assessing patients who are mentally ill. What is a characteristic of a mentally ill patient?

Differing from socially accepted behaviors

thought blocking, p. 347

Difficulty articulating a response, or stopping in midsentence

substance-induced sleep disorder, p. 295

Disorder characterized by sustained use of stimulants for staying awake, or of alcohol to induce sleep

Schizophreniform disorder, p. 340

Disorder very similar to schizophrenia except the person has not been ill for very long

Husband has a bad day at work and then comes home and takes it out on his wife.

Displacement

A 20 year old professional nursing student in the last semester of the program

Distance learning is a method of instructing pupils via the internet, Lectures and educational materials are delivered via the internet. Instead of working in a classroom, students work from home. Which student is the least likely to succeed in a distance learning course?

Nursing diagnosis for schizophrenia

Disturbed thought processes Disturbed sensory perception Social isolation Risk for violence Impaired verbal communication Self-care deficit Ineffective coping Disabled family coping Ineffective health maintenance Impaired home maintenance

This decrease in ________ in the frontal area of the brain may be responsible for negative symptoms of Schizophrenia

Dopamine

Side effects of antipsychotics

Drowsiness, dizziness, dry mouth, constipation, increased appetite, weight gain, ECG changes, EPS, hyperglycemia, diabetes

Side effects of mood stabilizers

Drowsiness, dizziness, headache, dry mouth, thirst, GI upset, N/V, fine hand tremors, HoTN, arrhythmias, pulse irregularities, polyuria, dehydration, weight gain, toxicity

mood stabilizers

Drugs used to control mood swings in patients with bipolar mood disorders.

Side effects of antidepressants (all)

Dry mouth, sedation, nausea, discontinuation syndrome **Others = priapism, hepatic failure**

The co-existence of substance abuse and psychiatric disorders within the same person

Dual Diagnosis

the presence of at least one psychiatric disorder in addition to a substance abuse or dependency problem

Dual Diagnosis

Joe is depressed and suicidal because he can't go to the bar every night for a drink.

Dual diagnosis

Diagnosis for schizophrenia

During a 1 month period... at least 2 or more positive or negative symptoms and... 1 or more areas of social or occupational dysfunction

Failure to warn, if it results in injury to the threatened person, can lead to civil damages for malpractice.

Duty to warn of threatened suicide or harm

Is not successful in reducing anxiety or solving the problem. Even minimal functioning becomes difficult, and new problems begin to develop.

Dysfunctional Coping

● The term e-learning places the emphasis on student learning and pedagogy ● Learning that is mediated by computer technology is termed " e-learning" ● Older terms, such as computer assisted instruction or computer aided instruction (CAI) and computer based learning (CBL), emphasized the technology ● Informatics is about using the best methods to manage information, and e-learning focuses on the appropriate use of computerized technology to achieve educational aims.

E-learning offers many benefits than more traditional training options, the following describes the e-learning basics, select all that apply

Mechanism to communicate with user

EBP is a process that has developed a need to improve the quality and manage the economics of health care delivery. The following are the components of EBP, except

For severely depressed-Electrical current passed through the brain for 0.2 to 8.0 seconds, causing a seizure -Patient receives 2-3 treatments per week, up to a total of 6-12 treatments.

ECT

Antipsychotics side effects produce

EPS,TD, anticholinergic effects, sedation, and orthostatic hypotension. Thorazine is Haldol

Monoamine Oxidase Inhibitors (MAOIs)

Efficacy similar to other antidepressants, but dietary restrictions and potential drug interactions make this drug less desirable Contraindicated in people taking other antidepressants Tyramine-rich food could bring about a hypertensive crisis

respite, 620

Elders can remain overnight in order to relieve family caregivers if they are feeling burdened.

electroconvulsive therapy (ECT), p. 378

Electroconvulsive therapy (ECT) A treatment procedure that involves inducing a brief convulsion by passing an electric current through the brain; used in the treatment of mood disorders in persons resistant to other known therapies; sometimes used in other types of severely disordered thinking and behavior

Suicide prevention hotline callers had a significant decrease in suicidality during the course of the telephone session when crisis hotline workers demonstrated:

Empathy, respect, and support.

He believed that each of his 8 stages of developement afforded oppurtunities for growth, even up to the acceptance of persons own death!

Eriksons

Positive symptoms

Excess of normal, delusions, hallucinations, disorganized speech, catatonic/disorganized behavior

Religiosity

Excessive demonstration of obsession with religious ideas and behavior

REDDENED OR ECCORIATED MUCOSA

Excessive dryness of buccal mucosa

HIRSUTISM

Excessive hair on a person's body and face, particularly in women

psychomotor agitation

Excessive motor and cognitive activity

hypersomnia, p. 364

Excessive sleep of abnormal depth and/or duration

Directly related to alcohol ingestion by the mother during pregnancy. Alcohol inhibits fetal development, particularly during the first trimester..Third leading cause of mental retardation. Totally preventable. Produces microcephaly

FETAL ALCOHOL SYNDROME

A working goal for the nurseclient relationship is to achieve:

Facilitative intimacy.

Unlawful restraint of an individual's personal liberty or the unlawful restraint or confinement of an individual.

False Imprisonment

Hallucinations

False sensory perceptions not associated with real external stimuli

Predisposing Factors

Family hx of depression Having experienced recent negative stressors Having childhood experiences in a negative home environment Lacking a social support system Having significant physical disease

anergy or anergia, p. 364

Fatigue and decreased energy that are characteristic symptoms of depression

Symptoms of postpartum depression

Fatigue, irritability, loss of appetite, sleep disturbances, loss of libido, concern about inability to care for infant

"TIGER is examining ways to reach out to nurses who lack the informatics skills that are needed to practice."

For a group of staff nurses, the nurse informaticist is explaining the function and conclusions of the Technology informatics guiding educational reform (TIGER) summit. Which of the following statements belongs in the presentation?

"TIGER is examining ways to reach out to nurses who lack the infomatiics skills that are needed to practice"

For a group of staff nurses, the nurse informaticist is explaining the function and conclusions of the technology informatics guiding educational reform (TIGER) summit.Which of the following statements belongs in the presentation?

Intellectual Disability p. 214

Formerly labeled mental retardation and describes impaired cognitive function and adapting.

Which behavior would be most characteristic of a client during a manic episode? A. Going rapidly from one activity to another B. Taking frequent rest periods and naps during the day C. Being unwilling to leave home to see other people D. Watching others intently and talking little

Going rapidly from one activity to another

A nurse is working with a student on the acute care psychiatric unit. The student asks the nurse why it is important to assess clients values when there are many other more important issues to attend. Which of the following replies should the nurse emphasize?

Health behaviors are strongly influenced by personal values.

Select the nursing diagnosis least likely to be chosen after analysis of data pertinent to a client with postpartum depression. A. Impaired parenting B. Ineffective role performance C. Health-seeking behaviors D. Risk for impaired parent/infant/child attachment

Health-seeking behaviors

What's the goal of relapse prevention?

Help the individual learn from these situations so that periods of sobriety can be lengthened over time and relapses aren't viewed as total failure

chakras, 727

Hindu tradition. Seven major chakras. Chakras are concentrated areas of energy vertically aligned through the center of the body from the crown of the head to the pelvis. Chakras influence physical body, emotions and spirit. Each corresponds to specific body structures and organs.

Mistrust vs. Trust

Hope

The charge nurse is reviewing the care plans for the clients on the unit. In several care plans, the nurse has noted that the words noncompliant and manipulative have been used to describe those clients with severe mental illness. The nurse plans on discussing this with the staff at the next unit meeting. Which of the following responses will demonstrate the charge nurses personal accountability to the staff?

How might these terms reflect negativity and stigma towards persons with mental illness?

Intrapersonal p. 4

How the person relates to oneself, the person's relationships within the mind or the self)

The nurse knows that including family members in the plan of care for a suicidal client is extremely important. Two important strategies that families need to know are: Standard Text: Select all that apply.

How to help loved ones avoid acting on suicidal thoughts. How to prevent suicide.

Side effects of MAOIs

Hypertensive crisis (brought on by eating tyramine), site reactions with transdermal

dopamine hypothesis, p. 342

Hypothesis that symptoms of schizophrenia may be related to overactive neuronal activity that is dependent on dopamine

The psychiatricmental health nurse is working with the new graduate nurse who is orienting to the psychiatric unit. Which comment by the new graduate indicates further clarification of the generalist-nursing role is needed?

I am a little nervous about conducting psychotherapy with clients.

A delusional client walks up to the nurse and says, I am the appointed overseer. Who are you and why are you here? The most therapeutic response is which of the following?

I am your nurse and I will be here to help you until suppertime.

A female client has made the decision to leave her husband, who abuses alcohol. She states she is very depressed. Which of the following statements best demonstrates the nurses empathy?

I can understand that you are feeling depressed right now. It must have been a very difficult decision to make. Ill sit here with you for a while.

In the immunization clinic, the nurse notices a client displaying tense body posture. Which of the following is the most therapeutic response for the nurse to make?

I notice you are clenching your fists.

The nurse is caring for a client with depression who is withdrawn. Which of the following statements suggests that the nurse is able to challenge his or her dogmatic beliefs?

I realize that clients with depression are not just avoiding their problems.

A nurse has completed orientation to a locked psychiatric unit. Which statement best demonstrates that the nurse is prepared to fulfill the professional role?

I will ask for support from colleagues when I need it.

The nurse is seeking supervision regarding the use of self-disclosure with a client who has anxiety. Which response by the nurse most accurately reflects an understanding of the therapeutic use of self-disclosure?

I will first ask myself whether what I am going to disclose meets the clients needs or just my own needs.

Pleasure principle (Freud)

ID

MoA of antidepressants

Increase concentration of norepi and serotonin in the body either by blocking their reuptake by neurons (TCAs, tetracyclics, SSRIs) or by inhibiting release of MAOIs

alcohol withdrawel Delirium tremens in 24 - 72 hours...s/s...

Increased graveness of Psychological signs esp. Illusion & Hallucination Librium / diazepam stat

Psychotic features

Indicates the presence of disorganized thinking, delusions or hallucinations

Which behavior exhibited by a patient with mania should the nurse choose to address first? 1. Indiscriminate sexual relations 2. Excessive spending of money 3. Declaration of "being at one with the world" 4. Demonstration of flight of ideas

Indiscriminate sexual relations

Nursing diagnosis for violent/aggressive pts

Ineffective coping, risk for self-directed violence, risk for other-directed violence

GINGIVITIS

Inflammation of the gums

STOMATITIS

Inflammation of the oral mucosa

PAROTITIS

Inflammation of the parotid salivary gland

GLOSSITIS

Inflammation of the tongue

ACNE

Inflammatory condition with papules and pustules

A nurse caring for a client who has been diagnosed with a personality disorder should expect that the client *will* exhibit which of the following characteristics? A. Frequent episodes of psychosis B. Constant involvement with the needs of significant others C. Inflexible and maladaptive responses to stress D. Abnormal ego functioning

Inflexible and maladaptive responses to stress

Side effects of SSRIs

Insomnia, agitation, headache, weight loss, sexual dysfuction, serotonin syndrome

The nursing assistant verbalizes to the psychiatric nurse that normal people dont have mental disorders. Which approach by the nurse would be best?

Instruct the nursing assistant that anyone can have a mental health problem.

flat affect, p. 337

Insufficiently intense emotional display in association with ideas or situations that ordinarily would call for a stronger response

To show no feelings of sadness over sons death

Intellectualization

Which behavior would be *inconsistent* with defining characteristics for the nursing diagnosis of ineffective coping? A. Difficulty in relationships B. High levels of anxiety C. Manipulation D. Interdependence

Interdependence

Death

Irreversible cessation of circulatory and respiratory functions or irreversible cessation of all functions of the entire brain, including the brainstem.

LIGH STROKING

Is designed primarily to be sedative. It is also used in the early stages of injury treatment

THE ROOT

Is embedded in the jaw and covered by a bony structure called CEMENTUM

Rich Site Summaries

It provides a clear and easy way of tracking information from a large number of sources, and nurses should be aware of the wealth of information available to them

Open access journals

It provides a clear and easy way of tracking information from a large number of sources, and nurses should be aware of the wealth of information available.

A client admitted to the inpatient psychiatric unit after a recent suicide attempt tells the nurse, Even though suicide is against my religion, I was in so much emotional distress that I didnt think I could keep on living. Im really struggling with my spiritual conscience and dont know what I should do. Which of the following nurse responses is most appropriate?

It sounds like spirituality plays a significant role in your life. Tell me what beliefs are most important to you.

A family member caring for a relative with dementia complains of exhaustion. Which of the following responses best conveys respect for this family members situation?

It sounds like you are overwhelmed. You may benefit from respite care services.

A client with chronic paranoid schizophrenia is scheduled to be discharged from the inpatient psychiatric unit in two days. The nurse is working with the clients family to develop a plan for managing psychotic symptoms and emergency behaviors after discharge. Which of the following nursing diagnoses is most appropriate?

Knowledge Deficit: Symptom Management related to inadequate understanding of disease processes

PETRISSAGE/KNEADING

Large pin of the skin The purpose is to stretch the underlying tissue, develop friction in the area and increase circulation around the joint.

Phase II: Prodromal Phase

Lasts from a few weeks-years manifested by deterioration in role functioning, social withdrawal, fxnal impairment, sleep disturbance, anxiety, irritability, depressed mood, poor concentration, and fatigue

Which of the following interventions are used by the nurse to demonstrate active listening?

Leaning in toward the client Nodding ones head in response to clients verbal comments

Low self-esteem r/t...

Learned helplessness Feelings of abandonment Impaired cognition fostering negative view of self

E-learning

Learning that is mediated by computer technology is termed?

The nurse is caring for a client who repeatedly talks about the role of spirituality in curing depression. Which approach best demonstrates the nurses acceptance of the client?

Listen to the client in a supportive manner.

Psychopathology p. 5

Literally, pathology of the mind; signs and symptoms of mental disorder

Mood stabilizing examples

Lithium carbonate, clonzepam, carbamazepine, valproic acid, lamotrigine, gabapentin, topiramate, verapamil

MoA of mood stabilizers

Lithium enhances reuptake of norepinephrine and serotonin in the brain lowering level in the body and resulting in decreased hyperactivity, role of anticonvulsants not understood

Typical meds used to treat bipolar disorder

Lithium, divalproex, carbamazepine

Pressured speech

Loud, intrusive, difficult to interpret

Intimacy vs. Isolation

Love

Severe depression

MDD, bipolar

A manic client in the acute phase is verbally and physically aggressive to himself. The nursing diagnosis Defensive coping related to biochemical changes as evidenced by aggressive verbal and physical behaviors has been identified. A desirable short-term goal would be that the client will A. Making no attempts at self harm within 12 hrs of admission. B. sleep soundly for 12 of the next 24 hours. C. willingly take prescribed medication as offered by staff within 24 hours of admission. D. develop psychomotor retardation associated with sedation from prescribed medication within 6 hours of admission.

Making no attempts at self harm within 12 hrs of admission.

Unsuccessful attempts to decrease the anxiety without attempting to solve the problem - the anxiety remains.

Maladaptive Coping

self-destructive behaviors

Maladaptive measures a person uses to restore inner equilibrium when overwhelmed or unable to cope with stressful life events

ineffective attempts to cope with anxiety

Maladaptive responses

Failure to exercise an accepted degree of professional skill that results in injury, loss, or damage

Malpractice

Which of the following statements regarding the impact of managed health care on the delivery of psychiatric services is true?

Managed mental health care has resulted in shorter, more intense psychiatric hospital stays to keep costs down.

Comorbidity p. 6

Many people with mental diseases have two or more psychiatric disorders--particularly depression, anxiety, and alcohol and other substance abuse---at any one given time or during their lifetime. (also known as co-occurring disorder.

Catatonic features

Marked by nonresponsiveness, extreme psychomotor retardation (may seem paralyzed), withdrawal, and negativity.

Stage II Acute Mania

Marked impairment in functioning of mood, cognition and perception, and activity and behavior, usually requires hospitalization

Suicidality

May exhibit suicidal behavior during a depressed or mixed episodic or when psychotic

Psychosis (bipolar symptom)

May experience hallucinations and delusions

Decreased need for sleep

May sleep for only 4-5 hours per night and wake up feeling rested

clinically significant behavioral or psychological syndrome or pattern that occurs in an individual outside of socially acceptable behavior

Mental illness

Allows care providers to observe and describe a client's behavior in an objective, nonjudgmental way

Mental status examination

Slight muscle tension, fidgeting, Energetic, Good eye contact, Emotional, slight irritability,Feeling challenged, Confident Alertness, Awareness of surroundings, Concentration, Accurate perceptions Attentiveness, Logical reasoning and problem-solving skills

Mild +1 ANXIETY LEVEL

delusions, p. 335

Mistaken or false beliefs about the self or the environment that are firmly held even in the face of disconfirming evidence

The nurse is assessing a client in the home. Given the nurses knowledge of the top 10 causes of disability worldwide, choose the priority area for data collection.

Mood and patterns of alcohol usage

What are people's reactions usually about?

More about that person than you

Age and depression

More common in younger and after age 65

massage, 729

Most helpful for people with pain and anxiety because it releases endorphisn.

The interdisciplinary treatment team is discussing appropriate strategies for a homebound client who has a history of medication nonadherence. Which of the following rationales should the nurse case manager use when a treatment team member suggests that the nurse case manager make daily home visits to administer medications?

My role as case manager is to remain objective and look at the whole picture. If I deliver direct care, I may lose this perspective.

Side effects of anticonvulsants

N/V, drowsiness, dizziness, blood dyscrasias, prolonged bleeding time, risk of severe rash, decreased efficacy with oral contraceptics

Is there a time line for a social relationship?

No time limitation in duration or frequency of contact and no planned termination

Phase I: Premorbid Phase

Normal functioning, shy and withdrawn, poor peer relationships, poor academic performance, antisocial behavior, withdrawn from society (stays at home)

Crises of anticipated life transitions

Normal life-cycle transitions that may be anticipated but over which the individual may feel a lack of control

Affective psychological management of A/V

Normalizing, validation, listening, exploring beliefs

psychomotor retardation, p. 364

Noticeable slowing of thinking, speech, and body movements seen in depression

● Cardiac functioning ● Pulmonary capacity ● Electronic signals

Now that many measurements taken from various type of imaging have become dignitized they can be entered directly from the patient into a computer program for analysis. Each of these applications is unique to measurements, select all that apply

Clinical Nurse Specialist (CNS) p. 22

Nurse with expanded/specialized role that prepares at the graduate level. Some specialize in psychiatric-mental health nursing, they can provide individual, group and family psychotherapy in a wide variety of settings and obtain 3rd party reimbursement in some states.

Self Assessment

Nurses can experience feelings of frustration, hopelessness, and annoyance. They can alter these responses by: Recognizing any unrealistic expectations they have of themselves or the client Identify feelings they are experiencing that originate with the client Understanding the part that neurotransmitters play in the client with depressed mood.

Which of the following qualifications best explains why nurses are better suited for the role of case management than social workers?

Nurses have thorough training in psychobiology and pharmacology.

What will be compared

Nurses who use the steps of EBP to formalize their questions about practice should use PICOT format C stands for?

State-Trait Anxiety Inventory p. 211

Objective personality test that is a paper-and-pencil self-report instrument that measures state (a transitory emotional state characterized by consciously perceived feelings of tension and apprehension and heightened autonomic nervous system activity) and trait anxiety (relatively stable individual differences in vulnerability to anxiety)

The nurse is documenting observations of client interactions during a group session. The nurse strives to document the behaviors of the client interactions with:

Objectivity.

Most commonly are related to cleanliness, dirt, and germs; aggressive and sexual impulses; health concerns; safety concerns; and order and symmetry.

Obsessive-compulsive disorder

The nurse is interviewing a Native American client who acknowledges seeing spirits. Which of the following actions will be most important for the nurse to take to assist in assessing this clients symptoms?

Obtain a profile of the clients cultural norms on which to interpret this symptom.

During an admission assessment on an adult unit, the nurse is thinking that the clients beliefs and actions regarding commonly accepted health practices are bizarre. To help establish the presence of a mental disorder, the nurse should first collect information about the clients: Standard Text: Select all that apply.

Occupational history. Psychiatric history. Culture.

secondary gain

One of two mechanisms believed to explain what a person "gets" from having a conversion disorder; this mechanism helps the person avoid a distressing, uncomfortable, or repugnant activity while at the same time receiving support from others

primary gain

One of two mechanisms believed to explain what a person "gets" from having a conversion disorder; this mechanism helps the person keep the psychologic need or conflict out of conscious awareness

PARTIAL BATH (ABBREVIATED BATH)

Only the parts of the client's body the might cause discomfort or odor.

-TO CLEAN THE MOUTH AND TEETH AS PART OF A PERSONAL HYGIENE ROUTINE -TO REMOVE SECRETIONS FROM UN UNCONCIOUS CLIENT -TO MAINTAIN INTACT AND WELL HYDRATED MUCOSA

PURPOSES OF MOUTH/ORAL CARE

-TO PROMOTE COMFORT AND CLEANLINESS - TO PROMOTE HEALING AFTER SURGERY OR CHILDBIRTH - TO PREVENT INFECTION IN A CLIENT WITH AN INDWELLING URINARY CATHETER

PURPOSES OF PERINEL-GENITAL CARE

-TO STIMULATE THE BLOOD CIRCULATION TO THE SCALP THROUGH MASSAGE TO CLEAN THE HAIR AND INCREASE THE CLIENT'S SENSE OF WELL -BEING

PURPOSES OF SHAMPOOING THE HAIR

Aggression characteristics

Pacing, restlessness, tense face/body, verbal/physical threats, suicide/homocide threats, loud voice, argumentative, increased agitation, overreaction to env't stimuli, panic anxiety, disturbed thought process, suspiciousness, disproportionate anger

Temporarily decreases the anxiety but does not solve the problem, so the anxiety eventually returns. Temporary relief allows the patient to return to problem solving

Palliative Coping

Actual flight, fight, or immobilization Suicide attempts or violence Depletion of body resources Eyes fixed, Hysterical or mute, Incoherent Feeling overwhelmed and out of control Rage , Desperation, Feeling totally drained Disorganized perceptions, Disorganized or irrational reasoning and problem solving Neologisms, Clang associations, Word salad Out of contact with reality, Personality disorganization

Panic 4+ ANXIETY LEVEL

Intense feeling of apprehension

Panic disorder

begins as a series of acute or unprovoked anxiety(panic) attacks involving an intense, terrifying fear. The attacks do not occur on exposure to an anxiety-causing situation as phobias do

Panic disorder

Wernicke's - Korsakoff syndrome

Paralysis of eye muscle Ataxia Amnesia & confabulation Peripheral Neuritis Muscular weakness

Perineal care

Part of the bed bath that is embarrassing for many clients.

Psychiatric history p. 205

Part of the psychiatric examination that includes chief complaint, present symptoms, previous hospitalizations and mental health treatment, family history, personal history, and personality

relapse, p. 339

Partial or complete return of symptoms, however brief or extended, after a period of stability

Waxy flexibility

Passive yielding of all movable parts of the body to any effort made at placing them in certain positions (if you move their limb to a certain position, they will stay there)

A high school nurse is sharing the results of a national survey with school faculty and staff. The nurse states that, if the students in their school are similar to those surveyed, close to one out of five have seriously considered suicide in the:

Past year.

OIL

Prevents the hair from breaking and scalp from drying

known facts would lead an ordinary person to believe that the detained person is mentally disordered and is a danger to self or others.

Probable cause. If probable cause exists, pt can be detained for observation and treatment

Comes home late from a dance and says its because her boyfriend would not bring her home on time. Blaming others.

Projection

The nurse is caring for a patient who exhibits disorganized thinking and delusions. The patient repeatedly states, "I hear voices of aliens trying to contact me." The nurse should recognize this presentation as which type of major depressive disorder (MDD)? 1. Catatonic 2. Atypical 3. Melancholic 4. Psychotic

Psychotic

The psychiatric nurse states that todays nursing practice is based on contemporary theories concerning the etiology of mental disorder. Given this theoretical basis, the nurse would most likely give priority to which of the following assessments? Standard Text: Select all that apply.

Psychotropic medications PET and CT scans of the brain

Initiative vs. Guilt

Purpose

Critical Thinking p. 43

Purposeful, reasonable, reflective thinking that drives problem solving and decision making and aims to make judgments based on evidence

A nurse is seeking consultation on strategies to cope with the potential for burnout while working on a psychiatric unit. Which of the following strategies demonstrates the nurses ability to reduce the occurrence of burnout?

Pursue personal needs for social interactions during days off.

Serotonin syndrome

Rapid development of hyperthermia, hypertension, rigidity, autonomic instability, and mental status changes that can include coma and delirium

What are some essential aspects to developing a healthy patient-nurse relationship?

Rapport, trust, respect, geniuneness, and empathy

Patient has schizophrenia. Says he cant go to work because they are all mean there. Really he just cant admit that his illness interferes with work.

Rationalization

Depression Nursing Diagnosis

Risk for suicide Hopelessness Ineffective coping Social isolation Spiritual distress Self care deficit

Assertive behaviors??

Role playing

Mental disorder that makes it difficult to tell the difference between real and unreal experiences, to think logically, to have normal emotional responses, and to behave normally in social situations.

Schizophrenia

Other psychotic disorders

Schizophreniform Brief psychotic disorder Schizoaffective disorder Delusional disorder Shared psychotic disorder( Folie a Deux) Induced or secondary psychosis Psychosis induced polydipsia

the process of isolating a person in a room in which they are physically prevented from leaving.

Seclusion

narcissism

Self-centered behavior in which one feels entitled to special favors due to a mistaken perception of oneself as the "center of the universe

self-serving roles, p. 628

Sometimes members of a group satisfy individual needs that are irrelevant to the group task and may also be negatively oriented to group maintenance functions.

Freud. Harmony=

Stability

What helps a hallucination pass more quickly?

Staying with the patient

F-Frequency

Symptoms occur most days in a week

PULP CAVITY

The center of the tooth that contains the blood vessels and nerves.

Resilience p. 4

The characteristic of being able to bounce back to normal function or an even higher level of functioning.

insomnia, p. 364

The chronic inability to get to sleep or to remain asleep during the usual sleep period

While helping an addicted individual plan for ongoing treatment, which of the following interventions is the first priority for a safe recovery? A. Securing ongoing support from at least two family members. B. The client needs to be employed. C. The client strives to maintain abstinence. D. A regular schedule of appointments with a primary care provider.

The client strives to maintain abstinence.

The nurse is writing a care plan for a client with schizophrenia. Which of the following interventions demonstrates that the nurse is working from the Medical model?

The client will learn about the therapeutic effects of medications.

The nurse is validating what was observed before documenting in the progress note. Validation is used as a mechanism to ensure which of the following? A) The client's affect is appropriate to the situation. B) The client's request is clarified. C) The client's perception of the response is communicated. D) The client's need for further intervention is understood.

The clients perception of the response is communicated

Data Coding

The coding may be generated by a computer program from measurements directly obtained through imaging or physiologic monitoring or entered into the computer by a patient or researcher from a printout or a questionnaire/survey into a database program.

Increased epinephrine and norepinephrine Increased endogenous opiate release Increased cortisol levels Increased peripheral sympathetic nervous system activity

The effects of stress can be observed. Objective measurements of structural and clinical changes in the body.

The new nurse is working with a preceptor on a medical-surgical unit. The nurse has just assessed a client and states to the preceptor, This client has many odd notions regarding several common health practices. He seems like a deviant to me. In planning a response, the preceptor is guided by:

The knowledge that beliefs and behaviors are judged by cultural and social considerations.

Which of the following is not related to the theory of successful versus disturbed communication patterns during an admission assessment?

The language level of the assessment nurse.

shaken baby syndrome, p. 530

The vigorous shaking of a baby held by the extremities or shoulders that causes whiplash-induced intracranial and intraocular bleeding

Which of the following communication theories provides the most appropriate rationale for a nursing intervention to utilize the perceived strengths of the client in promoting effective communication?

Therapeutic Communication Theory

Patient Simulators

This problem-based learning approach provides opportunities for the learner to develop higher-order thinking skills. Research reports on the use of patient simulators indicate that students consistently value the learning experiences.

Clicker technology

This refers to the personal response systems used in the traditional classroom to provide assessment-centered instruction.

Drill and practice

This software was among the first educational software introduced. It was relatively simple to produce, and it freed teachers from the mundane shores and repetitive teaching.

Symptoms of severe depression

Total despair, worthlessness, flat affect, psychomotor retardation, curled up position, absence of communication, delusional thinking, delusions of persecution and somatic delusions, confusion, suicidal thoughts, general slowing of whole body

Countertransference p. 53

Transference of feelings from the therapist to the client; the development by the helping person of counterproductive fantasies, feelings, and attitudes in response to the client's transference or personality; such feelings are counterproductive to the therapeutic process

light therapy

Treats seasonal affective disorder (SAD); scientifically proven to be effective, exposure to daily doses of intense light. Increases activity in the adrenal gland and the superchiasmatic nucleus.

Delirium tremens (DT's) p. 301

Tremor and clouding of consciousness that can accompany physiological withdrawal from alcohol

Eriksons 8 stages emotional development.

Trust vs. mistrust 0-18months Autonomy vs. Shame 18months-3years Initiative vs. Guilt 3-5years Industry vs. inferority 6-12years Identity vs. Isolation 18-30years Generativity vs. self absorption 30-65years integrity vs. despair 65-death

Which of the following interventions promotes mindful listening in any health care setting?

Turning off the television before interviewing a client

Schizophrenia, residual type, p. 339

Type of schizophrenia with at least one documented episode of schizophrenia and presents now without prominent positive symptoms of the illness. Negative symptoms like flat affect and inability to work are present, but prominent hallucinations, delusions, and disorganized thoughts and behavior are not.

schizophrenia, paranoid type, 338

Type of schizophrenia with prominent hallucinations and delusions. Delusions are often persecutory or grandiose, and they often connect into a somewhat organized story.

Anomic suicide

Type of suicide that occurs when the structure of society is weakened or disrupted and people feel hopeless and disillusioned.

withdrawal, p. 294

Uncomfortable and maladaptive physiological, cognitive, emotional, and behavioral changes associated with lowered blood or tissue concentrations of a substance after an individual has established some tolerance towards it, usually through heavy recent use

TARTAR

Untreated plaque, a visible hard deposit of plaque and dead bacteria that forms at the gum.

Use of open ended questions. Who, what, when, where, Whay did you say?, Tell me about it.

Use of questioning

Use eye contact, hear with all senses, attending posture, be patient, give feedback, ask questions, offer empathy and support, validate and clarify info.

Use of self

self-medicate, p. 318

Use substances such as drugs and alcohol to address symptoms of mental illness.

Webinars and teleconferences.

Used as continuing education or webinars, nurses can participate with experts on clinical topics anywhere internet service is available.

alternative medicine, 715

Used in place of conventional medicine.

Visualization, 724

Uses the healing power of a person's own imagination and positive thinking to create powerful mental pictures or images to reduce stress or promote healing. Some consider form of hypnosis.

reminiscence therapy, 616

Uses the recall of past events, feelings and thoughts to facilitate pleasure, quality of life, or adaption to present circumstances. Useful intervention for elders experiencing self-esteem disturbance, grief, hopelessness, powerlessness and altered role performance and social isolation.

Email notifications

Using technology to stay current is a smart decision. With registration at journal publisher websites thru this, this will be sent new content is available

Do-not-resuscitate (DNR or "no-code") order

Usually written by the physician for the patient who has a terminal illness or is near death, this order is usually based on the wishes of the patient and family that no cardiopulmonary resuscitation be performed for respiratory or cardiac arrest.

Recoil stage: Support groups and short-term counseling

Validation of victim and rights as a victim Referrals

post op ECT

Ventilate patient (100% O2) until breathing unassisted Monitor respirations Watch for post-ECT confusion Does cause confusion and disorientation Reorientation - person, place, time Evaluate for agitation on awakening; administer prn benzodiazepine Continue to monitor Document

psychological abuse, p. 528

Verbal assaults, threats, humiliation, and/or harassment

Agression

Verbal statements that are intended to threaten, consider the context

● Learners can proceed at a pace conducive to comprehension. ● Consistent instruction in a safe environment ● Learning offered at any site that has computer access ● Learners can skim through familiar content and focus on weak areas. ● 24-hours access

Virtual learning environment (VLes) are being used in nursing schools to instruct students. Which of the following features of VLes are considered advantages? All that apply should be selected.

biofeedback, 731

Visceral learning, known as biofeedback, is a technique for gaining conscious control over involuntary body functions such as blood pressure and heart rate which are mediated by the autonomic nervous system.

What's the best indicator to the difference between depression and normal angst?

Visible manifestation of behavioral change that lasts for several weeks

Posturing

Voluntary assumption of inappropriate or bizarre postures

-PILLOWS -MATTRESSES -BED BOARDS -FOOT BOARDS -CHAIR BEDS -FOOT BOOT

WHAT ARE THE SUPPORT DEVICES IN BED MAKING?

amnesia, clouded consciousness, memory loss, confabulation, peripheral neuropathy, it is caused by inadequate amounts of thiamine and niacin in diet, and neurotoxic nature of a alcohol.

Wernicke-Korsakoff Syndrome

Application

What bloom's taxonomy of learning indicates that the learner can connect the concept with other concept and apply it new ways?

Comprehension

What bloom's taxonomy of learning indicates that the learner can explain the concept?

Application

What bloom's taxonomy of learning indicates that the learner can understand the concepts well enough to apply them to a new situation?

A client states, I just know my brother will not come back from the war. Which of the following examples would be used to encourage the client to explore this concern?

What do you feel will happen to him?

An adolescent who is pregnant asks the nurse on the psychiatric unit, Do you think I should give my baby up for adoption? Which of the following responses best reflects the nurses empathy?

What do you feel would be the best thing for you to do?

The nurse has completed a new clients orientation to the inpatient psychiatric unit and asks if the client has any questions. Which of the following questions indicates further teaching is needed regarding the clients rights in a therapeutic environment?

What happens if I refuse to attend a group activity?

SPECIAL INTERVENTION ARE NEEDED

What intervention needed for unconscious clients and for clients recovering from eye surgery or having eye injuries, irritations, or infections

How the management and processing of information helps nurses enter, organize or retrieve information

Which of the following is the emphasis of nursing informatics research?

Geographic Resources Analysis Support System (GRASS)

Which of the following is the popular public-domain GIS product.

All of the options listed are correct

Which of the following is the use of computers in research?

THE NURSE

Who can appreciate the quantity of oil and dead skin cells produced when observing a person after removal of cast that has been on for 6 weeks.

While reviewing therapeutic communication techniques, a nursing student made a list of things not to do or say to a client. Which of the following comments should be on the students list?

Why do you think you will never get well?

Disturbed thought process r/t...

Withdrawal into self Underdeveloped ego Punitive superego Impaired cognition fostering negative perception of self/env't

result from a biological need that develops when the body becomes adapted to having the drug in the system.

Withdrawal symptoms

What action should the nurse take on learning that a manic client's serum lithium level is 1.8 mEq/L? A. Withhold medication and notify the physician B. Continue to administer medication as ordered C. Advise the client to limit fluids for 12 hours D. Advise the client to curtail salt intake for 24 hours

Withhold medication and notify the physician

In depth data collection, reality testing done, writing and journaling, promoting change

Working stage 2

The correct response of the nurse who is asked if Florence Nightingale had any impact on the role of the nurse in psychiatricmental health nursing should be which of the following?

Yes, Nightingale was among the first to note that the influence of nurses has psychological components.

In planning care for a client who is gaining mental stability, the nurse develops measures to confirm the clients view of self. Which of the following responses made by the nurse would be categorized as disturbed communication?

You are wrong.

deep breathing, 723

You move the diaphragm downward and fill the lower part of the lungs with air. The chest expands as the middle part fills with air, and the shoulders move upward as the upper part fills.

A new nurse is oriented to a position in a community health center that serves a diverse client population. The new nurse says, The first thing I need to do is learn everything possible about the cultures of all the clients. What is the best response staff can give the new nurse?

You need to first understand who you are.

The causation of schizophrenia is currently understood to be A. a combination of inherited and nongenetic factors. B. excessive amounts of the neurotransmitter dopamine. C. excessive amounts of the neurotransmitter serotonin. D. stress related.

a combination of inherited and nongenetic factors.

BACK RUB/ MASSAGE

a comfort measures that can aid relaxation, decrease muscle tension and may ease anxiety because the physical contact communicate caring.

psychomotor retardation

a generalized slowing of physical and mental reactions; seen frequently in depression, intoxications, and other conditions

mania

a mood disorder marked by a hyperactive, wildly optimistic state; decreased need for sleep

A client with paranoid schizophrenia tells the nurse "I have to get away. The volmers are coming to execute me." The term "volmers" can be assessed as A. a neologism. B. clang association. C. blocking. D. a delusion.

a neologism

A client with histrionic personality disorder winks at an attractive nurse and states, "You and I should be able to turn those resident physicians into jelly if you'd wear your skirts about two inches shorter." The nurse's reply should be based on the understanding that the client's use of seductive behavior is A. a response to stress. B. based on a need to dominate. C. seated in primitive rage. D. callous disregard for others.

a response to stress.

psychogerontology, 602

a subspecialty within gerontology that studies the psychosocial needs of elders.

A safe blood level of lithium is 0.6 and 1.2 milliequivalents per liter (mEq/L). Lithium toxicity can happen when this level reaches: a) 1.5 b) 2.1 c) 3.1

a) 1.5

It is a movement disorder that makes it hard for you to stay still. It causes an urge to move that you can't control. a) Akathisia b) Tardive Dyskinesia c) Dystonia

a) Akathisia

These drugs are atypical drugs except for this one: a) Chlorpromazine (Thorazine) b) Reperdone (Risperdal) c) Clozapine (Clozaril)

a) Chlorpromazine

It is a procedure which involves the application of the electrical stimulus through the electrodes placed on the client's temples or on the forehead just between each eye, & inducing a grand-mal seizure a) ECT b) MRI c) EEG

a) ECT

What are the side effects of typical antipsychotic drugs

a) EPS effects b) Neuroleptic Malignant Syndrome c) Agranulocytosis

The self or the Ï, known as the integrator of personality, part of the mind which acts with the outside world, partly conscious & partly unconscious, operates on reality principle - controls the demands of & mediates between the id & the superego. Developed during the toddler period. a) Ego b) Id c) Superego

a) Ego

It is an ability to use therapeutic tools appropriately a) Genuineness/Congruence b) Unconditional Positive Regard c) Empathy

a) Genuineness/Congruence

It is type of therapy where the clients participate in sessions with group of people who share a common purpose & are expected to contribute to the group to benefit others & to receive benefit from others in return a) Group Therapy b) Individual Therapy c) Somatic Therapy

a) Group therapy

It is a state of complete physical, mental, and social wellness, not merely the absence of disease or infirmity. (WHO) a) Health b) Mental Health c) Mental Illness d) Psychology

a) Health

It is a nurse role that acts as a symbol of health by serving as an example of healthful living a) Healthy Role Model b) Reality Based Model c) Therapist

a) Healthy Role Model

what type of factor influences mental illness that includes biologic makeup, intolerable or unrealistic worries or fears, inability to distinguish reality from fantasy, intolerance of life's uncertainties, a sense of disharmony in life, and a loss of meaning in one's life. a) Individual b) Interpersonal c) Social/Cultural

a) Individual

It is both a neurotransmitter and a hormone. It plays an important role in your body's "fight-or-flight" response. As a medication, it is used to increase and maintain blood pressure in limited, short-term serious health situations. Treatment of mania and depression. a) Norepinephrine b) Dopamine c) Serotonin

a) Norepinephrine

'The application of goal-oriented, purposeful activity, in the assessment & treatment of individuals with psychological, physical or developmental disabilities' (AOTA, 1972) ❒ first developed by Adolf Meyer a) Occupational b) Recreational c) Art d) Dance and Movement e) Psychodrama f) Music

a) Occupational

-Pleasure & gratification is achieved through the mouth -Period of complete dependence -Task is to distinguish self from mother -Primary narcissism (self-love) -Child develops body image or self-concept from the response of others What stage is this? a) Oral Stage b) Anal Stage c) Phallic Stage

a) Oral Stage

It is a role of a nurse that assists the patient in the performance of activities of daily living a) Parent Surrogate b) Patient Advocate c) Counselor

a) Parent Surrogate

The patient is taking an antipsychotic drug for about 2 weeks and is experiencing side effects such as tremors, rigidity and bradykinesia (sleepy and slowing of voluntary movements). As a student nurse, these side effects are called: a) Parkinson's Syndrome b) Tardive Dyskinesia c) Dystonia

a) Parkinson's Syndrome

largest part of the mind which exerts greatest influence in one's personality. Storehouse for all memories, feelings & responses experienced by the individual during his entire life. Memories cannot be recalled at will. Expressed in dreams, slip of the tongue, jokes. a) Unconscious b) Conscious c) Subconscious/Preconscious

a) Unconscious

TCA examples

amitryptiline (Elavil) clomipramine (Anafranil) desipramine (Norpramin) doxepin (Sinequan) imipramine (Tofranil) nortriptyline (Aventyl) protriptyline (Vivactil) trimipramine (Surmontil)

personality disorder

an enduring pattern of experience and behavior that deviates significantly from the expectations w/in the individuals culture.

(unwanted thought, idea, image or urge that a patient recognizes as time-consuming and senseless but that repeatedly intrudes into the consciousness despite attempts to ignore, prevent, or counteract it.

an obsession

maintenance roles, p. 628

are oriented toward building group-centered attitudes amonth the members and maintaining and perpetuating group-centered behavior.

task roles, p. 628

are related to the task of the group. The job of people assuming these roles is to facilitate and coordinate group efforts in the selection, definition and solution of a group problem.

The atypical antipsychotics

are used to treat the positive and negative s/s of schizophrenia. They have few or no EPS or TDs. They may improve the neurocognitive defects associated with schizophrenia.

A client has been receiving antipsychotic medication for 6 weeks. At her clinic appointment she tells the nurse that her hallucinations are nearly gone and that she can concentrate fairly well. She states her only problem is "the flu" that she's had for 2 days. She mentions having a fever and a very sore throat. The nurse should A. suggest that the client take something for her fever and get extra rest. B. advise the physician that the client should be admitted to the hospital. C. arrange for the client to have blood drawn for a white blood cell count. D. consider recommending a change of antipsychotic medication.

arrange for the client to have blood drawn for a white blood cell count.

A desired outcome for a client with schizophrenia who has a nursing diagnosis of Disturbed sensory perception: auditory hallucinations related to neurobiological dysfunction would be that the client will A. ask for validation of reality. B. describe content of hallucinations. C. demonstrate a cool, aloof demeanor. D. identify prodromal symptoms of disorder.

ask for validation of reality.

The *priority* nursing intervention for a client with borderline personality disorder is to A. protect other clients from manipulation. B. respect the client's need for social isolation. C. assess for suicidal and self-mutilating behaviors. D. provide clear, consistent limits and boundaries.

assess for suicidal and self-mutilating behaviors.

Alterations in speech:

associative looseness neologisms echolalia clang association word salad

Dysthymia cannot be diagnosed unless it has existed for A. at least 3 months. B. at least 6 months. C. at least 1 year. D. at least 2 years.

at least 2 years.

bipolar 1 disorder

at least one episode of mania alternates w/ major depression. Psychosis may accompany the manic episode.

codependent/enabling behaviors

attempt to control other people's drug use think about drug user too much find excuses for person's drug abuse feeling responsible for person drug/alcohol use

olanzapine (Zyprexa) and risperidone (Risperdal)

atypical antipsychotics that help w/ insomnia, anxiety, agitation and mood stabilizing properties. At times may be more effective in treatment than lithium.

The type of altered perception most commonly experienced by clients with schizophrenia is A. delusions. B. illusions. C. tactile hallucinations. D. auditory hallucinations.

auditory hallucinations.

ability to solve problems, fulfill one's capacity for love and work, cope with crises without assistance beyond the support of family or friends a) Health b) Mental Health c) Mental Illness d) Psychology

b) Mental Health

level of psychological well-being or an absence of mental illness a) Health b) Mental Health c) Mental Illness d) Psychology

b) Mental Health

What is the antidote for TCA overdose? a) Barbiturates b) Physostigmine salicylate c) Atropine

b) Physostigmine salicylate

-2-4 yrs: pre-conceptual development proceeds from sensory motor learning to pre-logical thought (understand symbols & learn languages to communicate) --Characterized by egocentricity -4-7 yrs: intuitive thought learns to understand (relationships: same-different, family placement, classes = books, birds) a) Sensorimotor b) Pre-Operational c) Formal operations

b) Pre-operational

It is an interpersonal process whereby the professional nurse practitioner through the therapeutic use of self, assist an individual, family, group or community to promote mental health, to prevent mental illness & suffering, to participate in the treatment & rehabilitation of the mentally ill & if necessary to find meaning in this experiences. a) Mental Illness b) Psychiatric Nursing c) ADPIE

b) Psychiatric Nursing

It is a framework focuses on disease approach such as identification of syndromes, establishment of diagnosis & search for etiologies. Mental illness have certain symptoms that can be classified & treated a) Behavioral framework b) Psychobiologic framework c) Humanistic Framework

b) Psychobiologic framework

❒ Its purpose is to increase enjoyment of life, stimulate activity & self expression, enhance socialization & counterbalance self-concern, conducted by trained professionals ❒ Includes activities such as arts & craft, audiovisual activities, dance, hobbies, games & sports, cooking, musical activities, special interest programs, nature & outing activities, etc. a) Occupational b) Recreational c) Art d) Dance and Movement e) Psychodrama f) Music

b) Recreational

These are the factors influencing Mental Health Development (Shives, 1994) except for: a) Heredity b) Social c) Childhood Experiences d) Life Circumstances

b) Social

what stage that evolves after the body's attempts to adapt to change fail to manage the stressors if appropriate interventions to reduce the stress are unsuccessful a) Stage of Resistance b) Stage of Exhaustion c) Alarm Reaction

b) Stage of Exhaustion

Respect a) Genuineness/Congruence b) Unconditional Positive Regard c) Empathy

b) Unconditional Positive Regard

what type of phase that mobilizes the body's defenses & homeostatic responses against the stressor - "fight or flight response" a) Stage of Resistance b) Stage of Exhaustion c) Alarm Reaction

c) Alarm Reaction

These are drugs which help reduce and control the psychotic symptoms such as delusions and hallucinations (Schizophrenia). a) Hypnotics b) Tranquilizers c) Antipsychotics (Neuroleptic)

c) Antipsychotics

the use of artistic activities such as painting & clay modeling in psychotherapy & rehabilitation a) Occupational b) Recreational c) Art d) Dance and Movement e) Psychodrama f) Music

c) Art

These are the early signs of lithium toxicity except for this one: a) Diarrhea b) Muscular Weakness c) Ataxia

c) Ataxia

These are the examples for antihistamines except for this one: a) Diphenhydramine b) Hydroxyzine c) Clozapine

c) Clozapine

Energy focused on gaining new skills & knowledge Behavior: sense of industry & mastery Learns control over aggressive & destructive impulses as child conforms to rules & restrictions Acquires friends, is preoccupied with peers of same sex Quiet stage What stage is this? a) Phallic Stage b) Oral Stage c) Latency Stage

c) Latency Stage

These are the anticonvulsant drugs except for this one: a) Carbamazepine b) Valporic Acid c) Lithium d) Clonazepam

c) Lithium

It is a disorder causing people to display abnormal behavior more consistently than most people a) Health b) Mental Health c) Mental Illness d) Psychology

c) Mental Illness

a clinically significant behavioral or psychological syndrome or pattern that occurs in an individual and is associated with present distress (e.g., a painful symptom) or disability (i.e., impairment in one or more important areas of functioning) or with a significantly increased risk of suffering death, pain, disability, or an important loss of freedom. a) Health b) Mental Health c) Mental Illness d) Psychology

c) Mental Illness

These are the sedative-hypnotic drugs except for this one: a) Flurazepam b) Triazolam c) Nadolol

c) Nadolol

Making global assumptions based on an isolated incident a) Selective abstraction b) Arbitrary inference c) Overgeneralization

c) Overgeneralization

It is a nurse role that enables the patient & his relatives to know their rights & responsibilities a) Teacher b) Counselor c) Patient Advocate

c) Patient Advocate

It is a type of intervention that alters causative or risk factors to hinder development of illness & promotion of mental health. (E.g. health education, stress reduction and counseling) a) Secondary b) Tertiary c) Primary

c) Primary

It is drug type that increases the level of serotonin and norepinephrine a) Typical Antidepressant b) MAOIs c) Tricyclic Antidepressant

c) Tricyclic Antidepressant

It is a role of a nurse that creates a therapeutic environment. a) Parent Surrogate b) Patient Advocate c) Ward-Manager

c) Ward Manager

Cluster A personality disorders

characteristics of eccentric and odd behaviors, such as social isolation and detachment. may also be perception distortions, unusual levels of suspiciousness, Paranoid PD, Schizoid OD, & Schizotypal PD

obsessive-compulsive PD

characteristics of perfectionism w/ a focus on orderliness and control. These people become so preoccupied w/ details and rules that they may not be able to accomplish a given task.

paranoid PD

characterized by distrust and suspiciousness toward others based on the belief that others want to exploit, harm, or deceive the person. these people are hypervigilant, anticipate hostility, and may provoke hostile responses by initiating a "counterattack." demonstrate jealousy, controlling behaviors, and unwillingness to forgive

Antipsychotic examples

chlorpromazine risperidone aripiprazole ziprasidone quetiapine

SSRI examples

citalopram (Celexa) escitlopram (Lexapro) fluoxetine (Luvox) paroxitine (Paxil) sertraline (Zoloft)

The physician tells the nurse "Mrs. G's appearance is that of a typical manic client." The nurse can expect Mrs. G to be attired in clothing that is A. dark colored and modest. B. colorful and outlandish. C. compulsively neat and clean. D. ill-fitted and ragged.

colorful and outlandish.

integrative medicine, 715

combines conventional medicine and CAM treatments for which there is quality evidence of safety and effectiveness. Is being used in more often in hospitals, medical centers and universities.

la belle indeifference

complete lack of concern about one's physical symptom; typically occurs in those experiencing conversion disorder

schizophrenia catatonic type, p. 339

complex disorder with an extremely varied presentation of cognitive, emotional, and behavioral symptoms

seclusion

consists of confining a person to a room or area where the person is physically prevented from leaving.

inclusion need, p. 630

consists of the ability to take an interest in others to a satisfactory degree and the ability to allow other people to take an interest in you to a satisfying degree to yourself. This need determines whether a person is outgoing or prefers privacy.

full-blown mania

constantly go from one activity, place, or project to another; many projects may be started, but few if any are finished.

the use of movement to promote increased awareness of the body & changes in feeling states, cognition & behavior a) Occupational b) Recreational c) Art d) Dance and Movement e) Psychodrama f) Music

d) Dance and Movement

It is a type of group therapy which its goal is to provide information on specific issues like assertiveness training, anger management, etc. a) Psychotherapy group b) Family Therapy c) Support Groups d) Educational Groups e) Self Help Groups

d) Educational Groups

A client who is dependent on alcohol and drinks several six-packs of beer daily tells the nurse "Alcohol is no problem to me. I can quit anytime I want to." The nurse can assess this statement as indicating A. denial. B. projection. C. rationalization. D. reaction formation.

denial.

suicide attempt

desperate call for help involving different levels of risk. The attempt may be planned to avoid serious injury, or it may be one in which the outcome depends upon the circumstances and is not under the individual's control

Schizophrenia is best characterized as A. split personality. B. multiple personalities. C. ambivalent personality. D. deteriorating personality.

deteriorating personality.

Avoidant personality disorder (APD)

disorder A personality disorder characterized by a pervasive pattern of social inhibition, feelings of inadequacy, and hypersensitivity to criticism ...

A descriptor for a subtype of schizophrenia is A. delusional. B. dissociated. C. disorganized. D. developmental.

disorganized.

a form of therapy in which an individual reenacts life situations in order to examine subjective experiences, promote insight & alter specific behavior patterns a) Occupational b) Recreational c) Art d) Dance and Movement e) Psychodrama f) Music

e) Psychodrama

It is type of group therapy which the meeting is not led by a professional and the members share a common experience a) Psychotherapy group b) Family Therapy c) Support Groups d) Educational Groups e) Self Help Groups

e) Self Help Groups

Dishonesty is hallmark of the illness

eating disorders

reality orientation, 617

emphasizes awareness of time, place, person and purpose. Approach provides consistency and a constant reminder to clients of where they are, why they are there and what is expected.

A person who covertly supports the substance-abusing behavior of another is called a(n) A. patsy. B. enabler. C. participant. D. minimizer.

enabler.

Dependent PD

establish relationships in which they are submissive, self-doubting, and avoid self responsibility. find it difficult to sustain autonomy and often seek out relationships in which they can be taken care of.

dialectical behavior therapy

evidence-based theory to successfully treat chronically suicidal pts w/ borderline personality disorder. combines cognitive behavioral techniques with mindfulness, which emphasizes being aware of thoughts and actively shaping them.

A client who has been assessed by the nurse as moderately depressed is given a prescription for daily doses of a selective serotonin reuptake inhibitor. The client mentions she will take the medication along with the St. John's wort she uses daily. The nurse should A. agree that taking the drugs at the same time will help her remember them daily. B. caution the client to drink several glasses of water daily. C. suggest that the client also use a sun lamp daily. D. explain the high possibility of an adverse reaction.

explain the high possibility of an adverse reaction.

schizotypal PD

expressed in strikingly odd characteristics, including magical thinking, derealization, perceptual distortions, and rigid, peculiar ideas. speech patterns may be distinctive and bizarre.

the use of music to provide a variety of listening & participatory experience adapted to the needs of the client a) Occupational b) Recreational c) Art d) Dance and Movement e) Psychodrama f) Music

f) Music

A client with severe depression has been regulated on a monamine oxidase inhibitor because trials of other antidepressants proved unsuccessful. She has a pass to go out to lunch with her husband. Given a choice of the following entrees, which can she safely eat? A. avocado salad plate. B. fruit and cottage cheese plate. C. kielbasa and sauerkraut. D. liver and bacon plate.

fruit and cottage cheese plate.

dissociative disorders

group of altered coping patterns in which the functions of memory, consciousness, identity, and/or perception of the environment is disrupted

Positive symptoms-

hallucinations delusions bizarre behavior speech pattern alterations

meditation, 726

helps one achieve inner peace and harmony by focusing uncritically on one thing at a time. Associated with various religious and cultural practices for thousands of years.

self-hypnosis, 726

hypnosis accomplished by oneself without the help of a second party as hypnotist--to achieve significant relaxation, reduce anxiety, manage low to moderate levels of chronic pain, make positive suggestions for change (lose weight, overcome fear of dark, insomnia) and to increase learning and remembering.

bipolar 2 disorder

hypomanic episodes alternate w/ major depression. No psychosis. euphoric w/ increase functioning. depression puts people at risk for suicide.

cyclothymia

hypomanic episodes alternate w/ minor depressive disorder (at least 2 yrs in duration). Episodes tend to be irritable.

Clients with personality disorders have various self-defeating behaviors and interpersonal problems despite having near-normal ego functioning and intact reality testing. A nursing diagnosis that addresses this sort of interpersonal dysfunction is A. spiritual distress. B. defensive coping. C. impaired social interaction. D. disturbed sensory perception.

impaired social interaction.

ego-syntonic

in concert with the person's sense of self.

Nurse is teasing patients. This is

inappropriate

amnesia

is a loss of or failure of memory caused by problems in the functioning of the memory areas of the brain. Can result from concussions, traumatic brain injury, alcoholism, or disorders of aging brain.

When a client with schizophrenia hears hallucinated voices saying he is a vile human being, the nurse can correctly assume that the hallucination A. is a projection of the client's own feelings. B. derives from neuronal impulse misfiring. C. is a retained memory fragment. D. may signal seizure onset.

is a projection of the client's own feelings.

anergia

lack of energy; passivity

Heterocyclic examples

maprotiline (Ludiomil) mirtazapine (Remeron) trazadone (Desyrell) nefazodone (Serzone) bupropion (Zyban, Wellbutrin)

histrionic PD

marked by emotional attention-seeking behavior in which the person needs to be the center of attention. -impulsive,melodramatic, flirtatious and provocative.

medical meditation, 727

medication combined with adaptations of kundalini yoga (which is much more than fitness-oriented practice). Used to target specific physiological effects associated with arthritis, anxiety disorder, diabetes, hypertension, etc.

what is the number one reason relapse usually occurs?

medication non-adherence

What are illusions?

misinterpretations, usually of a threatening nature, of objects in the environment

Borderline PD

most common and dramatic, is characterized by severe impairments in functioning; instability in emotion regulation, interpersonal relationships, impulsivity, identity or self-image distortions, and unstable mood.

Checks the light 17 times before leaving the room

obsessive compulsive

Munchausen by proxy syndrome (MBPS)

occurs when parents or caregivers deliberately induce sings of an illness in another person, usually their own child.

codependence/enabling

often exhibit over-responsible behavior doing things for people they can do for themselves care for others @ exclusion of own needs

The only class of commonly abused drugs that has a specific antidote is A. opiates. B. hallucinogens. C. amphetamines. D. benzodiazepines.

opiates.

Visualization, guided-imagery, prayer, meditation Deep-breathing Relaxation training Yoga Healthy lifestyle choices Avoiding smoking, drinking, and other substances Decreasing self-destructive behaviors, such as avoidance Engaging in laughter, hobbies, and noncompetitive activities that are fun Avoiding debt Affirmative, positive, and empowering thinking Increasing self-confidence Increasing self-awareness Listening to calming music Getting a massage, with or without aromatherapy

palliative techniques taught

A depressed client is noted to pace most of the time, pull at her clothes, and wring her hands. These behaviors are consistent with A. senile dementia. B. hypertensive crisis. C. psychomotor agitation. D. central serotonin syndrome.

psychomotor agitation.

What has changed the most within the last 50 years in psych

psychotropic drugs

When a client experiences 4 or more mood episodes in a 12 month period, the client is said to be: A. dysynchronous. B. incongruent. C. cyclothymic. D. rapid cycling.

rapid cycling.

Describe flashbacks

recurrences of perceptual disturbance caused by a person's earlier hallucinogenic drug use when in a drug free state

What is the most common course of schizophrenia? Initial episode followed by A. recurrent acute exacerbations and deterioration. B. recurrent acute exacerbations. C. continuous deterioration. D. complete recovery.

recurrent acute exacerbations and deterioration.

Psychotic disorder

refers to delusions, any prominent hallucinations, disorganized speech, or disorganized catatonic behavior

6 year old wets the bed since the birth of his new sister.

regression

A newly admitted client has an axis II diagnosis of schizoid personality disorder. The nursing intervention of *highest* priority will be to A. set firm limits on behavior. B. respect need for social isolation. C. encourage expression of feelings. D. involve in milieu and group activities.

respect need for social isolation.

A client has a severe sleep pattern disturbance and psychomotor retardation. The nurse has developed a plan for him to spend 20 minutes in the gym at 1 PM. The hour immediately after the exercise period should be scheduled for A. rest. B. group therapy. C. individual therapy. D. occupational therapy.

rest.

The priority nursing diagnosis for a hyperactive manic client during the acute phase is A. risk for injury. B. ineffective role performance. C. risk for other-directed violence. D. impaired verbal communication.

risk for injury.

Splitting is a process in which the client A. unconsciously represses undesirable aspects of self. B. places responsibility for his or her behavior outside the self. C. sees things as divided into "all good" or "all bad." D. evidences lack of personal boundaries.

sees things as divided into "all good" or "all bad."

selective serotonin reuptake inhibitors (SSRIs)

selectively inhibits serotonin reuptake and results in potentiation of serotonergic neurotransmissions (Luvox, Paxilo, Prozac, Zoloft)

Assessment of thought processes of a client with depression is most likely to reveal A. good memory and concentration. B. delusions of persecution. C. self-deprecatory ideation. D. sexual preoccupation.

self-deprecatory ideation.

Obsessional pursuit, intimidation

stalking

hypnotherapy, 725

the therapeutic use of suggestion during an altered sate of consciousness to effect positive changes in a person's behavior and to treat a wide range of health conditions. May be effective in reducing pain, anxiety, stress, hypertension, insomnia and symptoms of irritable bowel syndrome. Has been used as adjunct to anesthesia or in place of. Also to treat tension and migraine headache, decrease dependence on tobacco (studies show not helpful), weight control and dentistry.

anticonvulsant drugs

this class of medications have demonstrated efficacy and been approved for the tx of mood disorders; (Depakote, Tegretol, Lamictal.)

Antisocial PD

this disorder reflects constant disregard for others through exploitation and repeated unlawful actions. No remorse for others, neglect responsibilities, tell lies, and perform destructive or illegal acts w/o developing any insight into consequences. *Psychopaths/Sociopaths*

A depressed client tells the nurse he is in the 'acute phase' of his treatment for depression. The nurse recognizes that the client has been in treatment: A. for more than 4 months B. that is directed toward relapse prevention C. that focuses on prevention of future depression D. to reduce depressive symptoms

to reduce depressive symptoms

Withdrawal & Detoxification. Abstinence

tremulousness, nervousness, anxiety, anorexia, nausea, vomiting, insomnia, tachycardia, elevated BP, diaphoresis, diarrhea, fever, ataxia, increased startle reflex, hallucinations and craving for alcohol/drugs. Care with drugs to manage DT's and decrease risk of seizures.

Client p. 4

used instead of the term patient to refer to persons who seek or receive mental health services. The term client implies empowerment, self-responsibility, respect and optimism that people are capable of change. Patient is associated with traditional sick role.

Serotonin-Norepinephrine Reuptake Inhibitors (SNRIs) examples

venlafaxine (Effexor) duloxetine (Cymbalta)

Intimidation, ridicule, aggressive posturing, rude gestures, threats, ostracism, offensive notes/e-mails

verbal abuse

A syndrome that occurs after stopping use of a drug is A. amnesia. B. tolerance. C. enabling. D. withdrawal.

withdrawal.

People are motivated by unconscious desires and conflicts. Personality is developed by early adulthood.

Freud

Says personality consists of 3 processes.,ID, Ego and superEgo. PDefense mechanisms protect against anxiety Change is a process of insight ersonality is formed the first 6 years of life.

Freud

Excessive force used to restrain a patient : false imprisonment and battery. Preventing a patient from leaving a healthcare facility : false imprisonment. Wrongfully committing a patient to a psychiatric facility : false imprisonment.

EXAMPLES OF FALSE IMPRISONMENT

Physiologic data

Each of these application is unique to the measures, such as systems to capture cardiac functioning and/or pulmonary capacity, devices that relay contractions, or monitors that pick up electronic signals.

What's the avg onset of a first manic episode for bipolar disorder?

Early 20s

They go out to a dance and patient states that the boyfriend would not bring her home on time. What defense mechanism is she using?

Projection

What is "sense of self"?

The uniqueness and individuality a person feels

A response to feeling mistreated, injured, or insulted. May be directed inward or outward toward other. May be an irrational response to minor events of the day or interrupt the person's social functioning

Anger

Adolescent depression

Anger, aggressiveness, running away, delinquency, social withdrawal, sexual acting out, substance abuse, restlessness, apathy

Symptoms of mild depression

Anger, anxiety, tearfulness, regression, preoccupation with loss, anorexia, insomnia

Which of the following would be assessed as a negative symptom of schizophrenia? A. Anhedonia B. Hostility C. Agitation D. Hallucinations

Anhedonia

Refusal to maintain normal body weight Intense fear of gaining weight or of being fat, although underweight. Body image disturbance or denial of seriousness of low weight. Absence of three consecutive menstrual cycles

Anorexia Nervosa

situational crisis

A crisis that originates from one of three sources: material or environmental (fire or natural disaster); personal or physical (heart attack, diagnosis of fatal illness, bodily disfigurement); and interpersonal or social (death of a loved one or divorce)

Chronic sorrow

A cyclical, recurring, and potentially progressive pattern of pervasive sadness experienced in response to continual loss, throughout the trajectory of an illness or disability.

Blackouts p. 302

Anterograde amnesia; loss of short-term memories with retention of remote memories; can occur in alcoholic patients

Identifications

...

Interpersonal theory

...

Isolation

...

Projection

...

Psychoanalytic theory

...

Rationalization

...

Regression

...

Reverberation

...

Sublimation

...

Supression

...

Theory of moral development

...

Theory of object relations

...

Theory of psychosocial development

...

Types of perception

...

Undoing

...

delusional disorder, p. 341

...

grandiosity

(inflated self-regard) apparent in both ideas expressed and the person's behavior. People w/ mania may exaggerate their achievements or importance.

What are some signs of alcohol withdrawal delirium

*medical emergency* Tachycardia diaphoresis hypertension disorientation hallucinations change in LOC paranoid delusions, agitation fever

Beliefs

Conceptions that people accept as true, concerning how the world operates and where the individual fits in relationship to others

alcohol withdrawal delirium

*medical emergency* Usually die of sepsis, MI,fat embolism,electrolyte imbalance, suicide, aspiration pneumonia, or peripheral vascular collapse -delirium state usually peaks 2-3 days after last drink and lasts 2-3 days

You are attempting to interview Mr. Jones, a newly admitted involuntary patient with schizophrenia. Mr. Jones seems evasive and uncomfortable and gives one-word responses that are minimally informative. Which response would be most useful for facilitating the interview? 1. "Why did you come to the hospital today?" 2. "It must be difficult to be admitted to a hospital against your will." 3. "If you could cooperate for just a few minutes, we could get this done." 4. "Did your schizophrenia get worse because you stopped taking your medication?"

"It must be difficult to be admitted to a hospital against your will."

What critical information should the nurse provide about the use of lithium? 1. "You will still have hypersexual tendencies, so be certain to use protection when engaging in intercourse." 2. "Lithium will help you to only feel the euphoria of mania but not the anxiety." 3. "It will take 1 to 2 weeks and maybe longer for this medication to start working fully." 4. "This medication is a cure for bipolar disorder."

"It will take 1 to 2 weeks and maybe longer for this medication to start working fully."

Which patient statement indicates learned helplessness? 1. "I am a horrible person." 2. "Everyone in the world is just out to get me." 3. "It's all my fault that my husband left me for another woman." 4. "I hate myself."

"It's all my fault that my husband left me for another woman."

A manic client tells a nurse "Bud. Crud. Dud. I'm a real stud! You'd like what I have to offer. Let's go to my room." The best approach for the nurse to use would be A. "What an offensive thing to suggest!" B. "I don't have sex with clients." C. "It's time to work on your art project." D. "Let's walk down to the seclusion room."

"It's time to work on your art project."

A teaching need is revealed when a client taking disulfiram states A. "I usually treat heartburn with antacids." B. "I take ibuprofen or acetaminophen for headache." C. "Most over-the-counter cough syrups are OK for me to use." D. "I have had to give up using aftershave lotion."

"Most over-the-counter cough syrups are OK for me to use."

Mixed Message p. 187

Inconsistency between the verbal and nonverbal components. Communication in which the verbal and the nonverbal aspects contradict one another.

Denial

...

Displacement

...

THERAPEUTIC BATHS

- Given for physical effects - use to soothe irritated skin or to treat an area (e.g., the perineum) - generally taken in a tub one-third or one- half full

PERSONAL HYGIENE

- The self care by which people attend to each other to such functions as bathing, toileting, general body hygiene, and grooming. - It involves care of the skin ,hair, nails, teeth, oral and nasal cavities, eyes, ears, and perineal-genital areas.

buprenorphine (Subutex)

-blocks signs/symptoms of opioid withdrawal - partial opioid agnoist

Anxiety

. " I wonder if children who interact with therapy dogs have lower anxiety when they are in the hospital," a pediatric unit nurse says. Which of the following is the O in this PICOT question?

Webinars and Teleconferences

. It is used in continuing education or webinars, nurses can participate with experts on clinical topics anywhere internet service is available

A recent study about suicide risk based on the cultural worldview of African-American and European-Americans shows that:

. Resilience gives individuals more reasons to live.

Cognitive development theory

...

Compensation

...

Name 5 critical issues that arise in the first 6 months of sobriety

1) physical changes as body adapts to functioning w/out substances 2)Learning different responses to cues that would trigger substance abuse 3)Emotional responses now felt full strength 4) response of family/friends to new behavior 5)New coping skills must be developed to prevent relapse

What's the DSM criteria for substance dependence?

1) tolerance to drug 2)withdrawal syndrome 3)taken in larger amounts/for longer period than intended 4) unsuccessful or persistent desire to cut down or control use 5)^time in taking, getting, or recovering from substance 6)not showing up to important events 7)keep using even though you know its reason for problems

Nursing Guidelines for Paranoid PD

1. Avoid being too "nice" or "friendly." 2. Give clear and straightforward explanations of tests and procedures beforehand. 3. Use simple, clear language; avoid ambiguity. 4. Project a neutral but kind affect. 5. Warn about any changes, side effects of medication, and reasons for delay. Such interventions may help allay anxiety and minimize suspiciousness. A written plan may help encourage cooperation.

vegetative signs of depression

1. Change in bowel movement pattern (constipation) 2. Eating habits (anorexia) 3. Sleep 4. Disinterest in sex

Nursing Guidelines for Borderline PD

1. Set realistic goals, use clear action words. 2. Be aware of manipulative behaviors (flattery, seductiveness, instilling of guilt). 3. Provide clear and consistent boundaries and limits. 4. Use clear and straightforward communication. 5. When behavioral problems emerge, calmly review the therapeutic goals and boundaries of treatment. 6. Avoid rejecting or rescuing. 7. Assess for suicidal and self-mutilating behaviors, especially during times of stress.

Which of the following are included when documenting client education? Standard Text: Select all that apply.

1. The educational content discussed with the client 2. The clients response 3. The purpose for the educational interaction

Nursing Guidelines for Antisocial PD

1. Try to prevent or reduce untoward effects of manipulation (flattery, seductiveness, instilling of guilt): • Set clear and realistic limits on specific behavior. • Ensure that limits are adhered to by all staff. • Carefully document signs of manipulation or aggression. • Document behaviors (give times, dates, circumstances). Provide clear boundaries and consequences. 2. Be aware that antisocial patients can instill guilt when they are not getting what they want. Guard against being manipulated through feelings of guilt. 3. Substance abuse is best handled through a well-organized treatment program before counseling and other forms of therapy are started.

Nursing Guidelines for histrionic PD

1. Understand seductive behavior as a response to distress. 2. Keep communication and interactions professional, despite temptation to collude with the patient in a flirtatious and misleading manner. 3. Encourage and model the use of concrete and descriptive rather than vague and impressionistic language. 4. Teach and role-model assertiveness.

Which of the following programs are often available to mentally ill clients in community treatment settings?

2. Halfway houses 3. Sheltered workshops 4. Day treatment centers 5. Adult day care centers

MDD Signs and Symptoms

5 or more s/s must also be present one of which must be either a depressed mood or loss of interest in previously enjoyed activities At least 4 or more s/s which include: changes in appetite or weight, sleep disturbances, fatigue or loss of energy, feelings of worthlessness or guilt, difficulty concentrating, thinking, or making decisions, or recurrent thoughts of death or suicide. Additional s/s may be: bodily aches and pains, irritability, or crankiness rather than sadness, social withdrawal, and neglect of activities that previously brought pleasure. Many times begins in childhood

The child prescribed an antipsychotic medication to manage violent behavior is one most likely diagnosed with: a. attention deficit hyperactivity disorder. b. posttraumatic stress disorder. c. communication disorder. d. an anxiety disorder.

A Antipsychotic medication is useful for managing aggressive or violent behavior in some children diagnosed with attention deficit hyperactivity disorder. If medication were prescribed for a child with an anxiety disorder, it would be a benzodiazepine. Medications are generally not needed for children with communication disorder. Treatment of PTSD is more often associated with SSRI medications.

An adolescent was arrested for prostitution and assault on a parent. The adolescent says, "I hate my parents. They focus all attention on my brother, who's perfect in their eyes." Which type of therapy might promote the greatest change in the adolescent's behavior? a. Family therapy c. Play therapy b. Bibliotherapy d. Art therapy

A Family therapy focuses on problematic family relationships and interactions. The patient has identified problems within the family. Play therapy is more appropriate for younger patients. Art therapy and bibliotherapy would not focus specifically on the identified problem.

a repetitive, intentional, purposeful behavior performed to decrease the anxiety associated with an obsession

A compulsion

Parents of an adolescent diagnosed with a conduct disorder say, "We don't know how to respond when our child breaks the rules in our house. Is there any treatment that might help us?" Which therapy is likely to be helpful for these parents? a. Parent-child interaction therapy (PCIT) b. Behavior modification therapy c. Multi-systemic therapy (MST) d. Pharmacotherapy

A In parent-child interaction therapy (PCIT), the therapist sits behind one-way mirrors and coaches parents through an ear audio device while they interact with their children. The therapist can suggest strategies that reinforce positive behavior in the adolescent. The goal is to improve parenting strategies and thereby reduce problematic behavior. Behavior modification therapy may help the adolescent, but the parents are seeking help for themselves. Multi-systemic therapy is much broader and does not target the parents' need.

An adolescent diagnosed with conduct disorder has aggression, impulsivity, hyperactivity, and mood symptoms. The treatment team believes this adolescent may benefit from medication. The nurse anticipates the health care provider will prescribe which type of medication? a. Second-generation antipsychotic c. Calcium channel blocker b. Anti-anxiety medication d. Beta-blocker

A Medications for conduct disorder are directed at problematic behaviors such as aggression, impulsivity, hyperactivity, and mood symptoms. Second-generation antipsychotics are likely to be prescribed. Beta-blocking medications may help to calm individuals with intermittent explosive disorder by slowing the heart rate and reducing blood pressure. Calcium channel blockers reduce blood pressure but are not used for persons with impulse control problems. An anti-anxiety medication will not assist with impulse control.

Which assessment findings support a diagnosis of oppositional defiant disorder? a. Negative, hostile, and spiteful toward parents. Blames others for misbehavior. b. Exhibits involuntary facial twitching and blinking; makes barking sounds. c. Violates others' rights; cruelty toward people or animals; steals; truancy. d. Displays poor academic performance and reports frequent nightmares.

A Oppositional defiant disorder is a repeated and persistent pattern of having an angry and irritable mood in conjunction with demonstrating defiant and vindictive behavior. The distracters identify findings associated with conduct disorder, anxiety disorder, and Tourette's syndrome.

crisis intervention

A conceptual framework for intervention that calls for short-term, action-oriented assistance focused on problem-solving, with a goal of restoring the individual's equilibrium

A 16-year-old diagnosed with a conduct disorder has been in a residential program for 3 months. Which outcome should occur before discharge? a. The adolescent and parents create and agree to a behavioral contract with rules, rewards, and consequences. b. The adolescent identifies friends in the home community who are a positive influence. c. Temporary placement is arranged with a foster family until the parents complete a parenting skills class. d. The adolescent experiences no anger and frustration for 1 week.

A The adolescent and the parents must agree on a behavioral contract that clearly outlines rules, expected behaviors, and consequences for misbehavior. It must also include rewards for following the rules. The adolescent will continue to experience anger and frustration. The adolescent and parents must continue with family therapy to work on boundary and communication issues. It is not necessary to separate the adolescent from the family to work on these issues. Separation is detrimental to the healing process. While it is helpful for the adolescent to identify peers who are a positive influence, it's more important for behavior to be managed for an adolescent diagnosed with a conduct disorder.

Soon after parents announced they were divorcing, a child stopped participating in sports, sat alone at lunch, and avoided former friends. The child told the school nurse, "If my parents loved me, they would work out their problems." Which nursing diagnosis has the highest priority? a. Social isolation c. Chronic low self-esteem b. Decisional conflict d. Disturbed personal identity

A This child shows difficulty coping with problems associated with the family. Social isolation refers to aloneness that the patient perceives negatively, even when self-imposed. The other options are not supported by data in the scenario.

Which nursing diagnosis is universally applicable for children diagnosed with autism spectrum disorders? a. Impaired social interaction related to difficulty relating to others b. Chronic low self-esteem related to excessive negative feedback c. Deficient fluid volume related to abnormal eating habits d. Anxiety related to nightmares and repetitive activities

A Children diagnosed with autism spectrum disorders display profoundly disturbed social relatedness. They seem aloof and indifferent to others, often preferring inanimate objects to human interaction. Language is often delayed and deviant, further complicating relationship issues. The other nursing diagnoses might not be appropriate in all cases

DEEP STROKING

A THERAPEUTIC COMPRESSION OF SOFT TISSUE, WHICH ENCOURAGES VENOUS AND LYMPHATIC DRAINAGE

Aggressive behavior p. 40

A broad range of behaviors such as inconsiderate, offensive aggression, as well as destructive behaviors characterized, in the extreme, by violence; also includes behavior necessary for success and creativity

major depressive episode, p. 363

A change in several aspects of a person's life and emotional state that persists for at least 14 days

cyclothymic disorder, p. 368

A chronic, fluctuating mood disturbance characterized by numerous periods of hypomania alternating with numerous periods of depression

Conflict

A clash between opposing forces; may be conscious or unconscious, intrapersonal or interpersonal

learned helplessness, p. 370

A cognitive theory that proposes that learning plays an instrumental role in the development of depression. This theory holds that depression is based on the person's belief that the or she has no control over life situations.

What causes schizophrenia?

A combination of genetic, biochemical, physiological, and psychosocial factors/stressors/dysfunction

Schizoaffective disorder

A combination of the symptoms of schizophrenia and mood disorder

schizophrenia, p. 334

A complex disorder with an extremely varied presentation of cognitive, emotional, and behavioral symptoms

Acquiring knowledge

A nurse reads nursing research and a journal article about new patient care technology. This nurse practitioner's name is:

Acting out p. 52

A particularly destructive form of resistance in which the client puts into action (that is, "acts out") emotional conflicts. The client is externalizing an inner conflict to people in the immediate environment rather than verbalizing conflicts or feelings the client displays inappropriate behaviors.

substance dependence, p. 294

A pattern of substance use that is continued despite significant consequences, usually with physiological tolerance effects and a withdrawal syndrome if the substance is withdrawn

Borderline Personality disorder (BPD)

A personality disorder characterized by a pervasive pattern of unstable interpersonal relationships, self-esteem, mood swings, and impulsive behavior

obsessive-compulsive personality disorder (OCPD)

A personality disorder characterized by preoccupation with orderliness, structure, and rules. The individual is inflexible and indecisive.

schizoid personality disorder

A personality disorder characterized by the lack of enjoyment of close relationships and little, if any, desire for social involvement

Hospice

A philosophy of care rather than a program of care. It is a comprehensive and coordinated care for patients with limited life expectancy, provided at home, hospitals and long-term care facilities. It reaffirms the right of every patient and family to fully participate in the final stages of life.

affect, p. 363

An outward manifestation of a person's feelings or emotions in response to persons or events

EFFLEURAGE

A smooth, long stroke, moving the hands up and down at the back Consist of long, gliding strokes from the neck down to the base of the spine or from the fingertips up to the shoulder

Stigma p. 14

A societal perception that mental illness is a blemish of individual character. Is a world-wide problem but most prevalent in deprived, marginalized and minority communities.

Electroconvulsive therapy

A somatic treatment, mostly used for cases of severe depression, in which a brief electric current is passed through the brain to produce a grand mal seizure

hypochondriasis

A somatoform disorder characterized by the persistent belief or fear that one is ill; accompanied by no evidence of organic pathology

Mental Status Examination (MSE) p. 205

A structured interview to gather data on the person's present mental status; helps determines etiology, diagnosis, prognosis, and treatment, and immediately addresses any risk of violence or harm. Important in establishing the existence of delirium, dementia, amnestic, and other cognitive disorders.

"Nursing informatics is an important science used daily in the care of clients."

A student nurse inquires about how nurses use informatics in the clinical setting when conducting nursing research. What is the nurse instructor's best response?

suicide threat

A threat that is more serious than a casual statement of suicidal intent and that is accompanied by other behavior changes that may include mood swings, temper outbursts, a decline in school of work performance, personality changes, sudden or gradual withdrawal from friends, and other significant changes in attitude

The nurse assesses the wellness beliefs and values of a client from another culture best when asking: A) "What do you think is making you ill?" B) "What do you think will make you better?" C) "How can I help you get better?" D) "Did you do something to cause the illness?"

A) "What do you think is making you ill?"

The community mental health nurse establishes meeting a client's need for basic life necessities as being the highest priority. What other need is of high priority for a seriously and persistently ill client living in the community? A) Access to medication B) Socialization and diversion C) Independent decision making D) Engaging in meaningful work

A) Access to medication

Projection

An unconscious defense or coping mechanism in which what is emotionally unacceptable to the individual is rejected and attributed to others

Which activity best exemplifies an important focus for a case manager? A) Arranging for rapid assessment of a newly hospitalized client B) Providing a comprehensive client social history to the treatment team C) Writing a report describing best practices in care for clients with depression D) Gathering data for a research study concerning side effects of a new medication

A) Arranging for rapid assessment of a newly hospitalized client

The case manager is demonstrating an understanding of the primary goals of managed care when: A) Arranging for the client to have a screening for prostate cancer B) Notifying the family that the client will require a wheelchair when discharged C) Providing the client with organizations that help defray the cost of prescribed drug D) Arranges for respite care when the client's family needs to attend an out of state affair

A) Arranging for the client to have a screening for prostate cancer

On a hospital inpatient unit, the nurse manager primary responsible is to: A) Assure the safety of both clients and staff on the unit B) Assisting the client to prepare a support system that will promote mental health at the time of discharge C) Providing medication therapy for the patients on the unit D) Helping the (inpatient) community remain supportive, safe, and healthy

A) Assure the safety of both clients and staff on the unit

A community mental health nurse is responsible for medication management for clients. Which general approach can be anticipated to yield the best results? A) Empowering clients to be responsible for medication compliance B) Providing frequent oversight by nurse of daily medication ingestion C) Establishing a system in which client support figures oversee medications D) Scheduling monthly blood tests to determine serum levels of prescribed medications

A) Empowering clients to be responsible for medication compliance

A novice nurse is assigned to manage the milieu and provide for the safety of the other clients when there is a behavioral crisis with a client in the dining room. The nurse shows a lack of understanding of crisis management when he/she: A) Insists that all clients go in their rooms B) Move clients from the dining room to the day room/solarium C) Encourage the clients to express their reactions to the incident D) Reassure clients that staff will handle the agitated client

A) Insists that all clients go in their rooms

What phrase best describes the DSM-IV-TR? A) Is a multiaxial psychiatric assessment system B) Is a compendium of treatment modalities C) Offers a complete list of nursing diagnoses D) Suggests common interventions for mental disorders.

A) Is a multiaxial psychiatric assessment system

Which is a characteristic of a therapeutic inpatient milieu? A) It provides for client safety and comfort. B) Clients are responsible for all decisions about privileges. C) Rules and behavioral limits are inconsistently enforced. D) Staff provide frequent and ongoing negative feedback to clients.

A) It provides for client safety and comfort.

Which statement best clarifies the difference between the art and science of nursing? A) The art is the care, compassion, and advocacy component, and the science is the applied knowledge base. B) The art is the way in which knowledge is applied, and the science is the technological aspects of caregiving. C) The art is the applied technology of practice and the science is the problem-solving and teaching aspects of caregiving. D) The art is the assessing and planning phases of the nursing process, and the science lies in implementing and evaluating.

A) The art is the care, compassion, and advocacy component, and the science is the applied knowledge base.

Which idea held by the nurse would best promote the provision of culturally competent care? A) Western biomedicine is one several established healing systems. B) Some individuals will profit from use of both Western and folk healing practices. C) Use of cultural translators will provide valuable information into health-seeking behaviors. D) Need for spiritual healing is a concept that crosses cultural boundaries.

A) Western biomedicine is one several established healing systems.

A community mental health nurse whose culture is Hispanic is assigned to a mobile mental health care unit in a locale where the majority culture is Vietnamese. A measure that will help the nurse plan and implement culturally sensitive care for clients is A) arranging for the services of an interpreter and cultural consultant. B) reading about the predominant health beliefs held by members of Asian cultures. C) to try to place self in the position of being an unassimilated member of a culture. D) treat clients in the same manner that clients of the Hispanic culture find satisfying.

A) arranging for the services of an interpreter and cultural consultant.

The premise underlying behavioral therapy is: A) behavior is learned and can be modified. B) behavior is a product of unconscious drives. C) motives must change before behavior changes. D) behavior is determined by cognitions. Change in cognitions produces new behavior.

A) behavior is learned and can be modified.

Current information suggests that the most disabling mental disorders are the result of: A) biological influences. B) psychological trauma. C) learned ways of behaving. D) faulty patterns of early nurturance.

A) biological influences.

When a tumor of the cerebellum is present, the nurse should expect that the client would initially demonstrate: A) disequilibrium. B) abnormal eye movement. C) impaired social judgment. D) blood pressure irregularities.

A) disequilibrium.

The nurse is working with a severely depressed client who has very low self-esteem and is distrustful of unit staff. The client is facing role transition from wife to wife and mother. The nurse's priority is to: A) establish trust with the client. B) teach the client effective mothering skills. C) identify positive traits the client possesses. D) teach about the client the importance of her medication.

A) establish trust with the client.

The major difference between the psychiatric nursing assessment performed for a client who is hospitalized and for a client who will be treated in the community is A) for the latter, the general assessment must be expanded. B) for the latter, the nursing focus is primarily on the mental status examination. C) for the former, the general assessment must be reimbursement based. D) for the former, the nursing focus is limited to client willingness to accept treatment.

A) for the latter, the general assessment must be expanded.

The community mental health nurse recognizes that the mental ill population will require: A) frequent assessment for physical illness both acute and chronic in mature B) continuous supervision in order to assure that their physical needs are met C) government assistance in the form of health insurance and housing D) legal assistance to assure they retain there civic rights

A) frequent assessment for physical illness both acute and chronic in mature

An important concept for nurses to remember when planning care for mentally ill clients is that each client: A) has areas of strength on which to build. B) has right that must be respected. C) comes with experiences that contribute to their problem. D) share the same fears as mentally healthy individuals.

A) has areas of strength on which to build.

A client is admitted to the behavioral health unit. Her husband comes to visit, and she tells the nurse she does not wish to see him. The nurse tells the client's husband that A) his wife has the right to refuse visitors if she wishes. B) he is welcome to visit but may receive a cool reception. C) the client's plan of care calls for limiting visitors for 2 days. D) the client is acutely psychotic and not responsible for her present behavior.

A) his wife has the right to refuse visitors if she wishes.

Maslow's theory of Humanistic Psychology has provided nursing with a framework for: A) holistic assessment. B) determining moral development. C) identifying potential for success in therapy. D) conducting nurse-client interpersonal interactions.

A) holistic assessment.

The psychiatric nurse planning and implementing care for culturally diverse clients should understand A) holistic theory. B) systems theory. C) adaptation theory. D) political power theory.

A) holistic theory

A 14-year-old who belongs to a neighborhood gang is found by her parents to lie and engage in sexually promiscuous behavior. They bring her to the mental health center. The nurse performing the assessment is told by the 14-year-old that she is happy, does well in school, and sees herself as popular and well regarded by her group. She states her parents are just old fashioned and don't understand her. The assessment the nurse will most likely make is that she: A) is displaying deviant behavior. B) cannot accurately appraise reality. C) is seriously and persistently mentally ill. D) should be considered for group home placement.

A) is displaying deviant behavior.

The nurse responsible for the care of a client prescribed clonazepam (klonopin) would evaluate treatment as being successful when the client demonstrates: A) less anxiety. B) normal appetite. C) improved sleep pattern. D) reduced auditory hallucinations.

A) less anxiety.

The nurse caring for a client taking risperidone (Risperidal) observes the client carefully for: A) napping during the day, a weight gain, and reports of dizziness. B) reports of falls, heartburn and nausea. C) a rapid heartbeat, red rash and hives. D) dry mouth, poor urinary output, and constipation

A) napping during the day, a weight gain, and reports of dizziness.

The basic functional unit of the nervous system is called a: A) neuron. B) synapse. C) receptor. D) neurotransmitter.

A) neuron.

Cells that respond to stimuli, conduct electrical impulses, and release neurotransmitters are called: A) neurons. B) synapses. C) dendrites. D) receptors.

A) neurons.

The incoherent thought and speech patterns of the client with schizophrenia are related to the brain's inability to: A) regulate conscious mental activity. B) retain and recall past experience. C) regulate social behavior. D) maintain homeostasis.

A) regulate conscious mental activity.

One implication of Freud's theory of the unconscious on psychiatric mental health nursing is related to the consideration that conscious and unconscious influences can help nurses better understand A) the root causes of client suffering. B) the client's immature behavior. C) the client's interpersonal interactions. D) the client's psychological ability to reason.

A) the root causes of client suffering.

A nurse on an adolescent psychiatric unit assesses a newly admitted 14-year-old. An impulse control disorder is suspected. Which aspects of the patient's history support the suspected diagnosis? Select all that apply. a. Family history of mental illness b. Allergies to multiple antibiotics c. Long history of severe facial acne d. Father with history of alcohol abuse e. History of an abusive relationship with one parent

A, D, E Parents who are abusive, rejecting, or overly controlling cause a child to suffer detrimental effects. Other stressors associated with impulse control disorders can include major disruptions such as placement in foster care, severe marital discord, or a separation of parents. Substance abuse by a parent is common. Acne and allergies are not aspects of the history that relate to the behavior.

A patient would benefit from therapy in which peers as well as staff have a voice in determining patients' privileges and psychoeducational topics. Which approach would be best? a. Milieu therapy b. Cognitive therapy c. Short-term dynamic therapy d. Systematic desensitization

ANS: A Milieu therapy is based on the idea that all members of the environment contribute to the planning and functioning of the setting. The distracters are individual therapies that do not fit the description.

-SOAP -DETERGENT -BATH OIL -SKIN CREAM -POWDER -ANTIPERSPIRANT

AGENTS COMMONLY USED ON THE SKIN

A patient says, "Please don't share information about me with the other people." How should the nurse respond? a. "I will not share information with your family or friends without your permission, but I share information about you with other staff." b. "A therapeutic relationship is just between the nurse and the patient. It is up to you to tell others what you want them to know." c. "It depends on what you choose to tell me. I will be glad to disclose at the end of each session what I will report to others." d. "I cannot tell anyone about you. It will be as though I am talking about my own problems, and we can help each other by keeping it between us."

ANS: A A patient has the right to know with whom the nurse will share information and that confidentiality will be protected. Although the relationship is primarily between the nurse and patient, other staff needs to know pertinent data. The other incorrect responses promote incomplete disclosure on the part of the patient, require daily renegotiation of an issue that should be resolved as the nurse-patient contract is established, and suggest mutual problem solving. The relationship must be patient centered. See relationship to audience response question.

A therapy group adds new members as others leave. What type of group is evident? a. Open b. Closed c. Homogeneous d. Heterogeneous

ANS: A An open group is a group that adds members throughout the life of the group as other members leave and as more persons who would benefit from the group become available. A closed group does not add new members; the membership is established at the beginning and, except for the occasional losses as some members leave, does not change thereafter. A homogeneous group includes members who are similar, and a heterogeneous group includes dissimilar members; not enough data are provided here to determine which applies in this case.

A patient says, "I've done a lot of cheating and manipulating in my relationships." Select a nonjudgmental response by the nurse. a. "How do you feel about that?" b. "I am glad that you realize this." c. "That's not a good way to behave." d. "Have you outgrown that type of behavior?"

ANS: A Asking a patient to reflect on feelings about his or her actions does not imply any judgment about those actions, and it encourages the patient to explore feelings and values. The remaining options offer negative judgments.

Which statement shows a nurse has empathy for a patient who made a suicide attempt? a. "You must have been very upset when you tried to hurt yourself." b. "It makes me sad to see you going through such a difficult experience." c. "If you tell me what is troubling you, I can help you solve your problems." d. "Suicide is a drastic solution to a problem that may not be such a serious matter."

ANS: A Empathy permits the nurse to see an event from the patient's perspective, understand the patient's feelings, and communicate this to the patient. The incorrect responses are nurse- centered (focusing on the nurse's feelings rather than the patient's), belittling, and sympathetic.

While talking with a patient diagnosed with major depression, a nurse notices the patient is unable to maintain eye contact. The patient's chin lowers to the chest, while the patient looks at the floor. Which aspect of communication has the nurse assessed? a. Nonverbal communication b. A message filter c. A cultural barrier d. Social skills

ANS: A Eye contact and body movements are considered nonverbal communication. There are insufficient data to determine the level of the patient's social skills or whether a cultural barrier exists.

A nurse and patient discuss a problem the patient has kept secret for many years. Afterward the patient says, "I feel so relieved that I finally told somebody." Which term best describes the patient's feeling? a. Catharsis b. Superego c. Cognitive distortion d. Counter-transference

ANS: A Freud initially used talk therapy, known as the cathartic method. Today we refer to catharsis as "getting things off our chests." The superego represents the moral component of personality.

A patient is having difficulty making a decision. The nurse has mixed feelings about whether to provide advice. Which principle usually applies? Giving advice: a. is rarely helpful. b. fosters independence. c. lifts the burden of personal decision making. d. helps the patient develop feelings of personal adequacy.

ANS: A Giving advice fosters dependence on the nurse and interferes with the patient's right to make personal decisions. It robs patients of the opportunity to weigh alternatives and develop problem-solving skills. Furthermore, it contributes to patient feelings of personal inadequacy. It also keeps the nurse in control and feeling powerful.

Inpatient hospitalization for persons with mental illness is generally reserved for patients who: a. present a clear danger to self or others. b. are noncompliant with medication at home. c. have limited support systems in the community. d. develop new symptoms during the course of an illness.

ANS: A Hospitalization is justified when the patient is a danger to self or others, has dangerously decompensated, or needs intensive medical treatment. The distracters do not necessarily describe patients who require inpatient treatment.

A community mental health nurse has worked for months to establish a relationship with a delusional, suspicious patient. The patient recently lost employment and could no longer afford prescribed medications. The patient says, "Only a traitor would make me go to the hospital." Select the nurse's best initial intervention. a. With the patient's consent, contact resources to provide medications without charge temporarily. b. Arrange a bed in a local homeless shelter with nightly on-site supervision. c. Hospitalize the patient until the symptoms have stabilized. d. Ask the patient, "Do you feel like I am a traitor?"

ANS: A Hospitalization may damage the nurse-patient relationship, even if it provides an opportunity for rapid stabilization. If medication is restarted, the patient may possibly be stabilized in the home setting, even if it takes a little longer. Programs are available to help patients who are unable to afford their medications. A homeless shelter is inappropriate and unnecessary. Hospitalization may be necessary later, but a less restrictive solution should be tried first, since the patient is not dangerous. A yes/no question is non-therapeutic communication.

A nurse provides care for an adolescent patient diagnosed with an eating disorder. Which behavior by this nurse indicates that additional clinical supervision is needed? a. The nurse interacts with the patient in a protective fashion. b. The nurse's comments to the patient are compassionate and nonjudgmental. c. The nurse teaches the patient to recognize signs of increasing anxiety and ways to intervene. d. The nurse refers the patient to a self-help group for individuals with eating disorders.

ANS: A In the effort to motivate the patient and take advantage of the decision to seek help and be healthier, the nurse must take care not to cross the line toward authoritarianism and assumption of a parental role. Protective behaviors are part of the parent's role. The helpful nurse uses a problem-solving approach and focuses on the patient's feelings of shame and low self-esteem. Referring a patient to a self-help group is an appropriate intervention.

Three months ago a patient diagnosed with binge eating disorder weighed 198 pounds. Lorcaserin (Belviq) was prescribed. Which current assessment finding indicates the need for reevaluation of this treatment approach? The patient: a. now weighs 196 pounds. b. says, "I am using contraceptives." c. says, "I feel full after eating a small meal." d. reports problems with dry mouth and constipation.

ANS: A Lorcaserin is designed to make people feel full after eating smaller meals by activating a serotonin 2c receptor in the brain and blocking appetite signals. According to the FDA, this drug should be stopped if a patient does not have 5% weight loss after 12 weeks of use. If the patient now weighs 196 pounds, the medication has not been effective. The distracters indicate patient learning was effective and expected side effects of this medication.

A nurse explains to the family of a mentally ill patient how a nurse-patient relationship differs from social relationships. Which is the best explanation? a. "The focus is on the patient. Problems are discussed by the nurse and patient, but solutions are implemented by the patient." b. "The focus shifts from nurse to patient as the relationship develops. Advice is given by both, and solutions are implemented." c. "The focus of the relationship is socialization. Mutual needs are met, and feelings are shared openly." d. "The focus is creation of a partnership in which each member is concerned with growth and satisfaction of the other."

ANS: A Only the correct response describes elements of a therapeutic relationship. The remaining responses describe events that occur in social or intimate relationships.

A 4-year-old grabs toys from siblings and says, "I want that now!" The siblings cry, and the child's parent becomes upset with the behavior. According to Freudian theory, this behavior is a product of impulses originating in which system of the personality? a. Id b. Ego c. Superego d. Preconscious

ANS: A The id operates on the pleasure principle, seeking immediate gratification of impulses. The ego acts as a mediator of behavior and weighs the consequences of the action, perhaps determining that taking the toy is not worth the mother's wrath. The superego would oppose the impulsive behavior as "not nice." The preconscious is a level of awareness. This item relates to an audience response question.

A patient has talked constantly throughout the group therapy session, often repeating the same comments. Other members were initially attentive then became bored, inattentive, and finally sullen. Which comment by the nurse leader would be most effective? a. Say to everyone, "Most of you have become quiet. I wonder if it might be related to concerns you may have about how the group is progressing today." b. Say to everyone, "One person has done most of the talking. I think it would be helpful for everyone to say how that has affected your experience of the group." c. Say to everyone, "I noticed that as our group progressed, most members became quiet, then disinterested, and now seem almost angry. What is going on?" d. Say to the talkative patient, "You have been doing most of the talking, and others have not had a chance to speak as a result. Could you please yield to others now?"

ANS: A The most effective action the nurse leader can take will be the one that encourages the group to solve its own problem. Pointing out changes in the group and asking members to respond to them lays the foundation for a discussion of group dynamics. Asking members to respond to the talkative patient puts that patient in an awkward position, likely increasing her anxiety. As anxiety increases, monopolizing behavior tends to increase as well, so this response would be self-defeating. Asking members what is going on is a broader opening and might lead to responses unrelated to the issue that bears addressing; narrowing the focus to the group process more directly addresses what is occurring in the group. Focusing on the talkative patient would be less effective and involves the leader addressing the issue instead of members first attempting to do so themselves (giving them a chance to practice skills such as assertive communication).

When a female Mexican American patient and a female nurse sit together, the patient often holds the nurse's hand. The patient also links arms with the nurse when they walk. The nurse is uncomfortable with this behavior. Which analysis is most accurate? a. The patient is accustomed to touch during conversation, as are members of many Hispanic subcultures. b. The patient understands that touch makes the nurse uncomfortable and controls the relationship based on that factor. c. The patient is afraid of being alone. When touching the nurse, the patient is reassured and comforted. d. The patient is trying to manipulate the nurse using nonverbal techniques.

ANS: A The most likely answer is that the patient's behavior is culturally influenced. Hispanic women frequently touch women they consider to be their friends. Although the other options are possible, they are less likely.

A nurse uses Maslow's hierarchy of needs to plan care for a patient with mental illness. Which problem will receive priority? The patient: a. refuses to eat or bathe. b. reports feelings of alienation from family. c. is reluctant to participate in unit social activities. d. is unaware of medication action and side effects.

ANS: A The need for food and hygiene are physiological and therefore take priority over psychological or meta-needs in care planning.

As a nurse discharges a patient, the patient gives the nurse a card of appreciation made in an arts and crafts group. What is the nurse's best action? a. Recognize the effectiveness of the relationship and patient's thoughtfulness. Accept the card. b. Inform the patient that accepting gifts violates policies of the facility. Decline the card. c. Acknowledge the patient's transition through the termination phase but decline the card. d. Accept the card and invite the patient to return to participate in other arts and crafts groups.

ANS: A The nurse must consider the meaning, timing, and value of the gift. In this instance, the nurse should accept the patient's expression of gratitude. See relationship to audience response question.

A nurse performed these actions while caring for patients in an inpatient psychiatric setting. Which action violated patients' rights? a. Prohibited a patient from using the telephone b. In patient's presence, opened a package mailed to patient c. Remained within arm's length of patient with homicidal ideation d. Permitted a patient with psychosis to refuse oral psychotropic medication

ANS: A The patient has a right to use the telephone. The patient should be protected against possible harm to self or others. Patients have rights to send and receive mail and be present during package inspection. Patients have rights to refuse treatment.

Select all that apply. A nurse can best address factors of critical importance to successful community treatment by including making assessments relative to: a. housing adequacy. b. family and support systems. c. income adequacy and stability. d. early psychosocial development. e. substance abuse history and current use.

ANS: A, B, C, E Early psychosocial developmental history is less relevant to successful outcomes in the community than the assessments listed in the other options. If a patient is homeless or fears homelessness, focusing on other treatment issues is impossible. Sufficient income for basic needs and medication is necessary. Adequate support is a requisite to community placement. Substance abuse undermines medication effectiveness and interferes with community adjustment.

Select all that apply. A patient states, "I'm starting cognitive-behavioral therapy. What can I expect from the sessions?" Which responses by the nurse would be appropriate? a. "The therapist will be active and questioning." b. "You will be given some homework assignments." c. "The therapist will ask you to describe your dreams." d. "The therapist will help you look at your ideas and beliefs about yourself." e. "The goal is to increase subjectivity about thoughts that govern your behavior."

ANS: A, B, D Cognitive therapists are active rather than passive during therapy sessions because they help patients reality-test their thinking. Homework assignments are given and completed outside the therapy sessions. Homework is usually discussed at the next therapy session. The goal of cognitive therapy is to assist the patient in identifying inaccurate cognitions and in reality- testing and formulating new, accurate cognitions. One distracter applies to psychoanalysis. Increasing subjectivity is not desirable.

A group has two more sessions before it ends. One member was previously vocal and has shown much progress but has now grown silent. What explanation most likely underlies this behavior? The silent member: a. has participated in the group and now has nothing more to offer. b. is having trouble dealing with feelings about termination of this group. c. wants to give quieter members a chance to talk in the remaining sessions. d. is engaging in attention-seeking behavior aimed at continuation of the group.

ANS: B A chief task during the termination phase of a group is to take what has been learned in group and transition to life without the group. The end of a group can be a significant loss for members, who may experience loss and grief and respond with sadness or anger. It is unlikely he would have nothing to say; at the very least, he could be responding to the comments of others even if not focusing on his own issues. He may wish to give quieter members a chance to talk, but again, this would not require or explain his complete silence. Some members, faced with only two remaining sessions, may be becoming more dominant under this pressure of time, but here too this is unlikely to lead a previously active participant to fall completely silent. The member is not attention-seeking.

A patient says, "I always feel good when I wear a size 2 petite." Which type of cognitive distortion is evident? a. Disqualifying the positive b. Overgeneralization c. Catastrophizing d. Personalization

ANS: B Automatic thoughts, or cognitive distortions, are irrational and lead to false assumptions and misinterpretations. The stem offers an example of overgeneralization. See related audience response question.

A nurse assesses a confused older adult. The nurse experiences sadness and reflects, "The patient is like one of my grandparents...so helpless." Which response is the nurse demonstrating? a. Transference b. Countertransference c. Catastrophic reaction d. Defensive coping reaction

ANS: B Countertransference is the nurse's transference or response to a patient that is based on the nurse's unconscious needs, conflicts, problems, or view of the world. See relationship to audience response question.

An advanced practice nurse observes a novice nurse expressing irritability regarding a patient with a long history of alcoholism and suspects the new nurse is experiencing countertransference. Which comment by the new nurse confirms this suspicion? a. "This patient continues to deny problems resulting from drinking." b. "My parents were alcoholics and often neglected our family." c. "The patient cannot identify any goals for improvement." d. "The patient said I have many traits like her mother."

ANS: B Countertransference occurs when the nurse unconsciously and inappropriately displaces onto the patient feelings and behaviors related to significant figures in the nurse's past. In this instance, the new nurse's irritability stems from relationships with parents. The distracters indicate transference or accurate analysis of the patient's behavior.

A patient diagnosed with schizophrenia tells the nurse, "The CIA is monitoring us through the fluorescent lights in this room. Be careful what you say." Which response by the nurse would be most therapeutic? a. "Let's talk about something other than the CIA." b. "It sounds like you're concerned about your privacy." c. "The CIA is prohibited from operating in health care facilities." d. "You have lost touch with reality, which is a symptom of your illness."

ANS: B It is important not to challenge the patient's beliefs, even if they are unrealistic. Challenging undermines the patient's trust in the nurse. The nurse should try to understand the underlying feelings or thoughts the patient's message conveys. The correct response uses the therapeutic technique of reflection. The other comments are non-therapeutic. Asking to talk about something other than the concern at hand is changing the subject. Saying that the CIA is prohibited from operating in health care facilities gives false reassurance. Stating that the patient has lost touch with reality is truthful, but uncompassionate.

A nurse receives these three phone calls regarding a newly admitted patient. • The psychiatrist wants to complete an initial assessment. • An internist wants to perform a physical examination. • The patient's attorney wants an appointment with the patient. The nurse schedules the activities for the patient. Which role has the nurse fulfilled? a. Advocate b. Case manager c. Milieu manager d. Provider of care

ANS: B Nurses on psychiatric units routinely coordinate patient services, serving as case managers as described in this scenario. The role of advocate would require the nurse to speak out on the patient's behalf. The role of milieu manager refers to maintaining a therapeutic environment. Provider of care refers to giving direct care to the patient.

During a therapy group that uses existential/Gestalt theory, patients shared feelings that occurred at the time of their admission. After a brief silence, one member says, "Several people have described feeling angry. I would like to hear from members who had other feelings." Which group role is evident by this comment? a. Energizer b. Encourager c. Compromiser d. Self-confessor

ANS: B The member is filling the role of encourager by acknowledging those who have contributed and encouraging input from others. An energizer encourages the group to make decisions or take an action. The compromiser focuses on reducing or resolving conflict to preserve harmony. A self-confessor verbalizes feelings or observations unrelated to the group.

A parent says, "My 2-year-old child refuses toilet training and shouts 'No!' when given directions. What do you think is wrong?" Select the nurse's best reply. a. "Your child needs firmer control. It is important to set limits now." b. "This is normal for your child's age. The child is striving for independence." c. "There may be developmental problems. Most children are toilet trained by age 2." d. "Some undesirable attitudes are developing. A child psychologist can help you develop a plan."

ANS: B This behavior is typical of a child around the age of 2 years, whose developmental task is to develop autonomy. The distracters indicate the child's behavior is abnormal.

A patient says to the nurse, "My father has been dead for over 10 years, but talking to you is almost as comforting as the talks he and I had when I was a child." Which term applies to the patient's comment? a. Superego b. Transference c. Reality testing d. Counter-transference

ANS: B Transference refers to feelings a patient has toward the health care workers that were originally held toward significant others in his or her life. Counter-transference refers to unconscious feelings that the health care worker has toward the patient. The superego represents the moral component of personality; it seeks perfection.

Select all that apply. A patient cries as the nurse explores the patient's feelings about the death of a close friend. The patient sobs, "I shouldn't be crying like this. It happened a long time ago." Which responses by the nurse facilitate communication? a. "Why do you think you are so upset?" b. "I can see that you feel sad about this situation." c. "The loss of a close friend is very painful for you." d. "Crying is a way of expressing the hurt you are experiencing." e. "Let's talk about something else because this subject is upsetting you."

ANS: B, C, D Reflecting ("I can see that you feel sad," "This is very painful for you") and giving information ("Crying is a way of expressing hurt") are therapeutic techniques. "Why" questions often imply criticism or seem intrusive or judgmental. They are difficult to answer. Changing the subject is a barrier to communication.

Which behavior shows that a nurse values autonomy? The nurse: a. suggests one-on-one supervision for a patient who has suicidal thoughts. b. informs a patient that the spouse will not be in during visiting hours. c. discusses options and helps the patient weigh the consequences. d. sets limits on a patient's romantic overtures toward the nurse.

ANS: C A high level of valuing is acting on one's belief. Autonomy is supported when the nurse helps a patient weigh alternatives and their consequences before the patient makes a decision. Autonomy or self-determination is not the issue in any of the other behaviors.

A patient discloses several concerns and associated feelings. If the nurse wants to seek clarification, which comment would be appropriate? a. "What are the common elements here?" b. "Tell me again about your experiences." c. "Am I correct in understanding that . . ." d. "Tell me everything from the beginning."

ANS: C Asking, "Am I correct in understanding that..." permits clarification to ensure that both the nurse and patient share mutual understanding of the communication. Asking about common elements encourages comparison rather than clarification. The remaining responses are implied questions that suggest the nurse was not listening.

A suspicious, socially isolated patient lives alone, eats one meal a day at a local shelter, and spends the remaining daily food allowance on cigarettes. Select a community psychiatric nurse's best initial action. a. Explore ways to help the patient stop smoking. b. Report the situation to the manager of the shelter. c. Assess the patient's weight; determine foods and amounts eaten. d. Arrange hospitalization for the patient in order to formulate a new treatment plan.

ANS: C Assessment of biopsychosocial needs and general ability to live in the community is called for before any other action is taken. Both nutritional status and income adequacy are critical assessment parameters. A patient may be able to maintain adequate nutrition while eating only one meal a day. The rule is to assess before taking action. Hospitalization may not be necessary. Smoking cessation strategies can be pursued later.

A patient tells the nurse, "I don't think I'll ever get out of here." Select the nurse's most therapeutic response. a. "Don't talk that way. Of course you will leave here!" b. "Keep up the good work, and you certainly will." c. "You don't think you're making progress?" d. "Everyone feels that way sometimes."

ANS: C By asking if the patient does not believe that progress has been made, the nurse is reflecting by putting into words what the patient is hinting. By making communication more explicit, issues are easier to identify and resolve. The remaining options are non-therapeutic techniques. Telling the patient not to "talk that way" is disapproving. Saying that everyone feels that way at times minimizes feelings. Telling the patient that good work will always result in success is falsely reassuring.

Which comment best indicates that a patient perceived the nurse was caring? "My nurse: a. always asks me which type of juice I want to help me swallow my medication." b. explained my treatment plan to me and asked for my ideas about how to make it better." c. spends time listening to me talk about my problems. That helps me feel like I am not alone." d. told me that if I take all the medicines the doctor prescribes, then I will get discharged sooner."

ANS: C Caring evidences empathetic understanding as well as competency. It helps change pain and suffering into a shared experience, creating a human connection that alleviates feelings of isolation. The distracters give examples of statements that demonstrate advocacy or giving advice.

What is the desirable outcome for the orientation stage of a nurse-patient relationship? The patient will demonstrate behaviors that indicate: a. self-responsibility and autonomy. b. a greater sense of independence. c. rapport and trust with the nurse. d. resolved transference.

ANS: C Development of rapport and trust is necessary before the relationship can progress to the working phase. Behaviors indicating a greater sense of independence, self-responsibility, and resolved transference occur in the working phase.

A nurse provides health teaching for a patient diagnosed with binge-purge bulimia. Priority information the nurse should provide relates to: a. self-monitoring of daily food and fluid intake. b. establishing the desired daily weight gain. c. how to recognize hypokalemia. d. self-esteem maintenance.

ANS: C Hypokalemia results from potassium loss associated with vomiting. Physiological integrity can be maintained if the patient can self-diagnose potassium deficiency and adjust the diet or seek medical assistance. Self-monitoring of daily food and fluid intake is not useful if the patient purges. Daily weight gain may not be desirable for a patient with bulimia nervosa. Self-esteem is an identifiable problem but is of lesser priority than the dangers associated with hypokalemia.

A patient says, "I'm still on restriction, but I want to attend some off-unit activities. Would you ask the doctor to change my privileges?" What is the nurse's best response? a. "Why are you asking me when you're able to speak for yourself?" b. "I will be glad to address it when I see your doctor later today." c. "That's a good topic for you to discuss with your doctor." d. "Do you think you can't speak to a doctor?"

ANS: C Nurses should encourage patients to work at their optimal level of functioning. A nurse does not act for the patient unless it is necessary. Acting for a patient increases feelings of helplessness and dependency.

A psychotherapist works with an anxious, dependent patient. Which strategy is most consistent with psychoanalytic psychotherapy? a. Identifying the patient's strengths and assets b. Praising the patient for describing feelings of isolation c. Focusing on feelings developed by the patient toward the therapist d. Providing psychoeducation and emphasizing medication adherence

ANS: C Positive or negative feelings of the patient toward the therapist indicate transference. Transference is a psychoanalytic concept that can be used to explore previously unresolved conflicts. The distracters relate to biological therapy and supportive psychotherapy. Use of psychoeducational materials is a common "homework" assignment used in cognitive therapy.

Which principle should guide the nurse in determining the extent of silence to use during patient interview sessions? a. A nurse is responsible for breaking silences. b. Patients withdraw if silences are prolonged. c. Silence can provide meaningful moments for reflection. d. Silence helps patients know that what they said was understood.

ANS: C Silence can be helpful to both participants by giving each an opportunity to contemplate what has transpired, weigh alternatives, and formulate ideas. A nurse breaking silences is not a principle related to silences. It is inaccurate to say that patients withdraw during long silences or that silence helps patients know that they are understood. Feedback helps patients know they have been understood.

After several therapeutic encounters with a patient who recently attempted suicide, which occurrence should cause the nurse to consider the possibility of countertransference? a. The patient's reactions toward the nurse seem realistic and appropriate. b. The patient states, "Talking to you feels like talking to my parents." c. The nurse feels unusually happy when the patient's mood begins to lift. d. The nurse develops a trusting relationship with the patient.

ANS: C Strong positive or negative reactions toward a patient or over-identification with the patient indicate possible countertransference. Nurses must carefully monitor their own feelings and reactions to detect countertransference and then seek supervision. Realistic and appropriate reactions from a patient toward a nurse are desirable. One incorrect response suggests transference. A trusting relationship with the patient is desirable. See relationship to audience response question.

A college student received an invitation to attend the wedding of a close friend who lives across the country. The student is afraid of flying. Which type of therapy would be most helpful for this patient? a. Psychoanalysis b. Milieu therapy c. Systematic desensitization d. Short-term dynamic therapy

ANS: C Systematic desensitization is a type of therapy aimed at extinguishing a specific behavior, such as the fear of flying. Psychoanalysis and short-term dynamic therapy seek to uncover conflicts. Milieu therapy involves environmental factors.

The unit secretary receives a phone call from the health insurer for a hospitalized patient. The caller seeks information about the patient's projected length of stay. How should the nurse instruct the unit secretary to handle the request? a. Obtain the information from the patient's medical record and relay it to the caller. b. Inform the caller that all information about patients is confidential. c. Refer the request for information to the patient's case manager. d. Refer the request to the health care provider.

ANS: C The case manager usually confers with insurers and provides the treatment team with information about available resources. The unit secretary should be mindful of patient confidentiality and should neither confirm that the patient is an inpatient nor disclose other information.

A group is in the working phase. One member says, "That is the stupidest thing I've ever heard. Everyone whines and tells everyone else what to do. This group is a total waste of my time." Which comment by the group leader would be most therapeutic? a. "You seem to think you know a lot already. Since you know so much, perhaps you can tell everyone why you are back in the hospital?" b. "I think you have made your views clear, but I wonder if others feel the same way. How does everyone else feel about our group?" c. "It must be hard to be so angry." Direct this comment to another group member, "You were also angry at first but not now. What has helped you?" d. "I would like to remind you that one of our group rules is that everyone is to offer only positive responses to the comments of others."

ANS: C The member's comments demean the group and its members and suggest that the member is very angry. Labeling the emotion and conveying empathy would be therapeutic. Focusing on members who are likely to be more positive can balance the influence of demoralizing members. "You seem to know a lot..." conveys hostility from the leader, who confronts and challenges the member to explain how he came to be readmitted if he was so knowledgeable, implying that he is less knowledgeable than he claims. This comment suggests countertransference and is non-therapeutic. Shifting away from the complaining member to see if others agree seeks to have others express disagreement with this member, but that might not happen. In the face of his anger, they might be quiet or afraid to oppose him, or they could respond in kind by expressing hostility themselves. A rule that only positive exchanges are permitted would suppress conflict, reducing the effectiveness of the therapy group.

A nurse at the well child clinic realizes that many parents have misconceptions about effective ways of disciplining their children. The nurse decides to form a group to address this problem. What should be the focus of the group? a. Support b. Socialization c. Health education d. Symptom management

ANS: C The nurse has diagnosed a knowledge deficit. The focus of the group should be education. Support and socialization are beneficial but should not be the primary focus of the group, and symptoms are not identified for intervention here.

The parent of a 4-year-old rewards and praises the child for helping a younger sibling, being polite, and using good manners. The nurse supports this use of praise related to these behaviors. These qualities are likely to be internalized and become part of which system of the personality? a. Id b. Ego c. Superego d. Preconscious

ANS: C The superego contains the "thou shalts," or moral standards internalized from interactions with significant others. Praise fosters internalization of desirable behaviors. The id is the center of basic instinctual drives, and the ego is the mediator. The ego is the problem-solving and reality-testing portion of the personality that negotiates solutions with the outside world. The preconscious is a level of awareness from which material can be retrieved easily with conscious effort. This item relates to an audience response question.

An adult says, "I never know the answers," and "My opinion doesn't count." Which psychosocial crisis was unsuccessfully resolved for this adult? a. Initiative versus guilt b. Trust versus mistrust c. Autonomy versus shame and doubt d. Generativity versus self-absorption

ANS: C These statements show severe self-doubt, indicating that the crisis of gaining control over the environment was not met successfully. Unsuccessful resolution of the crisis of initiative versus guilt results in feelings of guilt. Unsuccessful resolution of the crisis of trust versus mistrust results in poor interpersonal relationships and suspicion of others. Unsuccessful resolution of the crisis of generativity versus self-absorption results in self-absorption that limits the ability to grow as a person.

Documentation in a patient's chart shows, "Throughout a 5-minute interaction, patient fidgeted and tapped left foot, periodically covered face with hands, and looked under chair while stating, 'I enjoy spending time with you.'" Which analysis is most accurate? a. The patient is giving positive feedback about the nurse's communication techniques. b. The nurse is viewing the patient's behavior through a cultural filter. c. The patient's verbal and nonverbal messages are incongruent. d. The patient is demonstrating psychotic behaviors.

ANS: C When a verbal message is not reinforced with nonverbal behavior, the message is confusing and incongruent. Some clinicians call it a "mixed message." It is inaccurate to say that the patient is giving positive feedback about the nurse's communication techniques. The concept of a cultural filter is not relevant to the situation because a cultural filter determines what we will pay attention to and what we will ignore. Data are insufficient to draw the conclusion that the patient is demonstrating psychotic behaviors.

Select all that apply. Which comments by a nurse demonstrate use of therapeutic communication techniques? a. "Why do you think these events have happened to you?" b. "There are people with problems much worse than yours." c. "I'm glad you were able to tell me how you felt about your loss." d. "I noticed your hands trembling when you told me about your accident." e. "You look very nice today. I'm proud you took more time with your appearance."

ANS: C, D The correct responses demonstrate use of the therapeutic techniques making an observation and showing empathy. The incorrect responses demonstrate minimizing feelings, probing, and giving approval, which are non-therapeutic techniques.

Select all that apply. A patient diagnosed with anorexia nervosa is hospitalized for treatment. What features should the milieu provide? a. Flexible mealtimes b. Unscheduled weight checks c. Adherence to a selected menu d. Observation during and after meals e. Monitoring during bathroom trips f. Privileges correlated with emotional expression

ANS: C, D, E Priority milieu interventions support restoration of weight and normalization of eating patterns. This requires close supervision of the patient's eating and prevention of exercise, purging, and other activities. There is strict adherence to menus. Observe patients during and after meals to prevent throwing away food or purging. Monitor all trips to the bathroom. Mealtimes are structured, not flexible. Weighing is performed on a regular schedule. Privileges are correlated with weight gain and treatment plan compliance.

A nurse finds a patient diagnosed with anorexia nervosa vigorously exercising before gaining the agreed-upon weekly weight. Which response by the nurse is appropriate? a. "You and I will have to sit down and discuss this problem." b. "It bothers me to see you exercising. I am afraid you will lose more weight." c. "Let's discuss the relationship between exercise, weight loss, and the effects on your body." d. "According to our agreement, no exercising is permitted until you have gained a specific amount of weight."

ANS: D A matter-of-fact statement that the nurse's perceptions are different will help to avoid a power struggle. Treatment plans have specific goals for weight restoration. Exercise is limited to promote weight gain. Patients must be held accountable for required behaviors.

Which belief will best support a nurse's efforts to provide patient advocacy during a multidisciplinary patient care planning session? a. All mental illnesses are culturally determined. b. Schizophrenia and bipolar disorder are cross-cultural disorders. c. Symptoms of mental disorders are unchanged from culture to culture. d. Assessment findings in mental disorders reflect a person's cultural patterns.

ANS: D A nurse who understands that a patient's symptoms are influenced by culture will be able to advocate for the patient to a greater degree than a nurse who believes that culture is of little relevance. The distracters are untrue statements.

During a group therapy session, a newly admitted patient suddenly says to the nurse, "How old are you? You seem too young to be leading a group." Select the nurse's most appropriate response. a. "I am wondering what leads you to ask. Please tell me more." b. "I am old enough to be a nurse, which qualifies me to lead this group." c. "My age is not pertinent to why we are here and should not concern you." d. "You are wondering whether I have enough experience to lead this group?"

ANS: D A question such as this is common in the initial phase of group development when members are getting to know one another, dealing with trust issues, and testing the leader. Making explicit the implied serves to role model more effective communication and prompts further discussion of the patient's concern. Asking the patient to tell the leader more about the question focuses on the reason for the member's concern rather than on the issue raised (the experience and ability of the leader) and is a less helpful response. "I am old enough to be a nurse" and "age is not pertinent" are defensive responses and fail to address the patient's valid concern.

The nurse assigned to assertive community treatment (ACT) should explain the program's treatment goal as: a. assisting patients to maintain abstinence from alcohol and other substances of abuse. b. providing structure and a therapeutic milieu for mentally ill patients whose symptoms require stabilization. c. maintaining medications and stable psychiatric status for incarcerated inmates who have a history of mental illness. d. providing services for mentally ill individuals who require intensive treatment to continue to live in the community.

ANS: D An assertive community treatment (ACT) program provides intensive community services to persons with serious, persistent mental illness who live in the community but require aggressive services to prevent repeated hospitalizations.

Which patient would be most appropriate to refer for assertive community treatment (ACT)? A patient diagnosed with: a. a phobic fear of crowded places. b. a single episode of major depression. c. a catastrophic reaction to a tornado in the community. d. schizophrenia and four hospitalizations in the past year.

ANS: D Assertive community treatment (ACT) provides intensive case management for persons with serious persistent mental illness who live in the community. Repeated hospitalization is a frequent reason for this intervention. The distracters identify mental health problems of a more episodic nature.

Disturbed body image is a nursing diagnosis established for a patient diagnosed with an eating disorder. Which outcome indicator is most appropriate to monitor? a. Weight, muscle, and fat congruence with height, frame, age, and sex b. Calorie intake is within required parameters of treatment plan c. Weight reaches established normal range for the patient d. Patient expresses satisfaction with body appearance

ANS: D Body image disturbances are considered improved or resolved when the patient is consistently satisfied with his or her own appearance and body function. This is a subjective consideration. The other indicators are more objective but less related to the nursing diagnosis.

Clinical pathways are used in managed care settings to: a. stabilize aggressive patients. b. identify obstacles to effective care. c. relieve nurses of planning responsibilities. d. streamline the care process and reduce costs.

ANS: D Clinical pathways provide guidelines for assessments, interventions, treatments, and outcomes as well as a designated timeline for accomplishment. Deviations from the timeline must be reported and investigated. Clinical pathways streamline the care process and save money. Care pathways do not identify obstacles or stabilize aggressive patients. Staff are responsible for the necessary interventions. Care pathways do not relieve nurses of the responsibility of planning; pathways may, however, make the task easier.

A patient expresses a desire to be cared for by others and often behaves in a helpless fashion. Which stage of psychosexual development is most relevant to the patient's needs? a. Latency b. Phallic c. Anal d. Oral

ANS: D Fixation at the oral stage sometimes produces dependent infantile behaviors in adults. Latency fixations often result in difficulty identifying with others and developing social skills, resulting in a sense of inadequacy and inferiority. Phallic fixations result in having difficulty with authority figures and poor sexual identity. Anal fixation sometimes results in retentiveness, rigidity, messiness, destructiveness, and cruelty. This item relates to an audience response question.

A patient tells members of a therapy group, "I hear voices saying my doctor is poisoning me." Another patient replies, "I used to hear voices too. They sounded real, but I found out later they were not. The voices you hear are not real either." Which therapeutic factor is exemplified in this interchange? a. Catharsis b. Universality c. Imitative behavior d. Interpersonal learning

ANS: D Here a member gains insight into his own experiences from hearing about the experiences of others through interpersonal learning. Catharsis refers to a therapeutic discharge of emotions. Universality refers to members realizing their feelings are common to most people and not abnormal. Imitative behavior involves copying or borrowing the adaptive behavior of others.

Which remark by a group participant would the nurse expect during the working stage of group therapy? a. "My problems are very personal and private. How do I know people in this group will not tell others what you hear?" b. "I have enjoyed this group. It's hard to believe that a few weeks ago I couldn't even bring myself to talk here." c. "One thing everyone seems to have in common is that sometimes it's hard to be honest with those you love most." d. "I don't think I agree with your action. It might help you, but it seems like it would upset your family."

ANS: D In the working stage, members actively interact to help each other accomplish goals, and because trust has developed, conflict and disagreement can be expressed. Focusing on trust and confidentiality typically occur in the orientation phase as part of establishing group norms. Commonality and universality are also themes typically expressed in the orientation phase, whereas reflecting on progress is a task addressed in the termination phase.

A person says, "I was the only survivor in a small plane crash. Three business associates died. I got depressed and saw a counselor twice a week for 4 weeks. We talked about my feelings related to being a survivor, and I'm better now." Which type of therapy was used? a. Milieu therapy b. Psychoanalysis c. Behavior modification d. Interpersonal psychotherapy

ANS: D Interpersonal psychotherapy returned the patient to his former level of functioning by helping him come to terms with the loss of friends and guilt over being a survivor. Milieu therapy refers to environmental therapy. Psychoanalysis would call for a long period of exploration of unconscious material. Behavior modification would focus on changing a behavior rather than helping the patient understand what is going on in his life.

A Puerto Rican American patient uses dramatic body language when describing emotional discomfort. Which analysis most likely explains the patient's behavior? The patient: a. has a histrionic personality disorder. b. believes dramatic body language is sexually appealing. c. wishes to impress staff with the degree of emotional pain. d. belongs to a culture in which dramatic body language is the norm.

ANS: D Members of Hispanic American subcultures tend to use high affect and dramatic body language as they communicate. The other options are more remote possibilities.

The Diagnostic and Statistical Manual of Mental Disorders (DSM-5) classifies: a. deviant behaviors. b. present disability or distress. c. people with mental disorders. d. mental disorders people have.

ANS: D The DSM-5 classifies disorders people have rather than people themselves. The terminology of the tool reflects this distinction by referring to individuals with a disorder rather than as a "schizophrenic" or "alcoholic," for example. Deviant behavior is not generally considered a mental disorder. Present disability or distress is only one aspect of the diagnosis.

A health care provider prescribed depot injections every 3 weeks at the clinic for a patient with a history of medication noncompliance. For this plan to be successful, which factor will be of critical importance? a. The attitude of significant others toward the patient b. Nutrition services in the patient's neighborhood c. The level of trust between the patient and nurse d. The availability of transportation to the clinic

ANS: D The ability of the patient to get to the clinic is of paramount importance to the success of the plan. The depot medication relieves the patient of the necessity to take medication daily, but if he or she does not receive the injection at 3-week intervals, non-adherence will again be the issue. Attitude toward the patient, trusting relationships, and nutrition are important but not fundamental to this particular problem.

A Filipino American patient had a nursing diagnosis of situational low self-esteem related to poor social skills as evidenced by lack of eye contact. Interventions were used to raise the patient's self-esteem, but after 3 weeks, the patient's eye contact did not improve. What is the most accurate analysis of this scenario? a. The patient's eye contact should have been directly addressed by role-playing to increase comfort with eye contact. b. The nurse should not have independently embarked on assessment, diagnosis, and planning for this patient. c. The patient's poor eye contact is indicative of anger and hostility that were unaddressed. d. The nurse should have assessed the patient's culture before making this diagnosis and plan.

ANS: D The amount of eye contact a person engages in is often culturally determined. In some cultures, eye contact is considered insolent, whereas in others eye contact is expected. Asian Americans, including persons from the Philippines, often prefer not to engage in direct eye contact.

A school age child tells the school nurse, "Other kids call me mean names and will not sit with me at lunch. Nobody likes me." Select the nurse's most therapeutic response. a. "Just ignore them and they will leave you alone." b. "You should make friends with other children." c. "Call them names if they do that to you." d. "Tell me more about how you feel."

ANS: D The correct response uses exploring, a therapeutic technique. The distracters give advice, a non-therapeutic technique.

A nurse supports a parent for praising a child behaving in a helpful way. When this child behaves with politeness and helpfulness in adulthood, which feeling will most likely result? a. Guilt b. Anxiety c. Humility d. Self-esteem

ANS: D The individual will be living up to the ego ideal, which will result in positive feelings about self. The other options are incorrect because each represents a negative feeling.

A nurse caring for a withdrawn, suspicious patient recognizes development of feelings of anger toward the patient. The nurse should: a. suppress the angry feelings. b. express the anger openly and directly with the patient. c. tell the nurse manager to assign the patient to another nurse. d. discuss the anger with a clinician during a supervisory session.

ANS: D The nurse is accountable for the relationship. Objectivity is threatened by strong positive or negative feelings toward a patient. Supervision is necessary to work through countertransference feelings.

A nursing diagnosis for a patient diagnosed with bulimia nervosa is Ineffective coping related to feelings of loneliness as evidenced by overeating to comfort self, followed by self-induced vomiting. The best outcome related to this diagnosis is that within 2 weeks the patient will: a. appropriately express angry feelings. b. verbalize two positive things about self. c. verbalize the importance of eating a balanced diet. d. identify two alternative methods of coping with loneliness.

ANS: D The outcome of identifying alternative coping strategies is most directly related to the diagnosis of Ineffective coping. Verbalizing positive characteristics of self and verbalizing the importance of eating a balanced diet are outcomes that might be used for other nursing diagnoses. Appropriately expressing angry feelings is not measurable.

Which nursing diagnosis is more appropriate for a patient diagnosed with anorexia nervosa who restricts intake and is 20% below normal weight than for a 130-pound patient diagnosed with bulimia nervosa who purges? a. Powerlessness b. Ineffective coping c. Disturbed body image d. Imbalanced nutrition: less than body requirements

ANS: D The patient with bulimia nervosa usually maintains a close to normal weight, whereas the patient with anorexia nervosa may approach starvation. The incorrect options may be appropriate for patients with either anorexia nervosa or bulimia nervosa.

A patient being admitted to the eating disorders unit has a yellow cast to the skin and fine, downy hair over the trunk. The patient weighs 70 pounds; height is 5 feet 4 inches. The patient says, "I won't eat until I look thin." Select the priority initial nursing diagnosis. a. Anxiety related to fear of weight gain b. Disturbed body image related to weight loss c. Ineffective coping related to lack of conflict resolution skills d. Imbalanced nutrition: less than body requirements related to self-starvation

ANS: D The physical assessment shows cachexia, which indicates imbalanced nutrition. Addressing the patient's self-starvation is the priority.

Which comment best indicates a patient is self-actualized? a. "I have succeeded despite a world filled with evil." b. "I have a plan for my life. If I follow it, everything will be fine." c. "I'm successful because I work hard. No one has ever given me anything." d. "My favorite leisure is walking on the beach, hearing soft sounds of rolling waves."

ANS: D The self-actualized personality is associated with high productivity and enjoyment of life. Self-actualized persons experience pleasure in being alone and an ability to reflect on events.

A patient says to the nurse, "I dreamed I was stoned. When I woke up, I felt emotionally drained, as though I hadn't rested well." Which response should the nurse use to clarify the patient's comment? a. "It sounds as though you were uncomfortable with the content of your dream." b. "I understand what you're saying. Bad dreams leave me feeling tired, too." c. "So you feel as though you did not get enough quality sleep last night?" d. "Can you give me an example of what you mean by 'stoned'?"

ANS: D The technique of clarification is therapeutic and helps the nurse examine the meaning of the patient's statement. Asking for a definition of "stoned" directly asks for clarification. Restating that the patient is uncomfortable with the dream's content is parroting, a non-therapeutic technique. The other responses fail to clarify the meaning of the patient's comment.

A young female member in a therapy group says to an older female member, "You are just like my mother, always trying to control me with your observations and suggestions." Which therapeutic factor of a group is evident by this behavior? a. Instillation of hope b. Existential resolution c. Development of socializing techniques d. Corrective recapitulation of the primary family group

ANS: D The younger patient is demonstrating an emotional attachment to the older patient that mirrors patterns within her own family of origin, a phenomenon called corrective recapitulation of the primary family group. Feedback from the group then helps the member gain insight about this behavior and leads to more effective ways of relating to her family members. Instillation of hope involves conveying optimism and sharing progress. Existential resolution refers to the realization that certain existential experiences such as death are part of life, aiding the adjustment to such realities. Development of socializing techniques involves gaining social skills through the group's feedback and practice within the group.

Which remark by a patient indicates passage from orientation to the working phase of a nurse-patient relationship? a. "I don't have any problems." b. "It is so difficult for me to talk about problems." c. "I don't know how it will help to talk to you about my problems." d. "I want to find a way to deal with my anger without becoming violent."

ANS: D Thinking about a more constructive approach to dealing with anger indicates a readiness to make a behavioral change. Behavioral change is associated with the working phase of the relationship. Denial is often seen in the orientation phase. It is common early in the relationship, before rapport and trust are firmly established, for a patient to express difficulty in talking about problems. Stating skepticism about the effectiveness of the nurse-patient relationship is more typically a reaction during the orientation phase.

The nurse should refer which of the following patients to a partial hospitalization program? A patient who: a. has a therapeutic lithium level and reports regularly for blood tests and clinic follow-up. b. needs psychoeducation for relaxation therapy related to agoraphobia and panic episodes. c. spent yesterday in a supervised crisis care center and continues to have active suicidal ideation. d. states, "I'm not sure I can avoid using alcohol when my spouse goes to work every morning."

ANS: D This patient could profit from the structure and supervision provided by spending the day at the partial hospitalization program. During the evening, at night, and on weekends, the spouse could assume responsibility for supervision. A suicidal patient needs inpatient hospitalization. The other patients can be served in the community or with individual visits.

● Synthesis ● Comprehension ● Application ● Evaluation ● Analysis ● Knowledge

According to bloom's taxonomy of learning (bloom, 1956), learning occurs at 6 levels. Select all that apply

Research priorities for nursing informatics include the development of a standard nursing language and the development of database for clinical information.

According to which statement, nursing informatics is regarded as a specialist field of practice by the nurse?

Which symptom would not be assessed as a positive symptom of schizophrenia? A. Delusion of persecution B. Auditory hallucinations C. Affective flattening D. Idea of reference

Affective flattening

Negative signs and symptoms of schizophrenia

Affective flattening Alogia Avolition Apathy Anhedonia Attention deficits Inappropriate affect and bizarre affect These are the most devastating affects on clients ability to function.

The nurse is reviewing orders given for a patient with depression. Which order should the nurse question? 1. A low starting dose of a tricyclic antidepressant 2. An SSRI given initially with an MAOI 3. Electroconvulsive therapy to treat suicidal thoughts 4. Elavil to address the patient's agitation

An SSRI given initially with an MAOI

Bereavement overload

An accumulation of grief that occurs when an individual experiences many losses over a short period and is unable to resolve one before another is experienced. This phenomenon is common among the elderly.

What was the National Mental Health Act?

An act passed in 1946 that provided funds for the education of psychiatrists, psychologists, social workers, and psychiatric nurses

Dispositional crises

An acute response to an external situational stressor

crisis

An acute, time-limited state of disequilibrium resulting from situational, developmental, or societal sources of stress

Palliative care

An area of care that has evolved out of the hospice experience, but exists outside of hospice programs and is not restricted to the end of life. Palliative care is focused on the relief of physical, mental, and spiritual distress for individuals who have an incurable illness and is used earlier in the disease experience than hospice care. The goal of palliative care is to prevent and relieve suffering by early assessment and treatment of pain and other physical, psychosocial, and spiritual needs to improve the patient's quality of life.

BED MAKING

An art of arranging linens

A client asks the nurse about the doctors comment that he may have problems due to delayed synaptic transmission in his brain. The nurse explains that the best way to describe a synaptic transmission is which of the following?

An electrochemical process called neurotransmission

grandiosity, p. 367

An exaggerated appraisal of one's worth, importance, power, knowledge, or identity; may assume delusional proportions

CROSS-BODY EFFLEURAGE TECHNIQUE

An excellent massage technique for the lower back

positive symptoms, p. 334

An excess or distortion of normal functioning, or an aberrant response; typically refers to hallucinations, delusions, and disorganized thinking and behavior

Psychoneuroimmunology (PNI)

An exploration of relationships between the central nervous system, the immune system, behavior, and human biology

During a group session, the clients are asked to make one positive statement about their home life. The nurse notices that one of the clients begins to fidget in the chair and interprets this behavior as:

An expression of discomfort.

Healthcare surrogate

An individual selected to make medical decisions when a person is no longer able to make themselves.

Crazy p. 15

An informal, denigrating, and stigmatizing term for "mentally ill" that carries with it unfounded and negative implications

repetitive transcranial magnetic stimulation (rTMS), 732

An insulated coil of wire is placed on the client's head. Short pulses of an electrical current, sent through the insulate coil, generate a magnetic field that causes neurochemical changes in specifically targeted structures of the brain.

PLAQUE

An invisible soft film that adheres to the enamel surface of teeth which consists of bacteria, molecules of saliva

Minnesota Multiphasic Personality Inventory -2 (MMPI-2) p. 211

An objective psychological test providing data on various aspects of the client's personality which is scored or analyzed using empirically derived criteria. It consists of 567 items to which the test-taker responds with "true" or "false."

Technology literacy

As the tools of information technology become more pervasive we are subtly changing how knowledge is acquired and integrated. This literacy is the ability to use the tools of automation in the process of accessing, evaluating and using information.

Severe signs of Lithium Toxicity

Ataxia, confusion, large output of dilute urine, serious electroencephalographic changes, blurred vision, clonic movements, seizures, stupor, severe hypotension, coma; death is usually secondary to pulmonary complications.

Avoid alcohol with use of:

Avoid with :Benzodiazepines.Barbiturates.Antipsychotics Antidepressants.Other sedative type drugs

A 15-year-old was placed in a residential program after truancy, running away, and an arrest for theft. At the program, the adolescent refused to join in planned activities and pushed a staff member, causing a fall. Which approach by nursing staff will be most therapeutic? a. Planned ignoring c. Neutrally permit refusals b. Establish firm limits d. Coaxing to gain compliance

B Firm limits are necessary to ensure physical safety and emotional security. Limit setting will also protect other patients from the teen's thoughtless or aggressive behavior. Permitting refusals to participate in the treatment plan, ignoring, coaxing, and bargaining are strategies that do not help the patient learn to abide by rules or structure.

An 11-year-old diagnosed with oppositional defiant disorder becomes angry over the rules at a residential treatment program and begins shouting at the nurse. What is the nurse's initial action to defuse the situation? a. Say to the child, "Tell me how you're feeling right now." b. Take the child swimming at the program's pool. c. Establish a behavioral contract with the child. d. Administer an anxiolytic medication.

B Redirecting the expression of feelings into nondestructive, age-appropriate behaviors such as a physical activity helps the child learn how to modulate the expression of feelings and exert self-control. This is the least restrictive alternative and should be tried before resorting to measures that are more restrictive. A shouting child will not likely engage in a discussion about feelings. A behavioral contract could be considered later, but first the situation must be defused.

Which assessment question would produce data that would help a nurse understand healing options acceptable to a client of a different culture? A) "Do you have a special person in your community who usually cures your illness?" B) "What usually helps people who have the same type of illness you have?" C) "What questions would you like to ask about your condition?" D) "What sorts of stress are you presently experiencing?"

B) "What usually helps people who have the same type of illness you have?"

Nick, a college student, comes to the mental health clinic with symptoms of feeling blue and having occasional difficulty sleeping. He is able to manage his course work with lowered grades but states he just doesn't get as much pleasure from life as he did several months ago. The nurse making his global assessment of functioning would probably assign the rating of: A) 100. B) 70. C) 40. D) 10.

B) 70.

A client tells the mental health nurse "I am terribly frightened! I hear whispering that someone is going to kill me." Which criterion of mental health can the nurse assess as lacking? A) Control over behavior B) Appraisal of reality C) Effectiveness in work D) Healthy self-concept

B) Appraisal of reality

Which organs secrete hormones that are a normal component of the body's general response to stress? A) Brain, thyroid gland, pancreas B) Brain, pituitary gland, adrenal glands C) Pituitary gland, pancreas, thyroid gland D) Adrenal glands, parathyroid glands

B) Brain, pituitary gland, adrenal glands

A client is admitted to the hospital with severe depression. The nurse recognizes the possibility that depression may be related to a stress induced hormonal imbalance associated with: A) Luteinizing hormone B) Cortisol C) Gronadotropin D) Clomipramine

B) Cortisol

Which statement about diagnosis of a mental disorder is true? A) The symptoms of each disorder are common among all cultures. B) Culture may cause variations in symptoms for each clinical disorder. C) All mental disorders listed in the DSM-IV-TR are seen in all other cultures. D) Global assessment of functioning is more diagnostic than symptoms noted.

B) Culture may cause variations in symptoms for each clinical disorder.

An example of deinstitutionalization would include: A) Providing a recovering alcoholic with transportation to night AA meetings B) Discharging a stabilized psychotic client to a transitional half way house C) Psychiatric in-home visits for a new mother experiencing post partum depression D) Conducting regular depression screening at a local homeless shelter

B) Discharging a stabilized psychotic client to a transitional half way house

The Eastern worldview can be identified by the belief that: A) One's identify is found in individuality B) Holds responsibility to family as central C) Time waits for no one D) Disease is a lack of harmony with the environment

B) Holds responsibility to family as central

Which imaging technique can provide information about brain function? A) Computed tomography scan B) Positron emission tomography scan C) Magnetic resonance imaging scan D) Skull radiograph

B) Positron emission tomography scan

When preparing an education program focused on the history of the community mental health movement in the United States, the nurse includes the fact that: A) Use of community settings began after World War II with the discharge of large numbers of veterans with mental illness. B) The shift from care in psychiatric hospitals to community mental health centers to the community began in the 1960s. C) Movement of services out of hospitals and into the community is a baby-boomer phenomenon that began in the 1980s. D) The community mental health movement is an outgrowth of managed care's response to rising health care costs beginning in the 1990s.

B) The shift from care in psychiatric hospitals to community mental health centers to the community began in the 1960s.

The nurse uses Maslow's model of needs to direct the identification of an anxious client' priority intervention to be: A) assessing the client's success at fulfilling her appropriate developmental level tasks. B) assessing the client for her strengths upon which a nurse-client relationship can be based. C) planning one-on-one time with the client to assist in identify the fears behind her anxiety. D) evaluating the client's ability to learn and retain essential information regarding her condition.

B) assessing the client for her strengths upon which a nurse-client relationship can be based.

The greatest negative aspect for the psychiatric client regarding the multidisciplinary treatment team approach to care is that it: A) is an expensive treatment model B) can increase anxiety in the newly admitted client C) requires the client to answer the same questions repeatedly D) puts demands on the client's time

B) can increase anxiety in the newly admitted client

On the basis of the current understanding of neurotransmitters, the nurse can view a client's symptoms of profound depression as likely related in part to A) increased dopamine level. B) decreased serotonin level. C) increased norepinephrine level. D) decreased acetylcholine level.

B) decreased serotonin level.

Freud believed that individuals cope with anxiety by using: A) the superego. B) defense mechanisms. C) security operations. D) suppression

B) defense mechanisms.

The primary advantage of using a case manager is to: A) increase collaborative practice. B) enhance resource management. C) increase client satisfaction with care. D) promote evidence-based psychiatric nursing.

B) enhance resource management.

Data concerning client age, sex, education, and income should be the focus of an assessment in order to best understand cultural issues related to A) health practices. B) power and control. C) psychological stability. D) assimilation and conformity.

B) power and control.

Mental illnesses recognized across cultures include: A) antisocial and borderline personality disorders. B) schizophrenia and bipolar disorder. C) bulimia and anorexia nervosa. D) amok and social phobia

B) schizophrenia and bipolar disorder.

A nurse mentions "I work with a mobile mental health unit." The listener can assume the nurse A) works with patients who are incarcerated. B) sees clients in unconventional settings. C) is a preferred provider for a large HMO. D) is a clinical specialist with the visiting nurse service.

B) sees clients in unconventional settings.

Homeostasis is promoted by interaction between the brain and internal organs mediated by: A) conscious behavior. B) the autonomic nervous system. C) the sympathetic nervous system. D) the parasympathetic nervous system.

B) the autonomic nervous system.

The nurse explains to a depressed client who was abused physical as a child that his id is: A) the control over the emotional frustration he feels over the lose of his job. B) the source of his instincts to save himself from hurting himself. C) is not in place since he was abused after the age of 5 months. D) is able to differentiate his believed experiences and reality.

B) the source of his instincts to save himself from hurting himself.

What are the primary distinguishing factors between the behavior of persons diagnosed with oppositional defiant disorder (ODD) and those with conduct disorder (CD)? Select all that apply. The person diagnosed with: a. ODD relives traumatic events by acting them out. b. ODD tests limits and disobeys authority figures. c. ODD has difficulty separating from loved ones. d. CD uses stereotypical or repetitive language. e. CD often violates the rights of others.

B, E Persons diagnosed with ODD are negativistic, disobedient, and defiant toward authority figures without seriously violating the basic rights of others, whereas persons with conduct disorder frequently behave in ways that do violate the rights of others and age-appropriate societal norms. Reliving traumatic events occurs with posttraumatic stress disorder. Stereotypical language behaviors are seen in persons with autism spectrum disorders.

A client was admitted to the psychiatric unit on a suicide watch. In an effort to include family members in the clients care, the nurse must: Standard Text: Select all that apply.

Be prepared to deal with family members who may be confused, angry, or uninterested. Strive to remain neutral, and do not make assumptions about the familys behavior. Decide with the client which family members and friends are to be contacted and by whom.

It enables learners to demonstrate administration of an injection

Because ____, virtual reality simulation is the finest way to improve communication abilities.

self-reflective loop, p. 637

Becoming aware of the here-and-now events—what happened—and then reflecting back on them—why it happened

A week later, Mr. Jones has begun to take the conventional antipsychotic haloperidol. You approach him with his bedtime dose and notice that he is sitting very stiffly and immobile. When you approach, you notice that he is diaphoretic, and when you ask if he is okay he seems unable to turn towards you or to respond verbally. You also notice that his eyes are aimed sharply upward and he seems frightened. How should the nurse respond? Select all that apply. 1. Begin to wipe him with a washcloth wet with cold water or alcohol. 2. Hold his medication, stat page his doctor, and check his temperature. 3. Administer a medication such as benztropine IM to correct his dystonic reaction. 4. Reassure him that although there is no treatment for his tardive dyskinesia, it will pass. 5. Explain that he has anticholinergic toxicity, hold his meds, and give IM physostigmine. 6. Hold his medication tonight, and consult his doctor after completing medication rounds.

Begin to wipe him with a washcloth wet with cold water or alcohol. Hold his medication, stat page his doctor, and check his temperature.

enabling, p. 298

Behavior by another that allows the alcoholic or addicted person to continue maladaptive behavior without suffering the consequences (e.g., calling the workplace to say that the alcoholic has the flu, when in reality the alcoholic has a hangover)

chronic self-destructive behavior

Behavior that harms the self, is habitual, and generally poses a low level of lethality; may include smoking, gambling, using drugs, self-mutilation

aggression

Behavior that violates the rights of other individuals, results in harm to others.

It is a framework that focuses on learned behavior a) Behavioral framework b) Personality framework c) Psychodynamic framework

Behavioral Framework

Values

Beliefs of a person or social group in which they have an emotional investment (either for or against something)

Lorazepam (ativan) or oxapam (serax) - more short acting, preferred drugs

Benzo

Most commonly used for anxiety because: More effective. Less likely to interact with other drugs. Less likely to cause overdose. Less potential for abuse. Account for 75% of 100 million annual prescriptions

Benzodiazepines

Not recommended during first trimester due to possible increase incidence in birth defects Mothers who are breastfeeding - should not receive benzodiazepines regularly as they cross easily into breast milk and affect infant

Benzodiazepines

period in which a drinker functions socially but for which they have no memory

Blackouts

Serotonin Norepinephrine Reuptake Inhibitors (SNRIs)

Block the reuptake of serotonin and norepinephrine; often used as 2nd line treatment if pt does not respond to SSRIs; similar side effect profile as SSRIs; need to monitor BP and HR

An unconscious client is admitted to the emergency department. The admitting diagnosis is "rule out opiate overdose." Which item of assessment data would be most consistent with opiate overdose? A. Blood pressure, 80/40 mm Hg; pulse, 120 beats/min; respirations, 10 breaths/min B. Blood pressure, 120/80 mm Hg; pulse, 84 beats/min; respirations, 20 breaths/min C. Blood pressure, 140/90 mm Hg; pulse, 76 beats/min; respirations, 24 breaths/min D. Blood pressure, 180/100 mm Hg; pulse, 72 beats/min; respirations, 28 breaths/min

Blood pressure, 80/40 mm Hg; pulse, 120 beats/min; respirations, 10 breaths/min

4 humors

Blood, black bile, yellow bile, and phlegm

Initial symptoms of lithium toxicity

Blurred vision, ataxia, tinnitus, persistent N/V, severe diarrhea

An adolescent diagnosed with a conduct disorder stole and wrecked a neighbor's motorcycle. Afterward, the adolescent was confronted about the behavior but expressed no remorse. Which variation in the central nervous system best explains the adolescent's reaction? a. Serotonin dysregulation and increased testosterone activity impair one's capacity for remorse. b. Increased neuron destruction in the hippocampus results in decreased abilities to conform to social rules. c. Reduced gray matter in the cortex and dysfunction of the amygdala results in decreased feelings of empathy. d. Disturbances in the occipital lobe reduce sensations that help an individual clearly visualize the consequences of behavior.

C Adolescents with conduct disorder have been found to have significantly reduced gray matter bilaterally in the anterior insulate cortex and the amygdala. This reduction may be related to aggressive behavior and deficits of empathy. The less gray matter in these regions of the brain, the less likely adolescents are to feel remorse for their actions or victims. People with intermittent explosive disorder may have differences in serotonin regulation in the brain and higher levels of testosterone. Neuron destruction in the hippocampus is associated with memory deficits. The occipital lobe is involved with visual stimuli but not the processing of emotions.

The psychiatric mental health nurse working with depressed clients of the Eastern culture must realize that a useful outcome criterion might be if client reports A) increased somatic expressions of distress. B) disruption of energy balance. C) appeasement of spirits. D) increased anxiety.

C) appeasement of spirits.

When group therapy is prescribed as a treatment modality, the nurse would suggest placement of a 9-year-old in a group that uses: a. guided imagery. b. talk focused on a specific issue. c. play and talk about a play activity. d. group discussion about selected topics.

C Group therapy for young children takes the form of play. For elementary school children, therapy combines play and talk about the activity. For adolescents, group therapy involves more talking.

Which factor presents the highest risk for a child to develop a psychiatric disorder? a. Having an uncle with schizophrenia c. Living with an alcoholic parent b. Being the oldest child in a family d. Being an only child

C Having a parent with a substance abuse problem has been designated an adverse psychosocial condition that increases the risk of a child developing a psychiatric condition. Being in a middle-income family and being the oldest child do not represent psychosocial adversity. Having a family history of schizophrenia presents a risk, but an alcoholic parent in the family offers a greater risk.

Which behavior indicates that the treatment plan for a child diagnosed with an autism spectrum disorder was effective? The child: a. plays with one toy for 30 minutes. b. repeats words spoken by a parent. c. holds the parent's hand while walking. d. spins around and claps hands while walking.

C Holding the hand of another person suggests relatedness. Usually, a child with an autism spectrum disorder would resist holding someone's hand and stand or walk alone, perhaps flapping arms or moving in a stereotyped pattern. The incorrect options reflect behaviors that are consistent with autism spectrum disorders.

An 11-year-old diagnosed with oppositional defiant disorder becomes angry over the rules at a residential treatment program and begins cursing at the nurse. Select the best method for the nurse to defuse the situation. a. Ignore the child's behavior. b. Send the child to time-out. c. Accompany the child to the gym and shoot baskets. d. Role-play a more appropriate behavior with the child.

C The child's behavior warrants an active response. Redirecting the expression of feelings into nondestructive age-appropriate behaviors, such as a physical activity, helps defuse the situation here and now. This response helps the child learn how to modulate the expression of feelings and exert self-control. This is the least restrictive alternative and should be tried before resorting to a more restrictive measure. Role-playing is appropriate after the child's anger is defused.

In an attempt to provide both safety and client comfort, psychiatric units generally have: A) a varied client menu served on non-breakable plastic dinnerware B) comfortable seating that is well padded and secured to the floor C) bedrooms that resemble hotel rooms but with specific safety features D) a dayroom with audio visual equipment and visible from the nurses' station

C) bedrooms that resemble hotel rooms but with specific safety features

A child diagnosed with attention deficit hyperactivity disorder (ADHD) shows hyperactivity, aggression, and impaired play. The health care provider prescribed amphetamine salts (Adderall). The nurse should monitor for which desired behavior? a. Increased expressiveness in communication with others b. Abilities to identify anxiety and implement self-control strategies c. Improved abilities to participate in cooperative play with other children d. Tolerates social interactions for short periods without disruption or frustration

C The goal is improvement in the child's hyperactivity, aggression, and play. The remaining options are more relevant for a child with intellectual development disorder or an anxiety disorder.

An adolescent was recently diagnosed with oppositional defiant disorder. The parents say to the nurse, "Isn't there some medication that will help with this problem?" Select the nurse's best response. a. "There are no medications to treat this problem. This diagnosis is behavioral in nature." b. "It's a common misconception that there is a medication available to treat every health problem." c. "Medication is usually not prescribed for this problem. Let's discuss some behavioral strategies you can use." d. "There are many medications that will help your child manage aggression and destructiveness. The health care provider will discuss them with you."

C The parents are seeking a quick solution. Medications are generally not indicated for oppositional defiant disorder. Comorbid conditions that increase defiant symptoms, such as attention deficit hyperactivity disorder, should be managed with medication, but no comorbid problem is identified in the question. The nurse should give information on helpful strategies to manage the adolescent's behavior.

An adolescent was arrested for prostitution and assault on a parent. The adolescent says, "I hate my parents. They focus all attention on my brother, who's perfect in their eyes." Which nursing diagnosis is most applicable? a. Disturbed personal identity related to acting out as evidenced by prostitution b. Hopelessness related to achievement of role identity as evidenced by feeling unloved by parents c. Ineffective coping related to inappropriate methods of seeking parental attention as evidenced by acting out d. Impaired parenting related to inequitable feelings toward children as evidenced by showing preference for one child over another

C The patient demonstrates a failure to follow age-appropriate social norms and an inability to problem solve by using adaptive behaviors to meet life's demands and roles. The defining characteristics are not present for the other nursing diagnoses. The patient never mentioned hopelessness or disturbed personal identity. The problem relates to the patient's perceptions of parental behavior rather than the actual behavior.

The parent of a child diagnosed with Tourette's disorder says to the nurse, "I think my child is faking the tics because they come and go." Which response by the nurse is accurate? a. "Perhaps your child was misdiagnosed." b. "Your observation indicates the medication is effective." c. "Tics often change frequency or severity. That doesn't mean they aren't real." d. "This finding is unexpected. How have you been administering your child's medication?"

C Tics are sudden, rapid, involuntary, repetitive movements or vocalizations characteristic of Tourette's disorder. They often fluctuate in frequency, severity, and are reduced or absent during sleep.

A nurse works with a child who is sad and irritable because the child's parents are divorcing. Why is establishing a therapeutic alliance with this child a priority? a. Therapeutic relationships provide an outlet for tension. b. Focusing on the strengths increases a person's self-esteem. c. Acceptance and trust convey feelings of security to the child. d. The child should express feelings rather than internalize them.

C Trust is frequently an issue because the child may question their trusting relationship with the parents. In this situation, the trust the child once had in parents has been disrupted, reducing feelings of security. The correct answer is the most global response.

When the nurse cares for a client taking an antipsychotic medication that blocks muscarinic receptors, the nurse would assess for: A) sedation, drowsiness, hypotension, and weight gain. B) orthostatic hypotension and memory dysfunction. C) blurred vision, dry mouth, and constipation. D) tremors, tachycardia, and ejaculatory dysfunction.

C) blurred vision, dry mouth, and constipation.

A suspicious client smokes several packs of cigarettes daily and drinks as much coffee or soda as he is able to afford. The nurse finds that every nursing intervention is met with sarcasm. When she asks for advice from a peer, the most helpful response would be A) "You are dealing with a very difficult and resistant client just stick to your plan." B) "If you haven't been able to establish client trust ask for a change of assignment." C) "It might help if you remember that sarcasm represents oral-stage fixation." D) "You are attempting to work with a client who likes to keep others off balance."

C) "It might help if you remember that sarcasm represents oral-stage fixation."

The Advanced Practice Psychiatric Nurse is asked to explain what a psychosocial rehabilitation program is. The nurse responds best when sharing: A) "The concept started in the 1960's and provides psychiatric care for those without private insurance." B) "The individual program can provide medication, therapies and social services for the mentally ill." C) "Referrals from inpatient and intensive out patient facilities are provided with long termcare that focuses on desired goals." D) "It is a multidisciplinary team approach that is composed of nurses, psychiatrics, social workers and psychologists."

C) "Referrals from inpatient and intensive out patient facilities are provided with long termcare that focuses on desired goals."

A nurse's identification badge includes the wording, 'Psychiatric Mental Health Nurse'. A client with a history of paranoia asks, "What does that title mean?" The nurse responds best when answering: A) "Don't be afraid, it means I'm here to help not hurt you." B) "Psychiatric Mental Health nurses care for people with mental illnesses." C) "We have the specialized skills needed to care for those with mental illnesses." D) "The nurses who work in mental health facilities have that title."

C) "We have the specialized skills needed to care for those with mental illnesses."

A client has begun to neglect her appearance, is withdrawn and stays in her room. Her mother hears her seemingly talking to others, but no one is in the room with her. Last night she threw a chair and broke the window of her bedroom. She tells the nurse nothing is wrong. The nurse rating her current global assessment of functioning would probably assign the code A) 100. B) 70. C) 40. D) 0.

C) 40.

An important difference between the developmental theories of Freud and Erikson is: A) Freud considers the entire lifespan from birth to old age. B) Freud focuses to a greater extent on cognitive development. C) Erikson viewed individual growth in terms of social setting. D) Erikson focuses on development of individual moral thinking.

C) Erikson viewed individual growth in terms of social setting.

Which information will be of greater importance to planning care for a client receiving mental health care in his home than to planning care for a client receiving services on an inpatient unit? A) Mental status examination B) Strengths and deficits of client C) Housing adequacy and stability D) Presenting problem and referring party

C) Housing adequacy and stability

In 1963 the Community Mental Health Centers Act was signed with the primary purpose of: A) Having mental health care services funded by the federal government while provided by state government B) Providing mental health care to the uninsured in an economical reasonable manner C) Providing mental health care on an outpatient basis in order to maintain the client as a member of an established community D) Assuring all Americans the mental health care services they need and/or desire

C) Providing mental health care on an outpatient basis in order to maintain the client as a member of an established community

The psychiatric community health nurse engages in primary prevention when: A) Visiting a homeless shelter to provide crisis intervention for its clients B) Discussing the need for proper nutrition with a depressed new mother C) Providing stress reduction seminars at the local senior center D) Visits the home of a client currently displaying manic behavior

C) Providing stress reduction seminars at the local senior center

Which source of healing might be most satisfactory to a client who believes his illness is caused by spiritual forces? A) Acupuncture B) Dietary change C) Religious ritual D) Herbal medicine

C) Religious ritual

Which criterion is necessary to refer a client to a partial hospitalization program? A) The client is hospitalized at night in an inpatient setting. B) The client must be able to provide his or her own transportation daily. C) The client is able to return home each day. D) The client is able to care for his or her own physical and psychological needs.

C) The client is able to return home each day.

The nurse planning care for a 14-year-old needs to take into account that the developmental task of adolescence is to: A) establish trust. B) gain autonomy. C) achieve identity. D) develop a sense of industry.

C) achieve identity.

According to Freud, the nurse recognizes that a client experiencing dysfunction of the conscious as the part of the mind will have problems with: A) only recent memory. B) recent and long-term memory. C) all material that the person is aware of at any one time. D) only material that should be easily retrieved.

C) all material that the person is aware of at any one time.

Sullivan viewed anxiety as: A) emotional experience felt after the age of 5 years. B) a sign of guilt in adults. C) any painful feeling or emotion arising from social insecurity. D) adults trying to go beyond experiences of guilt and pain.

C) any painful feeling or emotion arising from social insecurity.

A client with prolonged mental illness was readmitted when symptoms exacerbated after she stopped taking prescribed medication. The client lives alone and has few outside activities. To best prepare the client for discharge the nurse will focus on: A) improving client-family relationships. B) placing the client in a sheltered workshop. C) psychoeducation to promote medication compliance. D) involving the client in daily visits to a psychosocial club.

C) psychoeducation to promote medication compliance.

The behavior of an individual who seems unable to learn right from wrong and who repeatedly violates norms and laws demonstrates problems related to the brain's inability to A) regulate conscious mental activity. B) retain and recall past experience. C) regulate social behavior. D) maintain homeostasis.

C) regulate social behavior.

The client says to the nurse, Its my right to refuse medications. Which statement best reflects the nurses ability to create a mutual understanding?

Can you tell me what concerns you have about medications?

ATHLETE'S FOOT, TINEA PEDIS (ringworm of the foot)

Caused by fungus. The symptoms are scaling and cracking of the skin, particularly between the toes.

Binge Eating Disorder

Characterized by binge eating, but they don't compensate like bulimia

SELF-HELP BED BATH

Clients that are confined are able to bathe themselves with help from nurse for washing the back and perhaps the feet.

Premenstrual dysphoric disorder

Condition associated with severe emotional and physical problems that are closely linked to the menstrual cycle **symptoms begin week before menses and subside after onset of menstruation**

dissociative identity disorder (DID)

Condition characterized by the presence of two or more distinct identities or personality states

Due to a staff members absence, the nurse is reviewing staff assignments for the day. Which task can the nurse delegate to the psychosocial rehabilitation worker?

Conflict resolution teaching to a small group of clients

malingering

Conscious, deliberate feigning of an illness used to avoid a painful situation or for personal gain

Wechsler Adult Intelligence Scale-IV p. 212

Consists of a number of subtests for the derivation of intelligence quotient (IQ) scores.

manipulation

Controlling behavior; the manipulative individual uses and exploits others for personal gain

A nurse assesses a 3-year-old diagnosed with an autism spectrum disorder. Which finding is most associated with the child's disorder? The child: a. has occasional toileting accidents. b. is unable to read children's books. c. cries when separated from a parent. d. continuously rocks in place for 30 minutes.

D Autism spectrum disorder involves distortions in development of social skills and language that include perception, motor movement, attention, and reality testing. Body rocking for extended periods suggests autism spectrum disorder. The distracters are expected findings for a 3-year-old.

A 15-year-old ran away from home six times and was arrested for shoplifting. The parents told the court, "We can't manage our teenager." The adolescent is physically abusive to the mother and defiant with the father. Which diagnosis is supported by this adolescent's behavior? a. Attention deficit hyperactivity disorder (ADHD) b. Posttraumatic stress disorder (PTSD) c. Intermittent explosive disorder d. Conduct disorder

D Conduct disorders are manifested by a persistent pattern of behavior in which the rights of others and age-appropriate societal norms are violated. Intermittent explosive disorder is a pattern of behavioral outbursts characterized by an inability to control aggressive impulses in adults 18 years and older. Criteria for ADHD and PTSD are not met in the scenario.

The family of a child diagnosed with an impulse control disorder needs help to function more adaptively. Which aspect of the child's plan of care will be provided by an advanced practice nurse rather than a staff nurse? a. Leading an activity group c. Formulating nursing diagnoses b. Providing positive feedback d. Dialectical behavioral therapy (DBT)

D The advanced practice nurse role includes individual, group, and family psychotherapist; educator of nurses, other professions, and the community; clinical supervisor; consultant to professional and nonprofessional groups; and researcher. Dialectical behavioral therapy (DBT) is an aspect of psychotherapy. The distracters describe actions of a nurse generalist.

A 12-year-old has engaged in bullying for several years. The parents say, "We can't believe anything our child says." Recently this child shot a dog with a pellet gun and set fire to a neighbor's trash bin. The child's behaviors support the diagnosis of: a. attention deficit hyperactivity disorder. b. intermittent explosive disorder. c. defiance of authority. d. conduct disorder.

D The behaviors mentioned are most consistent with criteria for conduct disorder, for example, aggression against people and animals; destruction of property; deceitfulness; rule violations; and impairment in social, academic, or occupational functioning. Intermittent explosive disorder is a pattern of behavioral outbursts characterized by an inability to control aggressive impulses in adults 18 years and older. The behaviors are not consistent with attention deficit and are more pervasive than defiance of authority.

A 4-year-old cries for 5 minutes when the parents leave the child at preschool. The parents ask the nurse, "What should we do?" Select the nurse's best response. a. "Ask the teacher to let the child call you at play time." b. "Withdraw the child from preschool until maturity increases." c. "Remain with your child for the first hour of preschool time." d. "Give your child a kiss before you leave the preschool program."

D The child demonstrates age-appropriate behavior for a 4-year-old. The nurse should reassure the parents. The distracters are over-reactions.

A second-generation Chinese-American client reports the inability to concentrate and dysphoric mood. During the assessment interview the nurse also learns that the client is responsible for his elderly parents in addition to his wife and two children. The question that would give data of least value to the assessment of family dynamics is A) "What changes have occurred recently at work?" B) "Are your wife and children conforming to your expectations?" C) "Are you experiencing stress associated with conforming to family expectations?" D) "Do you expect others to shun or avoid you because you are seeing a therapist

D) "Do you expect others to shun or avoid you because you are seeing a therapist

As a result of Harry Stack Sullivan, the nurse in virtually all residential and day hospital settings is involved in providing clients with: A) Security operations B) Psychoanalysis C) Analysis of behavior patterns D) A psychotherapeutic environment

D) A psychotherapeutic environment

While mental illness still carries a stigma, acceptance has improved over the last 40 years in part due to: A) Better control of symptoms through drug therapy B) Public screenings that are well advertized in the community C) Community psychiatric nursing programs that provide in home care D) Acknowledgement of personal mental health issues by well known people

D) Acknowledgement of personal mental health issues by well known people

Which client problem would be most suited to the use of interpersonal therapy? A) Disturbed sensory perception B) Impaired social interaction C) Medication noncompliance D) Dysfunctional grieving

D) Dysfunctional grieving

Which client displays a symptom that gives priority for admission to an inpatient behavioral health unit? A) Ms. E, who has severe anxiety and feels as though she is suffocating. B) Ms. F, who is grieving over the recent death of her identical twin sister. C) Ms. G, who hears voices telling her she is "exalted queen of the universe." D) Mr. H, who had a seizure related to his elevated lithium level.

D) Mr. H, who had a seizure related to his elevated lithium level

Which would not be considered a crisis on a psychiatric unit? A) Mr. R goes into shock. B) Ms. T goes into cardiopulmonary arrest. C) Mr. S demonstrates anger that escalates to physical assault. D) Mrs. U reports hearing voices telling her to hit others

D) Mrs. U reports hearing voices telling her to hit others

In which situation is the nurse functioning in the role of advocate? A) Nurse A provides one-to-one supervision for a client on suicide precautions. B) Nurse B co-leads a medication education group for clients and families. C) Nurse C attends an inservice education program to obtain recertification in cardiopulmonary resuscitation. D) Nurse D negotiates with the client's HMO for extension of a 3-day hospitalization to 5 days.

D) Nurse D negotiates with the client's HMO for extension of a 3-day hospitalization to 5 days.

A man frequently bursts out by loudly singing operatic arias. The neighbors in the next apartment find the noise disturbing. They go to his apartment to confront him and find that in he is wearing only his bathrobe and his apartment is messy. He acts outraged and tells them he must sing several hours daily and will not promise to be quieter. The conclusion that can be drawn is: A) The man is demonstrating symptoms of bipolar disorder. B) The man is socially deviant. C) The man is egocentric. D) The man may or may not be mentally ill.

D) The man may or may not be mentally ill.

A function shared by advanced practice and general practice psychiatric nurses is A) prescriptive authority. B) hospital privileges. C) consultation services provided. D) collaboration with a multidisciplinary team.

D) collaboration with a multidisciplinary team.

A client comes to the clinic asking for help because his third fiancée has broken their engagement. He states "I don't know what is wrong with me, but my friends tell me I am too possessive." The type of therapy that might address his interpersonal deficit is A) psychoanalysis. B) cognitive therapy. C) behavioral therapy. D) interpersonal psychotherapy.

D) interpersonal psychotherapy.

While caring for a client who is very ill with a mental disorder, the nurse wonders if the client has always been so regressed or if he has functioned at a higher level in the recent past. The best way to obtain this information would be to: A) ask the client. B) ask the family. C) refer to the progress notes. D) look at axis V of the client's DSM-IV-TR sheet.

D) look at axis V of the client's DSM-IV-TR sheet.

A client is being discharged after a short hospitalization. He will receive continuing care from a community mental health nurse who will visit his home. Unlike in the hospital where the goal is the stabilization of the client and driven by the psychiatric staff, when in the community setting the goals are: A) client directed and focused on the individual's expressed wishes B) society driven and focused on returning the client into the social community C) determined by the care provider and focused on cost effective measures D) negotiated by both client and staff and focused on long term functioning

D) negotiated by both client and staff and focused on long term functioning

A client receiving a psychotropic drug reports to the nurse that he is drowsy all the time and is having difficulty focusing his attention. The nurse will correctly interpret this symptom as related to the drug's effect on the brain's ability to regulate A) mood. B) thought. C) memory. D) sleep.

D) sleep.

Risk for suicide r/t...

Depressed mood Feelings of worthlessness Anger turned inwards Misinterpretations of reality

During the evaluation of the effectiveness of the nurses discharge teaching, which client report would indicate to the nurse that the client understands the leading cause of disability and decrement in health? The client reports a need to incorporate strategies to prevent:

Depression

Mild depression

Depression associated with normal grieving

First priority of a short stay patient

Develop a goal directed problem centered relationship

The nurse is admitting a client to the psychiatric unit. Which nursing action is correct?

Discuss with the client information that is to be shared with family members and the mental health team.

The nurse is planning activities to enhance collaboration within the mental health care team. Which activities will be helpful toward this goal? Standard Text: Select all that apply.

Discussion of decisions that require team unity Discussion of decisions that can be made autonomously

TEPID SPONGE BATH

Done by sponging the body with the washcloth from cold or tepid water to reduce body temperature and refresh the patient thereby promoting comfort.

Cluster B Personality Disorders

Dramatic, emotional, erratic behavior; problems with impulse control and emotional processing, and relationships; manipulations and acting out; Antisocial PD, Borderline PD, Histrionic PD, and Narcissistic PD

Side effects of verapamil

Drowsiness, dizziness, HoTN, bradycardia, nausea, constipation

Moderate depression

Dysthmia

Indications of antidepressants

Dysthymic disorder, MDD, depression associated with organic disease, alcoholism, schizophrenia, mental retardation, depressive phase of bipolar, depression accompanied by anxiety

Hypermania

Extreme mania, criteria for bipolar I

Paranoia

Extreme suspiciousness of others

Euphoric/expansive mood

Extremely happy, silly, or giddy

-CULTURE -RELIGION -ENVIRONMENT -DEVELOPMENTAL LEVEL -HEALTH AND ENERGY -PERSONAL PREFERENCES

FACTORS INFLUENCING INDIVIDUAL HYGIENIC PRACTICE

FISSURES OR DEEP GROOVES

Frequently occur between the toes as a result of dryness and cracking of the skin.

Crisis intervention: Phase 2 Planning therapeutic intervention

From data, nurse selects pertinent nursing diagnoses that reflect the immediacy of crisis situation, desired outcome is established, nursing actions selected

Euthanasia

From the Greek for painless, easy, gentle or good death, now commonly used to signify a killing prompted by a humanitarian motive.

Anger characteristics

Frowning, clenched fists, low-pitched words through clenched teeth, yelling/shouting, intense eye contact, no eye contact, easily offended, defensive, passive-agression, emotional over-control, flushed face, intense discomfort, tension

Which drug is most apt to have been ingested by a young woman who comes to the emergency department with the report that although she has no recollection of the incident, she believes she was sexually assaulted at a party? A. LAAM B. GHB C. ReVia D. Clonidine

GHB

excessive and unrealistic worry about two or more life circumstances for 6 months or longer

Generalized Anxiety disorder

A nurse acknowledges feeling anxious about meeting new people. By acknowledging feelings to the client, the nurse is demonstrating:

Genuineness.

The student nurse realizes that individuals who self-mutilate may:

Have difficulty processing feelings.

CAGE Assessment tool

Have you ever felt you ought to CUT down on your drinking?Have people ANNOYED you by criticizing your drinking?Have you ever felt bad or GUILTY about your drinking?Have you ever had a drink first thing in the morning to steady your nerves or get rid of a hang-over (EYE-OPENER)? Scoring two "yes" answers indicate a probable alcohol abuse and warrants further assessment.

ambivalence

Having two conflicting desires at the same time, such as the desire to live and to die

complementary medicine, 715

Health care and medical practices that are not currently an integral part of conventional Western medicine and are used to relieve pain and cure illnesses

Symptoms of moderate depression

Helplessness, powerlessness, slowed physical movement, slumped posture, limited verbalizations, retarded thinking process, difficulty concentrating, anorexia, overeating, sleep disturbance, headaches

Gender and depression

Higher in women than men, 2:1

expressed emotion (EE), p. 344

Highly critical, hostile, or overinvolved emotional tone

False memories

Incomplete or inaccurate memories that are the result of faulty recollection or the result of suggestion from another person

sexual abuse, p. 538

Inappropriate sexual behavior, instigated by a perpetrator, for the purpose of the perpetrator's sexual pleasure or economic gain through child prostitution or pornography

Crisis intervention: Phase 1 Assessment

Info is gathered regarding precipitating stressor and the resulting crisis that prompted the individual to seek professional help

During the first interaction with a client, the nurse makes an introduction and identifies the purpose of the interaction. This serves to accomplish which of the following in developing a trusting relationship?

Initiating

Which of the following treatment programs would be most appropriate for homeless clients whose judgment is severely impaired by paranoid delusions and command hallucinations due to medication and treatment nonadherence?

Inpatient hospital-based care

The nurse is caring for a client who was recently admitted to the unit. During the nursing assessment of the client, the nurse finds the clients beliefs and actions related to many health practices to stray from the norm. Which action would be most appropriate for the nurse to take at this time?

Inquire as to the culture with which the client identifies.

● The purpose for use of games is to provide motivation to use of games is to provide motivation to students to learn the needed information. ● They should also be sure that they have the technology hardware and software necessary to play the game ● Online instructional games can add a competitive contest aspect to learning ● Learners must be clear about the purpose when using games

Introduce some interactive games or quizzes to the classroom as a method to get students enthused about learning. This game can assist to break up the monotony of traditional teaching methods by providing students with something interactive and entertaining. Select all that apply

MEDICAL SPONGE BATH

It is done to reduce skin irritation.

PubMed

It offers several noteworthy features. It has an extensive index of published medical literature with over 22 million citations.

Habitual use may result in physical and psychological dependence with withdrawl symptoms Common adverse effects - drowsiness, sedation, lethargy, transient hypotention(important for baseline VS's, sitting/standing) Check history of blood dyscrasias or hepatic disease

Key Points on Benzodiapzepines

What is best treatment for mild depression?

LIFESTYLE AND PSYCHOTHERAPY no antidepressants

Negative symptoms

Less than normal, alogia, anhedonia, flat affect, avolition

The nurse planning a brief presentation about the first American psychiatric nurse will research which of the following?

Linda Richards

A client comes to the nurses station yelling, I have to call the FBI. The bombs are set to destroy Washington, D.C. at 1:00 p.m. Please help me. It will be your fault if I dont call. Which intervention best demonstrates the nurses sensitivity?

Listen carefully for the underlying emotion expressed by the clients request.

The nurse is working with a teen admitted with a diagnosis of depression. Which of the following interventions demonstrates that the nurse is sensitive to the clients needs?

Listening to the clients feelings

Loss

Loss may be defined as an actual or potential situation in which a a valued object, person, body part, or emotion that was formerly present is lost or changed and can no longer be seen, felt, heard, known, or experienced. A loss may be temporary or permanent, complete or partial, objectively verifiable or perceived, physical or symbolic.

-CUPPING -HACKING -PINCER LIKE OR PINCHING MOVEMENTS

MOST POPULAR METHODS OF TAPOTEMENT

tx for dependancy

MOTIVATION is the KEY to Recovery

Neologisms

Made up words that have meaning only to the person who invents them

The psychiatric nurse is reflecting on the treatment and care of the mentally ill throughout history. Which of the following philosophical beliefs most guided treatment of the mentally ill during 17th century Europe?

Madness was best overcome by discipline and brutality.

bipolar disorders, p. 366

Major mood disorders characterized by episodes of mania and depression

The nurse reflecting on the nursing role within the mental health team, understands that the main purpose of delivering care using a multidisciplinary team is to do which of the following?

Make the best use of the different abilities of mental health team members in order to facilitate client progress.

Focus on total person and emphasized health instead of illness/problems

Maslows hierarchy of needs

The nurse is teaching a group of students the various historical explanations of mental illness. Which statement by the students indicates understanding of the nurses teaching regarding the era of magico-religious explanations?

Mental and physical illness were the result of superhuman forces that inflicted suffering.

Illusion p. 185

Misperception and misinterpretation of externally real stimuli. Visual and auditory illusions are more common than tactile, olfactory, and gustatory illusions. The eye and brain are constantly being tricked into seeing things that are not what they really seem.

illusions, p. 346

Misperception and misinterpretation of externally real stimuli. Visual and auditory illusions are more common than tactile, olfactory, and gustatory illustions.

Illusions

Misperceptions of real external stimuli

Nancy is on edge. She is fidgeting, pacing back and forth and irritable. What level of anxiety is she in?

Moderate

Moderate muscle tension, Increased blood pressure, pulse, and respirations. Startle reflex Slight perspiration, Difficulty sitting still Repeated fidgeting, Periodic slow pacing, Increased rate of speech, Sporadic eye contact Emotional,Feeling uncomfortable, on edge, keyed up, Motivated to decrease anxiety Increased irritability, Decreased confidence Difficulty in concentrating, Easily distracted, can focus with assistance, Circumstantiality, Tangentiality, Loose associations, Narrowed perceptions, Decreased attention span Misperception of stimuli, Problem solving and reasoning skills with effort, or assistance

Moderate 2+ ANXIETY LEVEL

Which client with a personality disorder is *most* likely to be admitted to a psychiatric unit? A. Mr. A, with paranoid personality disorder who is suspicious of his neighbors B. Mr. B, with narcissistic personality disorder who is highly self-important C. Ms. C, with borderline personality disorder who is impulsive D. Mrs. D, with dependent personality disorder who clings to her husband

Ms. C, with borderline personality disorder who is impulsive

Toxic effects of antipsychotics

Neuroleptic malignant syndrome NMS Agranulocytosis Liver involvement

Mental Health p. 4

No overall accepted definition of mental health. In general, we consider an individual to be mentally healthy when what that person does (the person's behavior), how that person relates to others (the person interpersonal relationships between oneself and others) and how that person relates to him or herself (the person's intrapersonal relationships within the mind or the self) give evidence of psychological, emotional and social health.

- BATHING ALSO PRODUCES A SENSE OF WELL-BEING -IT IS REFRESHING AND RELAXING AND FREQUENTLY IMPROVES MORALE, APPEARANCE AND SELF- RESPECT -BATHING OFFERS AN EXCELLENT OPPURNITY FOR THE NURSE TO ASSESS ALL THE CLIENTS.

PURPOSES OF BED BATH

-TO STIMULATE THE BLOOD CIRCULATION TO THE SCALP -TO DISTRIBUTE HAIR OILS AND PROVIDE FOR A HEALTHY SHEEN -TO INCREASE THE CLIENT'S COMFORT

PURPOSES OF HAIR CARE

Judy eats to deal with stress.

Palliative

tolerance, p. 294

Physiological habituation to a substance, resulting in need for progressively greater amounts to achieve intoxication and/or a diminished effect from continued use of the same amount of the substance

Rorschach Test p. 211

Projective personality test in which 10 standardized inkblots in black and white or color on separate cards, are displayed one by one. Clients are asked to respond in terms of their associations, thoughts, and impressions.

The nurse has provided education for a patient in the continuation phase after discharge from the hospital. What indicates that the plan of care has been successful? Select all that apply. 1. Patient identifies three signs and symptoms of relapse. 2. Patient states, "My wife doesn't mind if I still drink a little." 3. Patient describes the purpose of each medication he has been prescribed. 4. Patient states, "I no longer have a disease." 5. Patient identifies two ways to problem-solve a specific situation.

Patient identifies three signs and symptoms of relapse. Patient describes the purpose of each medication he has been prescribed. Patient identifies two ways to problem-solve a specific situation.

What's the DSM criteria for substance abuse?

Pattern of drug use leading to clinically significant impairment or distress 1) inability to fulfill major role @ home/work/school 2) participating in physically hazardous situations while impaired 3) recurrent legal/interpersonal problems 4) keep using even though having problems

personality traits

Persistent behavioral patterns that do not significantly interfere with one's life even though the behaviors may be annoying or frustrating to others

Perseveration

Persistent repetition of the same word or idea in response to different questions

Interpersonal Communication p. 189

Person-to person communication that takes place in dyads (two people) or small groups.

Axis II

Personality disorders and mental retardation

Terrified of a mouse running across the room

Phobia

Is an internal fear reaction. Differ from common fear. Are obsessive in nature. People with _____handle their anxieties ineffectively. Characteristics of ____vary with the culture.

Phobias

irrational fear of a specific object, activity or situation. Social - fear of being humiliated, scrutinized or embarrassed in public Simple or specific - fear of a specific object or situation

Phobias

Muscle tension, Fidgeting, Headache, Problems with sleep, Fight-or-flight reaction

Physical symptoms of anxiety

somatoform disorders

Physical symptoms suggesting physical disorders for which there is no evidence of organic or physiologic cause; formerly referred to as psychosomatic disorders

References and call-outs

Physiologic data

(symptoms that occur 1 month or more after the trauma)Somewhat different from the other anxiety disorders, can develop after exposure to a clearly identifiable traumatic event that threatens self, others, resources and/or sense of control or hope

Post traumatic stress

Able to enjoy substance, attempt to repeat the first pleasurable experiences of the substance of choice leads to frequent pattern of use. Excessive use begins to develop, begins to ignore some responsibilities and obligations, resulting in denial of problem.Ignores others comments. Lies to cover up activity Conceals, sneaks drinking or doses

Pre-Dependent (Early stage):

The nurse is planning care for a patient experiencing the acute phase of mania. Which is the priority intervention? 1. Prevent injury. 2. Maintain stable cardiac status. 3. Get the patient to demonstrate thought self-control. 4. Ensure that the patient gets sufficient sleep and rest.

Prevent injury.

most effective way to cope with anxiety

Prevention

Projective Personality Test's p. 211

Psychological tests that involve presenting the client with a somewhat ambiguous stimulus, often a visual one, to which the client responds with an idiosyncratic perception

Objective Personality Tests p. 211

Psychological tests that provide data on various aspects of the client's personality which are scored or analyzed using empirically derived criteria

Crisis intervention: Phase 4 Evaluation of crisis resolution and anticipatory planning

Reassessment is conducted to determine whether the stated objectives were achieved, plan of action developed to cope with stressor in the future

Certification p. 24

Recognition of competency by a professional organization; a way of protecting consumers

Individual choice (suicide)

Recognition that in the end it was their own choice

Recognition (suicide)

Recognizing that it exists and it is a real problem

The nurse assesses that the mental health client has problems choosing productive, safe leisure activities. Which member of the mental health team should the nurse consult with?

Recreational therapist

A client and his wife are visiting in the day room of an inpatient psychiatric unit when a client with acute psychosis experiencing auditory hallucinations sits down next to them and begins talking to the voices. Which of the following nursing actions is most appropriate?

Remind the client with psychosis to respect the privacy of the other client and his wife.

Learned helplessness

Repeated failure to control life, leading to feelings of helplessness and dependence on others **predisposes the individual to depression**

substance abuse, p. 294

Repeated use of a substance despite significant and repeated negative substance-related consequences

Echopraxia

Repeating movements that are observed

Echolalia

Repeating words that are heard

A victim of incest no longer remembers why she always hated the uncle who molested her

Repression

Learner-learner

Students interact with each other to provide an exchange of ideas with different perspectives, which may reinforce understanding of what is being taught in course.

substance intoxication, p. 294

Reversible substance-specific changes in thinking, emotions, behavior, and/or physiological functions caused by recent substance ingestion

Nursing diagnosis for bipolar disorder

Risk for injury, risk for violence, imbalanced nutrition less than body requirements, disturbed thought processes, disturbed sensory perception, impaired social interaction, insomnia

Nursing diagnosis for depression

Risk for suicide, complicated grieving, low self-esteem, powerlessness, spiritual distress, social isolation/impaired social interaction, disturbed thought process, imbalanced nutrition, insomnia, oversleeping, self-care deficit

The nurse is planning care for an Asian client who is Buddhist. Which of the following actions is most important for the nurse to take to provide culturally relevant mental health care?

Seek clarification of this clients health beliefs.

Preparation of body for fight-or-flightExtreme muscle tension, Increased perspiration Continuous and rapid pacing, Reflex responses Loud or rapid speech, or both, Poor eye contact Somatic symptoms, Sleep disturbances Extreme discomfort, Feeling of dread Hypersensitivity, Defensiveness with threats and demand, Distorted perceptions Difficulty focusing, even with assistance Flight of ideas, Ineffective reasoning and problem-solving skills, Disorientation Delusions and hallucinations, if prolonged Suicidal or homicidal ideations, if prolonged

Severe 3+ ANXIETY LEVEL

Munchausen syndrome

Severe, chronic, and unremitting form of factitious disorder involving repeated hospitalizations, traveling between health care providers and health care facilities, and pathological lying of an intriguing and fantastic nature (termed pseudologica fantastica)

Mary, a patient diagnosed with schizophrenia, is encouraged to attend groups but stays in her room instead. Staff and peers encourage her participation, but her hygiene remains poor. She does not seem to care that others wish that she would behave differently. Which is the most likely explanation for Mary's failure to respond to others' efforts to help her behave in a more adaptive fashion? Select all that apply. 1. She is avolitional. 2. She is displaying anergia. 3. She is displaying negativism. 4. She is exhibiting paranoid delusions. 5. She is being resistant or oppositional. 6. She is experiencing social withdrawal. 7. She is apathetic due to her schizophrenia.

She is avolitional. She is displaying anergia. She is displaying negativism. She is experiencing social withdrawal. She is apathetic due to her schizophrenia.

Naivete (suicide)

Shocked by methods, general shock about suicide, feel you don't know enough about it to understand client

A psychiatric home care nurse knocks on the clients door for the first time and is told by the client, Go away and leave me alone. I dont want to buy any Girl Scout cookies. What should the nurse do in this instance?

Show the client identification and explain the purpose of the visit

The nurse educator is teaching a group of students about psychiatricmental health nursing concepts. Which intervention best demonstrates practicing with the concept known as detached concern?

Sitting quietly with a client who is sobbing uncontrollably

group therapy, p. 632

Six to eight members come together to learn about themselves and their relationships with others as a means of improving their mental health.

EXCESSIVE DRYNESS

Skin can appear flaky and rough

relates to clients' perceptions. The experiences of pain, nausea, and anxiety cannot be measured by anyone other than the individual who is experiencing them.

Subjective data

Treatment approaches that use physiologic or physical interventions to effect behavioral change.

Somatic therapies. Examples; Drug treatment therapy Electroconvulsive treatment therapy, Biofeedback Phototherapy, Acupuncture

body dysmorphic disorder (BDD)

Somatoform disorder characterized by preoccupation with a defect (imagined or exaggerated) in physical appearance

factitious disorder by proxy

Sometimes called Munchausen by proxy syndrome; occurs when parents or caregivers deliberately induce signs of an illness in another person, usually their own child

A nurse is working with a client who was admitted for treatment of bipolar disorder. The client asks the nurse if it is OK to pray for recovery. Which response best conveys the nurses ability to be a spiritual activist for the client?

Spiritual practices, such as praying, can nurture ones spirit and enhance healing.

ALCOHOL SPONGE BATH

Sponging the body with washcloth wrung from a mixture of alcohol and ordinary tap water.

2 groups of antipsychotics

Standard- traditional dopamine antagonists Atypical- serotonin-dopamine antagonists Treatment of choice is individualized based on the adverse side effects

factitious disorder

State characterized by symptoms (psychological or physical) that are intentionally produced in order to assume the sick role and experience subsequent benefits

dissociative fugue

State in which the individual is unable to recall one's past; often has sudden, unexpected travel away from routine environment; the flight may be accompanied by identity confusion or assumption of a new identity

Cocaine exerts which of the following effects on a client? A. Stimulation after 15 to 20 minutes B. Stimulation and anesthetic effects C. Immediate imbalance of emotions D. Paranoia

Stimulation and anesthetic effects

What are various roles the nurse takes on in "the helping relationship"?

Stranger, resource person, teacher, leader, technical expert, surrogate, counselor

The nurses new job description at the generalist level of practice reflects the definition of psychiatricmental health nursing and the PsychiatricMental Health Nursing Standards of Practice (ANA, APNA, ISPN). In which of the following areas might the nurse plan programs and intervention to fulfill employment expectations? Standard Text: Select all that apply.

Stress management strategies Parenting classes for new parents Family and group psychotherapy Medication teaching for anti-anxiety medications

The client on the psychiatric unit is asking questions about prevention of sexually transmitted diseases. Given the PsychiatricMental Health Nursing Standards of Practice, which action would be most appropriate for the nurse to take at this time?

Teach safer sexual practices.

progressive relaxation, 724

Technique based on the premise that muscle tension is the body's physiological response to anxiety-provoking thoughts.

SULCULAR TECHNIQUE

Technique recommended for brushing teeth is called _______ which removes plaque and cleans under the gingival margins

PYORRHEA

Teeth are loose and pus is evident when gums are pressed.

DENTAL CARIES

Teeth have darkened areas, may be painful

During a nurseclient interaction, the client tells the nurse, I dont think I can deal with feeling so sad much longer. The nurses best response is which of the following?

Tell me about your feelings of sadness.

What neurophysiological disorders are implicated in episodic aggression and violence?

Temporal or frontal lobe epilepsy, brain tumors, brain trauma, encephalitis

the arms, chest, abdomen, legs, and feet

The body parts in which are omitted in partial bed bath are the following:

the face, hands, axillae, perineal area, and back

The body parts in which needed to be washed in partial bed bath are the following:

CEMENTUM

The bony tissue that covered the jaw and the root of the tooth

A client was in an automobile accident. Although he has the odor of alcohol on his breath, his speech is clear and he is alert and answers questions posed to him. The law enforcement officer requests that the emergency department staff draw a blood sample for blood alcohol level determination. The level is determined to be 0.30 mg%. What conclusion can be drawn? A. The client has a high tolerance to alcohol. B. The client ate a high-fat meal before drinking. C. The client has a decreased tolerance to alcohol. D. No conclusions can be drawn from the data.

The client has a high tolerance to alcohol.

The nurse is sharing client assessment data with the multidisciplinary health care team. Which comment by the nurse is irrelevant and indicates a misunderstanding of the concept of a mental disorder?

The client has some very inappropriate religious ideas and spiritual beliefs.

Affect

The feeling or emotional state

ego-dystonic

The feelings experienced by people with depersonalization disorder are ego-dystonic, meaning that they are unacceptable to the person's sense of self.

End of life

The final days or weeks of life when death is imminent.

The nursing assistant asks the psychiatric nurse the location of the first asylum for the mentally ill. Which response by the nurse is most appropriate?

The first asylum for the mentally ill was in Morocco.

Distress p. 5

The person's psychological suffering.

What factors help predict the stress response?

The precipitating event, the individuals perception of the event, and the predisposing factors

Mental disorder due to a general medical condition

The presence of mental symptoms that are judged to be the direct physiological consequence of a general medical condition

Bereavement

The time of mourning experienced after a loss.

What's the difference between a professional and social relationship?

The roles and boundaries pertaining to the two individuals involved

HYGIENE

The science of health and its maintenance

Spirituality p. 42

The search for meaning and purpose in life through a connection with others, nature and/or a belief in a higher power.

APPLICATION OF OILS

The skin is crusty, flaky, and dry underneath the cast, what usually necessary thing to do to remove the debris from the cast?

Geropsychiatry, 602

The treatment of psychiatric problems in elders.

Virtual Reality

The use of technology is to allow the participant to exist in another reality, experiencing an event that although it appears real does not physically exist. The objective is to create scene in which the participants is free to concentrate on the tasks, problems, and ideas that he or she would face in the real situation

may become noncompliant with medication. Which factor would be of least concern to the nurse developing a psychoeducation plan to foster compliance? A. The side-effects are unpleasant B. The voices tell the client to stop taking it C. The client prefers to feel "high" and energetic D. The client feels well and denies the possibility of recurrence

The voices tell the client to stop taking it

AMBULATORY CLIENTS

These are the clients that prefers to take partial bath at the sink.

Which statement is descriptive of clients with personality disorders? A. They are resistant to behavioral change. B. They have an ability to tolerate frustration and pain. C. They usually seek help to change maladaptive behaviors. D. They have little difficulty forming satisfying and intimate relationships.

They are resistant to behavioral change.

What do all people who commit suicide have in common?

They are unable to see alternative solutions

Which of the following would be considered a disadvantage of Assertive Community Treatment (ACT) programs?

They control access to resources such as housing and money.

I will read the record and talk with the physician to understand the clients disabilities.

This assignment may be a challenge for me and I will need to be aware of my feelings and any potential fears related to caring for this client.

BAG BATH

This bath is commercially prepared product that contains 10-12 presoaked disposable washcloths that contain no-rinse cleanser solution.

Learner - Instructor

This type of interaction is used by student to reinforce understanding of course content and clarify nebulous points.

schizoaffective disorder, p. 340

Two sets of symptoms present concurrently in the same illness episode: symptoms of schizophrenia and symptoms of a mood disorder (either a major depressive or manic disorder)

schizophrenia disorganized type, p. 339

Type of schizophrenia with disorganized speech and behavior, and flat or inappropriate affect; considered the most disabling form of the disease

Tutorials

Well-designed tutorials are interactive when the learner is presented with learning content and then provided with self-assessment multiple choice questions to assess the learning.

Simulation

What learning methods in applying technology that provides essential knowledge for the learner such as assessing and taking blood pressure? Wherein, the student uses a mouse to pump the virtual blood pressure bulb and then releases the valve to hear the sounds

Knowledge

What level of learners is the focus of the online educational activities?

Learning Management System (LMS)

What term describes the software that facilitates delivering course content electronically?

A nurse is taking a class on providing culturally competent care for clients with severe mental illnesses. Which response best reflects the nurses self-awareness of the sociocultural factors influencing his or her beliefs?

When I was growing up, my parents believed that mental illnesses were the work of evil spirits.

Transference

When a client takes a feeling they have and projects it onto the nurse (negative or positive)

When planning care for a specific client, of what significance to the psychiatric nurse is the fact that the DSM-IV-TR is multiaxial? a) Pertinent aspects of client functioning and problems are reported. b) Standardized treatment plans are available for each diagnosis. c) Nursing diagnoses are included for each medical diagnosis. d) No particular significance exists.

a) Pertinent aspects of client functioning and problems are reported.

It is a theory that indicates events in our childhood have a great influence on our adult lives, shaping our personality. a) Psychoanalytic Model/Psychodynamic Framework b) Psychosexual Development c) Psychosocial Model

a) Psychoanalytic Model/Psychodynamic Framework by Freud

It is a group therapy where the members will learn their behavior & make positive changes in their behavior a) Psychotherapy group b) Family Therapy c) Support Groups d) Educational Groups e) Self Help Groups

a) Psychotherapy group

these are drugs that are typically used as antidepressants in the treatment of major depressive disorder, anxiety disorders, and other psychological conditions. a) SSRI b) Antihistamine c) Benzodiazepine

a) SSRI

It limits the severity of a disorder. Examples: Screening & case finding Crisis intervention Prompt treatment - administration of medication Hospitalization a) Secondary b) Tertiary c) Primary

a) Secondary

Concentrating on a single detail while ignoring others a) Selective abstraction b) Arbitrary inference c) Over generalization

a) Selective abstraction

-Uses senses & motor abilities to understand the world -Egocentric -Primary Circular Reaction =infants begin to display behavior (cry-hungry, wet) -Object permanence = awareness of independence of his own actions and perceptions a) Sensorimotor b) Pre-Operational c) Formal operations

a) Sensorimotor

The first scientist to identify 'stress' as underpinning the nonspecific signs and symptoms of illness. He believed that "stress response occurs whenever a person encounters continuous stress" a) Seyle b) Freud c) Erickson

a) Seyle

It is a role of a nurse that assists the patient to feel comfortable with others a) Socializing Agent b) Counselor c) Ward-Manager

a) Socializing Agent

what type of phase that the body attempts to reduce damage from the stressor a) Stage of Resistance b) Stage of Exhaustion c) Alarm Reaction

a) Stage of Resistance

What is the new treatment for Alzheimer's disease? a) Tacrine b) Clonazepam c) Lithium

a) Tacrine

It is a role of a nurse that assists the patient to learn more adaptive ways of coping a) Teacher b) Counselor c) Patient Advocate

a) Teacher

It is a role of a nurse that explores the patient's needs, problems & concerns through varied therapeutic means a) Therapist b) Technician c) Reality - Based Model

a) Therapist

It is a antidepressant which causes priapism a) Trazodone b) Nefazodone c) Mirtrazapine

a) Trazodone

It is a type of antipsychotic drug that controls positive symptoms of Schizophrenia. a) Typical Antipsychotics b) Atypical Antipsychotics

a) Typical Antipsychotics

Resulting from experiences related to increase anxiety & leading to anxiety state a) Bad me b) Good me c) Not me

a) bad me

If a hypertensive crisis occurs when taking MAOIs, what should the nurse do? a) discontinue the medication and give Phentolamine and tolazoline. b) discontinue the med and call the physician c) discontinue the med and wait the BP to lower down

a) discontinue the medication and give Phentolamine and tolazoline.

Many antipsychotic medications, both "first generation" agents like chlorpromazine and "second-generation" agents such as clozapine, have anticholinergic properties. Anticholinergic effects would be dry mouth and thirsty. What is the management for this? a) give a sugar free hard candy or gum b) give patient a juice c) give patient an ice chip

a) give a sugar free hard candy or gum

Learns to relate to peers, learns competition, compromise & cooperation Turns to peers of the same sex to provide the sense of satisfaction Period of gang loyalties/ gang formation Acquires important interpersonal tools - ability to compete, compromise & values of loyalty a) juvenile - schooler b) infancy c) toddlerhood

a) juvenile

When taking doxepin (antidepressant), what should be monitored? a) monitor the BP b) monitor RR c) monitor temperature

a) monitor BP

Known as Later Childhood Characterized by Consensual Validation- learns to use words to communicate & checkout his perceptions & feelings with others Behavioral traits: Love to watch adults & imitate their behavior, very creative & curious (favorite word "WHY"), love to tell lies, to brag & boast in order to impress others, very imaginative, imaginative playmates are common, love offensive language, question about sex should be answered honestly at the level of their understanding. a) preschool b) toddlerhood c) infancy

a) preschool

If the patient is experiencing dystonia, what is your management? a) remain with the client and administer antiparkinson's drug b) avoid abrupt withdrawal and administer Anti-parkinson's drug c) no treatment except discontinue meds

a) remain with the client and administer antiparkinson's drug

If the patient is experiencing akathisia, what is your management? a) rule out anxiety or agitating before administering antiparkinson's drug b) none of the above c) no treatment except discontinue meds

a) rule out anxiety or agitating before administering antiparkinson's drug

It is an important neurotransmitter that plays a role in brain functions, such as memory, and body functions, such as muscle contractions to move your muscles. Treatment for Alzheimer's and Parkinson's disease. a) Monoamine Oxidase b) Acetylcholine c) GABA

b) Acetylcholine

-Pleasure through elimination & retention of feces -Behavior: control of holding on & letting go -Develops concept of power, punishment, ambivalence, concerned with cleanliness & being dirty -Break symbolic ties with mother, as the ties are broken, the child learns independence -Exhibits motor self-control & independence through negativistic behavior -Toilet training is achieved: Bowel control-18 mos, Daytime bladder control-2 ½, Nighttime bladder control- 3 yrs. What stage is this? a) Oral Stage b) Anal Stage c) Phallic Stage

b) Anal Stage

These are drugs that reduce anxiety and control or manage the symptoms of alcohol withdrawal. a) Antidepressants b) Anxiolytics c) Antipsychotics

b) Anxiolytics

Holding beliefs in absence of supporting evidence a) Selective abstraction b) Arbitrary inference c) Over generalization

b) Arbitrary inference

It is a type of antipsychotic drug that controls positive and negative symptoms of Schizophrenia. a) Typical Antipsychotics b) Atypical Antipsychotics

b) Atypical Antipsychotics

-focus on the modification of overt symptoms without regard to patient's experience or inner conflicts -based on the premise that behavior & responses are learned & may be relearned a) Brief Cognitive Therapy b) Behavior Therapy c) Brief Interpersonal Therapy

b) Behavior Therapy

Focuses on the cognitive processes which includes expectations, beliefs & memories & thinking patterns which influence behavior & feelings a) Psychosocial Theory b) Cognitive Theory c) Psychodynamic Theory

b) Cognitive Theory

Aware of here and now / Existence of ones current awareness a) Unconscious b) Conscious c) Subconscious/Preconscious

b) Conscious

It is a role of a nurse that listens to the patient's verbalization a) Ward Manager b) Counselor c) Patient Advocate

b) Counselor

It is a chemical released in the brain that makes you feel good and a treatment for schizophrenia. a) Norepinephrine b) Dopamine c) Serotonin

b) Dopamine

It is a type of group therapy where the client & their family participates for the goal is to understand how family dynamics contribute to the client's psychopathology, mobilizing the family's inherent strength & resources, restructure the family's maladaptive behavior styles, strengthen family's problem solving behavior. a) Psychotherapy group b) Family Therapy c) Support Groups d) Educational Groups e) Self Help Groups

b) Family Therapy

What type of food should avoid when taking MAOIs? a) Food rich in protein b) Food rich in tyramine c) Food rich in vitamin A

b) Food rich in tyramine

He is the father of psychoanalysis, stressed that early childhood experiences is important in the development of personality. a) Erickson b) Freud c) Watson

b) Freud

Sexual pleasure through genitals Behavior: Becomes independent from parents, responsible to self Develops sexual identity, ability to love & work What stage is this? a) Oral Stage b) Genital Stage c) Phallic Stage

b) Genital Stage

unconscious part of the person which serves as the reservoir of primitive & biologic drives & urges. Part of personality in which we are born, primitive & demands immediate satisfaction, functions according to pleasure principle & developed during infancy. a) Ego b) Id c) Superego

b) Id

❒ Method of bringing out change in a client by exploring their feelings, attitude, thinking & behavior ❒ involves one-to-one relationship between the therapist & the client which proceeds in three stages: introductory, working & termination phase a) Group Therapy b) Individual Therapy c) Somatic Therapy

b) Individual Therapy

what type of factor influences mental illness that includes ineffective communication, excessive dependency on or withdrawal from relationships, no sense of belonging, inadequate social support, and loss of emotional control. a) Individual b) Interpersonal c) Social/Cultural

b) Interpersonal

what type of factor influences mental health that includes effective communication, ability to help others, intimacy, and a balance of separateness and connectedness. a) Individual or Genetic b) Interpersonal or Childhood Experiences c) SOCIAL/CULTURAL/LIFE CIRCUMSTANCES

b) Interpersonal or Childhood Experiences

It is a procedure where 2 holes are drilled on the frontal aspect of the skull. Then, a 2cc of absolute alcohol is introduced at the brain's central ovale. a) Colostomy b) Lobotomy c) Craniotomy

b) Lobotomy

Greatly exaggerating a situation a) Minimization b) Magnification c) Dichotomous

b) Magnification

It affects how we think, feel, and act. It helps determine how we handle stress, relate to others, and make choices a) Health b) Mental Health c) Mental Illness d) Psychology

b) Mental Health

It is a side effect of the antipsychotic drug which the patient is doing an involuntary movements of the tongue, lips smacking, face, trunk, and extremities. a) Parkinson's Syndrome b) Tardive Dyskinesia c) Dystonia

b) Tardive Dyskinesia

It is a role of a nurse that facilitates the performance of nursing procedures a) Teacher b) Technician c) Healthy Role Model

b) Technician

Are the final changes in the clinical symptom induced by a drug such as stabilization of anxiety & depression a) Primary effects b) Tertiary effects c) Secondary effects

b) Tertiary effects

What are the side effects of atypical antipsychotic drugs, except for this one: a) Diabetes b) Weight Loss c) Decreased Libido

b) Weight Loss

Learns independence & how to relate to opposite sex Begins to experience sexual urges - LUST Development of heterosexual relationship a) juvenile - schooler b) adolescence c) preadolescence - schooler

b) adolescence

These are psychotropic drugs that are used to treat symptoms of mania. a) Antidepressants b) Antimanic c) Anticonvulsants

b) antimanic

MAOI examples

isocarboxazid (Marplan) phenelzine (Nardil) tranylcypromise (Parnate) selegiline transdermal system (Emsam)

-Pleasure through genitals -Girl: Elektra complex or attraction to father -Boy: Oedipal complex or attraction to mother -Behavior: Masturbation, touching of genitals -Because of desire to possess parent of opposite sex, the child develops guilt feelings & fear of punishment by parent of the same sex (castration complex) -To gain the affection of the parent of the opposite sex, the child imitates the parent of the same sex, this leads to the internalization of the traits of the parent the child is identifying with. What stage is this? a) Oral Stage b) Anal Stage c) Phallic Stage

c) Phallic Stage

It is a role of a nurse enables the patient to distinguish objective reality & subjective reality a) Teacher b) Healthy Role Model c) Reality Based Model

c) Reality Based Model

what type of factor influences mental health that includes a sense of community, access to adequate resources, intolerance of violence, support of diversity among people, mastery of the environment, and a positive, yet realistic, view of one's world. a) Individual or Genetic b) Interpersonal or Childhood Experiences c) SOCIAL/CULTURAL/LIFE CIRCUMSTANCES

c) SOCIAL/CULTURAL/LIFE CIRCUMSTANCES

It is a monoamine neurotransmitter which its biological function is complex and multifaceted, modulating mood, cognition, reward, learning, memory, and numerous physiological processes such as vomiting and vasoconstriction. a) Norepinephrine b) Dopamine c) Serotonin

c) Serotonin

what type of factor influences mental illness that includes lack of resources, violence, homelessness, poverty, an unwarranted negative view of the world, and discrimination such as stigma, racism, classism, ageism, and sexism. a) Individual b) Interpersonal c) Social/Cultural

c) Social/Cultural

part of the mind in which ideas & reactions are stored & partially forgotten. Thoughts & experiences can be recalled at will. Manifested during the "tip of the tongue" experience. a) Unconscious b) Conscious c) Subconscious/Preconscious

c) Subconscious/Preconscious

-"the conscience" or the censoring force. -Developed during preschool age or about 7 years of age. -Incorporates standards, restrictions, taboos, ideals imposed by parents & other individuals with whom the child associates with. a) Ego b) Id c) Superego

c) Superego

It is a type of group therapy which the members help other members who share a common problem and help cope with the problem. a) Psychotherapy group b) Family Therapy c) Support Groups d) Educational Groups e) Self Help Groups

c) Support Groups

If the patient is precipitating seizures due to abrupt withdrawal of benzodiazepine drugs, what will you do? a) call the physician b) administer higher dose of benzodiazepine c) administer lower dose of benzodiazepine

c) administer lower dose of benzodiazepine

These are anxiolytic drugs except for this one: a) Lorazepam b) Chlordiazepoxide c) Haloperidol

c) haloperidol

-dependent on others to meet needs -Self-concept development -Solitary play a) preschool b) toddler c) infancy

c) infancy

If the patient is experiencing tardive dyskinesia, what is your management? a) Withdraw the meds and call the physician. b) avoid abrupt withdrawal and administer Anti-parkinson's drug. c) no treatment except discontinue meds

c) no treatment except discontinue meds

Results from very disapproving message & leading to overwhelming anxiety a) Bad me b) Good me c) Not me

c) not me

refers aggregate of the physical & mental qualities of the individual as these interact in characteristic fashion with his environment. a) Social b) Behavior c) Personality

c) personality

Avoidant PD

characteristics are an extreme sensitivity to rejection and robust avoidance of interpersonal situations. These individuals demonstrate poor self-confidence and are prone to misinterpreting others' feedback because they are overly sensitive to rejection.

anger

is a universal emotional response to a threat or feeling of powerlessness and a common automatic response to anxiety.

restraint

is any device attached to a person's body that cannot be easily removed by the person; the device restricts the individual's freedom of movement. Is a measure of last resort that is used with caution.

An outcome for a manic client during the acute phase that would indicate that his treatment plan was successful would be that the client A. reports racing thoughts. B. is free of injury. C. is highly distractible. D. ignores food and fluid.

is free of injury.

lithium carbonate

is used to treat some forms of depression and manic episodes of manic-depressive disorder

That part of the consciousness that perceives, sorts, and combines information. People with a clear ______ are oriented to time, place, and person.

sensorium

A client has reached the stable plateau phase of schizophrenia. An appropriate clinical focus for planning would be A. safety and crisis intervention. B. acute symptom stabilization. C. stress and vulnerability assessment. D. social, vocational, and self-care skills.

social, vocational, and self-care skills

Playing one staff member against another is an example of A. devaluation. B. splitting. C. impulsiveness. D. social ineptitude.

splitting.

It is likely that a client with seasonal affective disorder will begin to feel better in the A. fall. B. winter. C. spring. D. summer.

spring.

Discontinuation syndrome

stopping SSRIs abruptly (esp paroxetine and sertraline); dizziness, nausea, vomiting, lethargy, flu-like sx's, sensory/sleep disturbances

clang associations

stringing together words that sounds alike or rhyming without regard to their meaning.

A depressed client tells the nurse "There is no sense in trying. I am never able to do anything right!" The nurse can best begin to attack this cognitive distortion by A. suggesting "Let's look at what you just said, that you can 'never do anything right.'" B. querying "Tell me what things you think you are not able to do correctly." C. asking "Is this part of the reason you think no one likes you?" D. saying "That is the most unrealistic thing I have ever heard."

suggesting "Let's look at what you just said, that you can 'never do anything right.'"

A nursing intervention designed to help a schizophrenic client manage relapse is to A. schedule the client to attend group therapy. B. teach the client and family about behaviors associated with relapse. C. remind the client of the need to return for periodic blood draws. D. help the client and family adapt to the stigma of chronic mental illness.

teach the client and family about behaviors associated with relapse.

The term tolerance, as it relates to substance abuse, refers to A. use of a substance beyond acceptable societal norms. B. the additive effects achieved by taking two drugs with similar actions. C. the signs and symptoms that occur when an addictive substance is withheld. D. the need to take larger amounts of a substance to achieve the same effects.

the need to take larger amounts of a substance to achieve the same effects.

The clients treatment plan includes teaching related to possible side effects of psychotropic medications. Which member of the mental health team should plan to implement the teaching?

the nurse

To plan care for a manic client the nurse must consider that lithium cannot be started until A. the physical examination and laboratory tests are analyzed. B. the initial doses of antipsychotic medication have brought behavior under control. C. seclusion has proven ineffective as a means of controlling assaultive behavior. D. electroconvulsive therapy can be scheduled to coincide with lithium administration.

the physical examination and laboratory tests are analyzed.

Here-and-now p. 56

the processes and dynamics at that very moment in the interaction, helps to recognize what is going on in the moment.

When the wife of a manic client asks about genetic transmission of bipolar disorder, the nurse's answer should be predicated on the knowledge that A. no research exists to suggest genetic transmission. B. much depends on the socioeconomic class of the individuals. C. highly creative people tend toward development of the disorder. D. the rate of bipolar disorder is higher in relatives of people with bipolar disorder.

the rate of bipolar disorder is higher in relatives of people with bipolar disorder.

alcohol Withdrawal can occur....

within 8 - 12 hrs hours

An acute phase nursing intervention aimed at reducing hyperactivity is redirecting the client to A. write. B. exercise. C. direct unit activities. D. orient a new client to the unit.

write.

seclusion protocol

written order, restraints used, reviewed and rewritten every 24 hr. Used to reduce stimuli, protection of client from self/others, protects destruction of personal property. Only used when other measures have failed


Set pelajaran terkait

Film exam 2 short answer questions

View Set

Nursing in Nutrition Chapter 19 diabetes mellitus

View Set

Basic Electricity (Test Guide Version)

View Set

AP World History Unit 4 Progress Check

View Set

Environmental Science A - Habitats (4)

View Set

Human Development, Section Eight: Middle Adulthood

View Set

prenatal health and individual development

View Set

соціологія | модуль

View Set